Financial-Management-1-munotes

Page 1

1 1
INTRODUCTION TO FINANCIAL
MANAGEMENT
Unit Structure
1.0 Objective
1.1 Meaning of Business Finance
1.2 Financial Management – Meaning, Scope, Objectives and Functions.
1.2.1 Meaning of Financial Management
1.2.2 Scope of Financial Management / Different d ecisions under
Financial Management
1.2.3 Objectives of Financial Management
1.3 Functions of Financial Management
1.4 Introduction to the Indian Financial System
1.4.1 Meaning and features of our financial system
1.4.2 Components of a financial system – Institutions, Markets and
Assets
1.4.3 Financial Institutions
1.4.4 Financial Assets
1.4.5 Financial Services
1.4.6 Financial Markets
1.5 Regulatory aspects of Indian Financial System
1.5.1 Introduction to regulations
1.5.2 Regulators in India
1.5.3 Securities and Exchange Board of India (SEBI)
1.5.4 RBI
1.5.5 IRDA
1.6 Practice Questions

munotes.in

Page 2


Financial Management
2 1.0 OBJECTIVE
After going through this chapter, the learner will be able to;
1. Understand the meaning of business finance.
2. Explain the meaning of Financial Man agement
3. Understand the functions of Financial Management
4. Understand in detail the different decisions taken under Financial
management
5. Analyze the Indian financial system and evaluate it’s functions.
6. Evaluate different financial assets, intermediaries and the regulatory
system.
1.1 MEANING OF BUSINESS FINANCE
Let us say that you come across a news where a company is looking
forward to acquire another company overseas. The company is trying hard
to arrange finances in the form of debt. There are several key deci sions to
be made will put the very future of the company at stake. These choices
necessitate careful budgeting, knowledge of the resulting capital structure,
and awareness of the enterprise's profitability and risk. All of these affect
both employees and s hareholders. They require knowledge of the
business's financial operations, key areas for financial decision -making,
financial risk, and working capital requirements. We are all aware of how
important finance is to operating a business. The efficiency with which
funds are allocated to assets and operations, as well as how quickly and
affordably they are provided from outside or within the company,
determines how successful a corporation will be.
According to B.O. Wheeler Meaning of Business Finance includes those
business activities that are concerned with the acquisition and
conservation of capital funds in meeting the financial needs and overall
objectives of a business enterprise.”
Business is defined as the creation and distribution of goods and services
to meet societal demands. Businesses need money, or what is known as
business finance, in order to conduct any operation properly. Be a result,
money is sometimes referred to as a business's lifeblood. Without
sufficient money readily available for use, a firm could not operate. The
amount of money the businessman invested to start the company is
insufficient to cover its expenses. As a result, the businessman must look
for a way to raise money. In order to achieve successful financial
management and keep the firm operating, it is necessary to conduct
research on the financial demands and available solutions to meet those
needs.
The basic requirements of a business would be to purchase a facility or
equipment, or it may be to purchase raw materials, build a business that
results in additional enrollments, pay workers, and so forth.
munotes.in

Page 3


Intro duction to Financial
Management
3 The following categories apply to the financial requirements of a business:
Fixed Capital Requirement: Money is needed to purchase fixed assets,
such as land, buildings, plants, and machinery, in order to launch a firm.
The Fixed Capital Requirement is what we refer to as.
Working capital needs: A firm needs money for daily operations. The
term "working capital requirements" refers to this. For the acquisition of
raw materials, p aid salaries, wages, rent, and taxes, working capital is
necessary.
Diversification: A business requires additional money to expand its range
of operations which are viatl for its growth. A company would like to add
several verticals to its business and ev entually dream of conducting
operations across several countries.
Technology upgrade: It costs money to adopt the newest technology,
such as using certain software and the newest computers in the workplace.
1.2 FINANCIAL MANAGEMENT – MEANING, SCOPE,
OBJECT IVES AND FUNCTIONS.
1.2.1 Meaning of Financial Management
Planning, arranging, managing, and controlling financial activities, such as
the acquisition and use of an organization's funds, is known as financial
management. It entails applying general managem ent ideas to the
company's financial resources.
1.2.2 Scope of Financial Management / Different decisions under
Financial Management
1. Investment decisions: These decisions involve buying fixed assets
(called as capital budgeting). Investment choices referre d to as
working capital choices can include investments in current assets.

2. Financial decisions: These decisions are related to raising money
from a variety of sources, and they will rely on the choice of source,
length of time for financing, cost of finan cing, and returns obtained
from financing.

3. Dividend decisions: A decision about the distribution of net profits
must be made by the finance manager. Net profits are typically split
into two categories:

a. Dividend for shareholders —A dividend must be declare d, along with
its rate.

b. Retained earnings: The amount of retained profits must be decided,
and it will be based on the company's plans for growth and
diversification.
munotes.in

Page 4


Financial Management
4 1.2.3 Objectives of Financial Management
Financial Management plays an important role i n the overall business
management. Money is the life blood of any organization and financial
management helps the management with key decision maing skills for
optimum utilization of monetary resources. The purchase, allocation, and
control of a concern's financial resources are within the purview of
financial management. The goals can include;
1. To guarantee a consistent and sufficient flow of funding to the
organization so that the organization never runs out of money. This is
also known as liquidity manage ment. Most businesses fail when they
run out of money.

2. To make sure that shareholders receive acceptable returns, which will
rely on their earning potential, the share's market price, and their
expectations. Hence financuiual management ensures that adequ ate
returns are generated through sound decisions.

3. To guarantee optimal use of the budget. Once funds have been
secured, they should be used as efficiently and effectively as feasible.
As such, funds are always available in limited quantities and always
come with a cost. Managing it wisely and reducing money leakages is
an important task.

4. To assure safety on investment, money should be put into safe
endeavors in order to get a sufficient rate of return. This involves
several elements of project management including business and project
evaluation to check its financial viability.

5. To design a reliable capital structure:To maintain a balance between
debt and equity capital, there should be a healthy and fair composition
of capital. This is very much importa nt as financial stability to a great
extent relies on the soundness of the capital structure.
1.3 FUNCTIONS OF FINANCIAL MANAGEMENT
1. Calculating the company's capital needs : A finance manager must
calculate the capital needs of the business. This is depe ndent on a
concern's future programmes, policies, and projected expenses, profits,
and gains. Estimates must be created in a way that increases the
enterprise's earning potential.
2. Determining the capital composition: Following estimation, the
capital st ructure must be chosen. Debt equity analysis, both short - and
long-term, is involved in this. This will depend on how much equity
capital a company currently has and how much money has to be raised
from other sources.
3. Choice of sources of financing: A corporation can choose from a
variety of options to raise additional funds, including; munotes.in

Page 5


Intro duction to Financial
Management
5 a. the issuance of shares and debentures.
b. Loans to be obtained from financial institutions and banks
c. Public deposits that will be drawn, through the issue of corporate
bonds and debentures.
The choice of factor will depend on the relative strengths and drawbacks
of each finance source and time period.
4. Investment of funds: To ensure investment safety and regular returns,
the finance manager must cho ose to deploy funds to successful initiatives.
5. Surplus disposition: The financial manager must decide how to use the
net gains. There are two ways to do this:
a. Dividend declaration - It lists the rate of dividends as well as other
advantages, suc h as bonuses to be given.
b. Retained profits - The amount must be chosen, and it will rely on the
company's expansion, innovation, and diversification objectives.
6. Cash management: It is a choice that falls under the purview of the
finance manage r. Cash is needed for a variety of things, including paying
workers and salaries, utilities like electricity and water, paying creditors,
covering current obligations, keeping enough stock on hand, buying raw
materials, etc.
7. Financial controls: The fina nce manager must conduct financial
control in addition to planning, obtaining, and using the funds. Numerous
methods, including ratio analysis, financial forecasting, cost and profit
control, etc., can be used to achieve this.
1.4 INTRODUCTION TO THE INDIA N FINANCIAL
SYSTEM
1.4.1 Meaning and features of our financial system
The financial system is made up of the services that are offered to a person
by the various financial institutions, such as banks, insurance firms,
pension plans, funds, etc.
The charact eristics of the Indian financial system are listed below:
1. It promotes both saving and investment, it is essential to the nation's
economic growth.
2. It facilitates mobilising and distributing of savings.
3. It makes it easier for financial institutions and mark etplaces to grow.
4. It has a crucial impact on capital formation.
5. It facilitates communication between the saver and the investor.


munotes.in

Page 6


Financial Management
6 1.4.2 Components of a financial system – Institutions, Markets and
Assets
The Indian Financial System is made up of four bas ic parts. This
comprises:
1. Financial Institutions
2. Financial Assets
3. Financial Services
4. Financial Markets
1.4.3 Financial Institutions
Financial institutions typically act as an intermediary between the
borrower and the investor in the market. Savings from th e investor are
either directly or indirectly accessed through the financial markets.
The following are the primary responsibilities of the financial institutions:
1. It is possible to turn a short -term obligation into a long -term investment.
2. It facilita tes the transformation of a risky investment into a risk -free
investment.
3. It also serves as a convenient medium of exchange, matching small
deposits with big loans and large deposits with smaller loans.
A bank is the ideal illustration of a financial in stitution. People who have
extra cash save it in their accounts, while others who are in a financial
emergency take out loans. The bank serves as a go -between for the two.
These institutions can be further classified into two categories:
1. Banking Institu tions or Depository Institutions - These are
institutions like banks and other credit unions that collect funds from the
public in exchange for interest on deposits made and then lend those funds
to those in need.
2. Non-Banking Institutions or Non -Deposit ory Institutions - This
group includes brokerage firms, mutual funds, and insurance companies.
They can only sell customers financial items; they are not permitted to
request cash deposits.
Additionally, financial institutions can be divided into one of th ree groups:
Regulatory - Organizations like the RBI, IRDA, SEBI, etc. that oversee
the financial markets.
Intermediates - Commercial banks like SBI, BOB, and PNB that offer
loans and other forms of financial support.
Non-Intermediaries - Institutions that o ffer financial assistance to
corporate clients are known as non -intermediaries. It consists of SIBDI,
NABARD, etc. munotes.in

Page 7


Intro duction to Financial
Management
7 1.4.4 Financial Assets
Financial Assets are those assets and instruments that are exchanged on
financial markets. The securities on the marke t vary from one another
based on the various demands and needs of the loan applicant.
The following is a brief discussion of some significant financial assets:
Call money market - It is the term for a loan that is given for one day and
paid back the follow ing day. There is no need for collateral security in this
form of transaction.
Notice money market - Loans that are extended for more than a day but
less than 14 days are referred to as notice money. There is no need for
collateral security in this form of transaction.
Term money market - is used to describe deposits with maturities longer
than 14 days.
T-Bills - It also referred to as Treasury Bills, are government bonds or
debt securities that have a maturity of less than a year. A T -Bill represents
a financial loan to the government.
Certificate of deposits - A certificate of deposit is a dematerialized form
for money deposited in a bank for a set amount of time that is
electronically generated.
Commercial Papers - Commercial Paper is a form of short -term,
unsecured debt issued by businesses.
1.4.5 Financial Services
Services offered by companies that manage assets and liabilities are called
financial services./. They assist in obtaining the necessary cash and ensure
that they are invested effectively.
India's financial services include:
Banking Services - Any small or large service offered by banks, such as
opening an account, approving a loan, depositing money, issuing debit or
credit cards, etc.
Insurance services - Services related to insurance include issuing
insurance, marketing policies, undertaking insurance, and brokerages,
among other things.
Investment services - Asset management is primarily included in
investment services.
FOREX services - Currency exchange, foreign exchange, and other
related services are included in the category of foreign exchange services. munotes.in

Page 8


Financial Management
8 The primary goal of financial services is to let a person buy, sell, or
borrow securities, as well as facilitate lending, investing, and facilitating
payments and settlements.
1.4.6 Financ ial Markets
A financial market is a venue where buyers and sellers can interact and
engage in the trading of money, bonds, shares, and other assets.
Four categories further categorize the financial market as follows:
1. Capital Market: The Capital Market deal s with transactions that
have been going on for more than a year and is intended to finance long -
term investments. Three categories further split the capital market:
a. Corporate securities market
b. Government securities market
c. Long term loan market

2. Money Mark et: The money market The sort of market, which is
mostly dominated by the government, banks, and other large institutions,
is only authorised for short -term investments. It is a wholesale debt market
that uses very liquid, low -risk instruments. You can fur ther categorise the
money market into two categories:
a. Organizational Money Market
b. Unstructured Money Market

3. FOREX markets: The Foreign Exchange Market, one of the most
sophisticated marketplaces in the world, deals with the demands of multi -
currency. On the basis of the exchange rate, money is transferred in this
market.

4. Credit Market: A market where numerous banks and financial and
non-financial institutions provide short -term and long -term loans to people
or organisations is known as the Credit Market .
1.5 REGULATORY ASPECTS OF INDIAN FINANCIAL
SYSTEM
1.5.1 Introduction to regulations
Regulations are an important part of and an integral aspect of any financial
system. Regulations are required for the orderly growth of the financial
sector and the entir e economy. Regulations are required to maintain the
integrity of the financial system and its components. Failure of banks and
other financial services create larger systematic problems for the
economy. It is the job of different regulators to oversee reg ulations of
their respective domains. Sound regulations aim to enforce existing laws,
prevent scams and financial frauds, prosecute current cases of financial
misconduct, investigate complaints and protect the interests of investors
and clients. In simple terms, regulations ensure that all stakeholders have
confidence in the financial system. munotes.in

Page 9


Intro duction to Financial
Management
9 There are two aspects of regulations;
1. Prudential Regulations: These norms are in place to ensure that
firms have the requisite amount of money to remain solvent. These
regulations are in place to make sure that the institutions and
intermediaries are governed properly and there are appropriate risk
controls measures in place. These rules prevent a systemic failure of the
financial services ecosystem.
2. Investor Protection : A vital aspect of regulations is protecting the
rights of investors. The Indian financial system is characterized by a large
number of small investors who bring in small savings and deposits into the
system. It is absolutely essential that their interest s are protected.
Investors' rights need to be managed properly and their complaints need to
be addressed at the earliest.
1.5.2 Regulators in India
The regulatory system in India is such that the regulators are placed at the
top of the hierarchy and they a re responsible for managing all aspects of
that particular financial market / service. Following are some of the key
regulators in the Indian context.
1.5.3 Securities and Exchange Board of India (SEBI)
Securities and Exchange Board of India (SEBI) is the nodal regulator of
securities and commodities markets in India. It is directly placed under the
jurisdiction of the Ministry of Finance, Government of India. Although
SEBI was established in 1988, it gained statutory status and power only in
1992 under SEB I Act, 1992. Before SEBI was established, Controller of
Capital Issues was the regulator of Capital MArkets in India. It is
headquartered in Mumbai and has regional offices in New Delhi, Chennai,
Ahmedabad and Kolkata. The membership structure of SBI is su ch that it
has;
a. One chairman nominated by the Union Government of India.
b. Two members / officers nominated by the UNion Finance Ministry
c. One member nominated by the reserve Bank of India
d. The remaining five members are appointed by the Union Government
of In dia out of whom three shall be full time members.
The scope of SEBI covers the three broad groups of market participants;
a. Market Intermediaries
b. Issuers of securities
c. Investors
To ensure that SEBI does not falter in its functioning, it acts as a quasi
legislative, quasi judicial and quasi executive functions. These three munotes.in

Page 10


Financial Management
10 makes SEBI a very potent market regulator. Over the years, SEBI has
brought in plenty of market reforms which has made Indian markets at par
with the best in the world.
The functions of SE BI can be categorized into two areas a. Regulatory
Functions and b. Developmental Functions.
Regulatory Functions of SEBI
a. Regulation of Stock Exchanges and other related self regulated
organizations.
b. Regulation of Brokers, Sub brokers, Registrars to the is sue,
Underwriters etc.
c. Regulation of Portfolio Managers and collective investment schemes
like Mutual Funds.
d. Prevention of unfair market practices like insider trading and other
fraudulent practices.
e. Regulating any other areas concerning capital markets.
Developmental Functions of SEBI
a. Promoting Investor education and creating awareness about stock
markets.
b. Conducting market related research and publishing information
useful to all market participants.
c. Promotion of self regulatory organizations and drafting guidelines
for compliance and disclosure.
d. Undertaking training and development of different intermediaries.
1.5.4 Reserve Bank of India (RBI)
The Reserve Bank of India is the central bank of the country. It is the apex
bank and the principal regulator of the Banking sector in the country. The
main objectives of RBI is to issue currency notes and to create a stable
monetary policy. In other words, the actions of the RBI have a direct
bearing on the economic growth and stability of the country.
The Reserve Bank of India is driven by a 21 member central board of
directors. It has 1 Governor, 4 Deputy Governors, 10 Government
Nominated Directors, 2 Finance Ministry representatives (who usually are
Economic Affairs Secretary and FInance Affairs Secretary) and 4
Directors who represent the 4 local boards at New Delhi, Mumbai,
Kolkata and Chennai. Each of these Boards consist of 5 members who
adequately represent the regional banking landscape.

munotes.in

Page 11


Intro duction to Financial
Management
11 Following are some of the key functions of the Reserve Bank of Ind ia
1. Issuing Bank Notes - One of the most important functions of the RBI
is issuing currency notes and coins of all denominations except Re1
which is issued by the Ministry of Finance. Along with issuing notes
and coins, it is also responsible for the effect ive distribution of these
notes and coins across the length and breadth of the country.
2. Custodian of reserves of commercial banks - The RBI acts as the
custodian of cash reserves collected from other commercial banks.
Commercial Banks are required to maint ain CRR (Cash reserve ratio)
at a rate decided in the monetary policy.
3. Banker to the Government - A major function of the central bank is to
act as the banker to the government. The RBI is responsible for
maintaining and operating all the deposit accounts of the central
government. In international financial institutions like IMF and World
Bank, RBI represents the government of India.
4. Custodian of foreign exchange reserves - RBI is responsible for
maintaining the nation’s FOREX reserve. It is absolutely es sential
especially for dealing with any Balance of Payment crisis. FOREX
imbalance may cause an economic crisis of huge proportions.
5. Lender of Last Resort - The central bank acts as the banker to all
banks. In fact it is the responsibility of the RBI to m onitor the financial
health of all banks in the country. RBI provides commercial banks
with loans in case the commercial bank is facing any financial crisis.
6. Controller of credit in the economy - To achieve the monetary policy
goals, the central bank is re sponsible for controlling the credit creation
function of commercial banks. RBI uses qualitative and quantitative
methods to regulate and control the flow of money in the economy.
Interest rates and money supply regulations are required to achieve
objectiv es related to inflation, consumption and liquidity.
1.5.5 Insurance Regulatory and Development Authority of India
(IRDAI)
Insurance Regulatory and Development Authority (IRDA) is the regulator
of the overall insurance business in India. It works under the directives of
the Ministry of Finance and is tasked with the important task of regulating
and licencing insurance and reinsurance business in the country. It was set
up under the IRDA Act, 1999 and is headquartered in Hyderabad,
Telangana.
Section 4 of t he IRDA Act, 1999 gives guidelines about the composition
of governing members. IRDA is governed by a 10 member body which
comprises 1 chairman, 5 fulltime members and 4 part time members. All
the members are appointed by the Government of India.
Insurance business is classified into life and general insurance. Life
Insurance as the name suggests covers life insurance policies. General munotes.in

Page 12


Financial Management
12 Insurance on the other hand includes all the non life insurance business
like health insurance, motor vehicle insurance, c ommercial insurance,
travel insurance to name a few. We can see that the scope of IRDA is very
vast.
Following are some of the key functions of IRDA
a. IRDA has the sole authority over issuing, modifying, renewing,
suspending or cancelling the licenses of in surance companies.
b. Specifying the code of conduct and monitoring the activities of
surveyors and assessors.
c. IRDA settles disputes between insurers and other insurance
intermediaries.
d. It is tasked with maintaining the overall financial soundness of
insuranc e companies.
e. A key primary function is to safeguard the rights and interest of the
policyholders.
f. IRDA is tasked with ensuring orderly growth of insurance business in
the country. This is to ensure that economic development through
insurance business is ma intained at an optimum level.
g. IRDA also acts as a quasi judicial agency and is responsible for
settling disputes and redressing grievances of policy holders.
h. Another key function is monitoring the premium rates charged and
regulating any other business wh ich benefits insurance business.
1.6 QUESTIONS FOR DISCUSSION
1. Explain the nature of business finance and its relative importance for a
business.
2. Explain the meaning and scope of Financial management
3. Discuss the relative importance of financial management i n managing
the monetary resources of a firm.
4. What are the different decisions that a finance manager is required to
make?
5. Discuss the importance of a robust financial system from an economic
perspective.
6. Discuss the different components of Indian financial system.
Activity
One of the largest purchases occurred in the Indian steel sector during the
years 2006 –2007. Tata Steel made an offer to purchase Corus, the largest
steel company in the UK. After multiple rounds of bidding, Tata Steel
finally paid $12.9 billion for Corus. Tata Steel is now the sixth -largest
producer of steel in the world thanks to the Corus acquisition. Tata Steel's
action was applauded by the Indian steel industry, but there was a munotes.in

Page 13


Intro duction to Financial
Management
13 nationwide discussion about whether the company paid too much for the
Corus acquisition. The acquisition's proponents noted that there were
obvious synergies between the two businesses. This acquisition appeared
to have long -term advantages for Tata Steel as well.
Relate the above caselet with your learnings and discuss the financial
management decision aspects. You may take the help of online resources
to understand the case better.




munotes.in

Page 14

14 2
FINANCIAL PERFORMANCE ANALYSIS
USING RATIOS ANALYSIS

Unit Structure
2.0 Objective
2.1 What does Ratio Analysis tell you?
2.2 Liquidity Ratios
2.3 Working Capital Ratios
2.4 Capital Structure Ratios
2.5 Overall Profitability Ratios
2.6 Solved Problems
2.7 Problems for Practice
2.0 OBJECTIVE
After going through this chapter, the learner will be able to;
1. Understand the role played by financial statement analysis in
decision making.
2. Understand the different Liquidity ratios
3. Understand the different Profitabili ty ratios
4. Understand the different Debt ratios.
5. Understand other key financial ratios
Financial statements provide a thorough and an in -depth understanding of
the different monetary aspects of any company. Different techniques are
used in financial stateme nt analysis to compare financial data and assess a
company's situation.

These methods include of ratio analysis, common -size analysis, company
comparisons, trend analysis, and year -over-year analysis. Ratio analysis is
a mathematical technique for analysi ng a company's financial documents,
such as the balance sheet and income statement, to gather knowledge
about its liquidity, operational effectiveness, and profitability. The fact
that ratios differ between industries must be mentioned. A company in the
furniture industry, for instance, should only compare to benchmarks
within its own sector. The topics are illustrated in this self learning
material using hypothetical financial information from ABC Industries
(assumed to be a publicly traded company).



munotes.in

Page 15


Financial Performance Analysis Using Ratios Analysis
15 2.1 WHAT DOES RATIO ANALYSIS TELL YOU?

By carefully examining both historical and current financial statements,
investors and analysts use ratio analysis to assess a company's financial
health. Comparative data may show how a business is doing through time
and be used to predict how it will likely do in the future. This information
can be used to assess how a company compares to others in its industry
and to benchmark its financial performance against industry averages.

The financial statements of a compan y contain all the information needed
to calculate the ratios, making it simple for investors to apply this method.
Ratios serve as benchmarks for businesses. They assess the stocks of a
particular sector. They also compare a company's present performance t o
its past results. Understanding the factors that affect ratios is typically
significant as management has the freedom to occasionally change its
approach to improve the stock and company ratios.

In most cases, ratios are employed in conjunction with ot her ratios rather
than alone. You'll get a thorough understanding of the organisation from
several perspectives and be better able to identify potential red flags if you
have a solid understanding of the ratios in each of the four previously
mentioned cate gories.

2.2 LIQUIDITY RATIOS

A crucial group of financial indicators known as liquidity ratios is used to
assess a debtor's capacity to settle current debt commitments without the
need for outside funding. The measurement of indicators such as the
curren t ratio, quick ratio, and operating cash flow ratio allows us to
calculate liquidity ratios, which assess a company's capacity to satisfy debt
obligations as well as its margin of safety.

S. No. RATIOS FORMULAS
1 Current Ratio Current Assets/Current Liab ilities
2 Quick Ratio Liquid Assets/Current Liabilities
3 Absolute Liquid
Ratio Absolute Liquid Assets/Current
Liabilities

Profitability Ratios
These ratios examine a company's utilisation of its assets and the
efficiency with which it makes money off of its equity and assets, which
is another important component. Additionally, this provides the analyst
with data on how well the company's operations are used.

munotes.in

Page 16


Financial Management
16 S. No. RATIOS FORMULAS
1 Gross Profit Ratio Gross Profit/Net Sales X 100
2 Operating Cost Ratio Operating Cost/Net Sales X 100
3 Operating Profit
Ratio Operating Profit/Net Sales X 100
4 Net Profit Ratio Net Prof it/Net Sales X 100
5 Return on Investment
Ratio Net Profit After Interest And
Taxes/ Shareholders Funds or
Investments X 100
6 Return on Capital
Employed Ratio Net Profit after Taxes/ Gross
Capital Employed X 100
7 Earnings Per Share
Ratio Net Profit After Tax & Preference
Dividend /No of Equity Shares
8 Dividend Pay Out
Ratio Dividend Per Equity Share/Earning
Per Equity Share X 100
9 Earning Per Equity
Share Net Profit after Tax & Preference
Dividend / No. of Equity Share
10 Dividend Yield Ratio Dividend Per Share/ Market Value
Per Share X 100
11 Price Earnings Ratio Market Price Per Share Equity
Share/ Earning Per Share X 100
12 Net Profit to Net
Worth Ratio Net Profit after Taxes /
Shareholders Net Worth X 100


2.3 WORKING CAPITAL RATIOS

Simi lar to the liquidity ratios, it examines whether the corporation can
settle its current liabilities or debts with its current assets. This ratio is
essential for creditors to determine a company's liquidity and how quickly
it can turn assets into cash to p ay off debts.




munotes.in

Page 17


Financial Performance Analysis Using Ratios Analysis
17 S.
No. RATIOS FORMULAS
1 Inventory Ratio Net Sales / Inventory
2 Debtors Turnover
Ratio Total Sales / Account Receivables
3 Debt Collection
Ratio Receivables x Months or days in a
year / Net Credit Sales for the year
4 Creditors
Turnover Ratio Net Credit Purchases / Average
Accounts Payable
5 Average Payment
Period Average Trade Creditors / Net
Credit Purchases X 100
6 Working Capital
Turnover Ratio Net Sales / Working Capital
7 Fixed Assets
Turnover Ratio Cost of goods Sold / To tal Fixed
Assets
8 Capital Turnover
Ratio Cost of Sales / Capital Employed

2.4 CAPITAL STRUCTURE RATIOS

Each business has the capital or resources necessary to finance its
operations. These ratios, known as the capital structure ratios, examine
how a company uses money or funds structurally.

S. No. RATIOS FORMULAS
1 Debt Equity Ratio Total Long Term Debts / Shareholders
Fund
2 Proprietary Ratio Shareholders Fund/ Total Assets
3 Capital Gearing ratio Equity Share Capital / Fixed Interest
Bearing Fun ds
4 Debt Service Ratio Net profit Before Interest & Taxes /
Fixed Interest Charges


munotes.in

Page 18


Financial Management
18 2.5 OVERALL PROFITABILITY RATIOS
As implied by the name, these ratios assess a company's ability to generate
profits from its assets and capital for use in the future.
S. No. RATIOS FORMULAS
1 Overall Profitability Ratio Net Profit / Total Assets

There are several more ratios. We shall look at them along with their
formulas in the following solved examples.

2.6 SOLVED PROBLEMS

Problem -1

Thefollowing TradingandProfitandLossAccountofFantasyLtd.fortheyear31
‐3‐2000 is given below:
Particular Rs. Particular Rs.
To Opening Stock
“Purchases
“Carriage and Freight
“Wages
“Gross Profit b/d


To Administration
expenses “Selling and Dist.
Expenses “Non ‐operating
expenses “Financial
Expenses
Net Profit c/d 76,250
3,15,250
2,000
5,000
2,00,000
5,98,500
1,01,000
12,000
2,000
7,000
84,000
2,06,000 By Sales
“Closing stock





By Gross Profit b/d
“Non‐operating
incomes: “Interest
on Securities
“Dividend on shares
“Profit on sale of
shares 5,00,000
98,500



5,98,500

2,00,000

1,500
3,750
750
2,06,000

Calculate:
1.Gross Profit Ratio 2. Expenses Ratio 3.Operating Ratio 4. Net Profit Ratio 5. Operating (Net) Profit Ratio 6.Stock Turnover Ratio.
Solution –1(Problem related to Revenue Ratio)
1. Gross Profit Margin= Gross profit
Sales X100
2,00,000
5,00,000
X100

=40% munotes.in

Page 19


Financial Performance Analysis Using Ratios Analysis
19 2. Expenses Ratio= Op. Expenses NetSales X100
1,13,000
5,00,000
X100
=22.60%

3. Operating Ratio= Cost of goods sold+ Op.Expens es Net Sales X100
3,00,000+1,13,000
5,00,000
X100
=82.60%

Cost of Goods sold = Op. stock+ purchases+ carriage and Freight
+wages –Closing Stock

=76250 +315250+2000+5000 ‐ 98500

=Rs.3,00,000
4. Net Profit Ratio= Net Profit Net Sales X100
84,000
5,00,000
X100
=16.8%

5. Operating Profit Ratio= Op. Profit Net Sales X100
Operating Profit=Sales –(Op. Exp.+ Admin Exp.)
87,000
5,00,000
X100
=17.40%

6. Stock Turnover Ratio= Cost of goods sold Avg. Stock
3,00,000
87,375
=3.43times
munotes.in

Page 20


Financial Management
20 Probl em- 2

The Balance Sheet of Punjab Auto Limitedason31 ‐12‐2002 was as
follows:

Particular Rs. Particular Rs.
Equity Share Capital
Reserve
8% Loan on
Mortgage Creditors
Bank over draft
Taxation:
Current Future
Profit and Loss A/c 40,000
8,000
32,000
16,000
4,000

4,000
4,000
12,000 1,20,000 Plant and Machinery
Land and Buildings
Furniture & Fixtures
Stock
Debtors
Investments
(Short ‐term) Cash in
hand 24,000
40,000
16,000
12,000
12,000
4,000
12,000


1,20,000

From the above, compute (a) the Current Ratio, (b) Quick Ratio, (c )
Debt ‐Equity Ratio, and (d) Proprietary Ratio.


Solution – 2 (Problem related to Balance Sheet Ratio)


1.CurrentRatio = Current Assets Current liabilities Current Assets = Stock + debtors +
Investments (short term) +Cash In hand Current Liabilitie s = Creditors + bank
overdraft + Provision for Taxation(current&
Future) CA=12000+ 12000+4000+12000 =40,000 CL= 16000+4000+4000+4000 =28,000 =40,000
28,000 =1.43: 1



munotes.in

Page 21


Financial Performance Analysis Using Ratios Analysis
21
2. Quick Ratio = Quick Assets Quick Liabilities
Quick Assets =Curren t Assets ‐Stock
Quick Liabilities=Current Liabilities –(BOD+PFT future)
QA=40,000 –12,000 =28,000
QL=28,000 – (4,000+4,000) =20,000
=28,000
20,000
=1.40: 1


3.Debt –Equity
Ratio= Long Term Debt (Liabilities) Share holders Fund
LTL=Debentures + long term loans
SHF=Eq.Sh. Cap. + Reserves & Surplus +
Preference Sh.
Cap.–Fictitious Assets
LTL=32,000
SHF=40,000+8,000+12,000

=60,000
=32,000
60,000 =0.53: 1










munotes.in

Page 22


Financial Management
22
4. Proprietary Ratio = Shareholders’ Funds Total Assets SHF=Eq.Sh . Cap. + Reserves & Surplus+
Preference Sh.
Cap.–Fictitious Assets Total Assets =Total Assets –Fictitious Assets SHF=40,000+8,000+12,000 =60,000 TA=1,20,000 = 60,000
1,20,000 =0.5 :1
Problem =3
From the following particulars extracted from the books of Ashok & Co.
Ltd., compute the following ratios and comment:
(a) Current ratio, (b) Acid Test Ratio,(c) Stock ‐Turnover Ratio, (d)
Debtors Turnover Ratio, (e) Creditors' Turnover Ratio, and Average Debt
Collection period.
1‐1‐2002 31‐12‐2002
Rs. Rs.
Bills Receivable 30,000 60,000
Bills Payable 60,000 30,000
Sundry Debtors 1,20,000 1,50,000
Sundry Creditors 75,000 1,05,000
Stock ‐in‐trade 96,000 1,44,000

Additional in formation:
(a) On31 ‐12‐2002,there were assets: Building Rs.2,00,000, Cash
Rs.1,20,000 and Cash at Bank Rs.96,000.
(b) Cash purchases Rs.1,38,000 and Purchases Returns were Rs.18,000.
(c) Cash sales Rs.1,50,000 and Sale sreturns were Rs.6,000.
Rate of gross profit 25% on sales and actual gross profit was
Rs.1,50,000.






munotes.in

Page 23


Financial Performance Analysis Using Ratios Analysis
23
Solution – 3 (Problem related to find out missing item)

Notes : In this problem available information is not enoug h to solve
ratios asked so that need to prepare Trading Account to identify
values which are not given in the qu estion.
Trading Account

Particular Amou
nt
Rs. Particular Amount
Rs.
To Opening Stock 96,000 By Sales:
Cash: 1,50,000
To
Purchase:
Cash: 1,38,000 Credit : 4,56,000
Credit: 3,78,000 6,06,000
5,16,000 Less:S/R 6,000 6,00,000
Less:P/R 4,98,00
0 By Closing Stock 1,44,000
To Gross Profit 1,50,00
0
7,44,00
0 7,44,000



1. Gross Profit Margin= Gross profit Sales
X100
25%= 1,50,000
Sales X100

Sales= 1,50,000
25
X100
Sales=6,00,000








munotes.in

Page 24


Financial Management
24

2.CurrentRatio = Current Assets Current liabilities
Current Assets = Stock + debtors + Bills receivable + Cash +
Bank Balance
Current Liabilities = Creditors + Bills payable
CA=1,44,000+1,50,000+60,000+1,20,000+96,000
=5,70,000
CL= 1,05,000+30,000
=1,35,000
=5,70,000
1,35,000
=4.22 :1



3.AcidTest Ratio = Cash & Cash Equivalent Assets Liquid Liabilities Cash & Cash equivalent Assets = Cash + Bank + Short term Investments
(Liquid) Quick Liabilities = Current Liabilities –BOD
=1,20,000+ 96,000
=2,16,000
QL=1,05,000+30 ,000
=1,35,000
=2,16,000
1,35,000
=1.6 :1








munotes.in

Page 25


Financial Performance Analysis Using Ratios Analysis
25 4. Stock Turnover Ratio= Cost of goods sold
Avg. Stock
Avg.stock= Opening Stock + Closing
Stock
2
COGS=Sales –GP
96,000+1,44,000
2
AS=1,20,000
COGS=6,00,000 –1,50,000
4,50,000
=4,50,000
1,20,000
=3.75times


5.DebtorsRatio =

(Avg. debt collection period)
Debtors + Bills receivable
Credit sales
X365/360days
=1,50,000+ 60,000
4,56,000 X365days
=0.461 X365days
=168days


6.CreditorsRatio = Creditors + Bills payable Credit Purchase X365/360days
=1,05,000+ 30,000
3,78,000 X365days
=0.357 X365days
=130days







munotes.in

Page 26


Financial Management
26 Problem - 4

Following is thesummarisedBalanceSheetofMonaLtd.ason31 ‐3‐04.

Particular Rs. Particular Rs.
Equity Shares of Rs.
10 each 10%Pref. Sh.
of Rs.100 each
Reserves and Surplus
15% Debentures
Sundry Creditors Bank
Overdraft 10,00,000
4,00,000
7,00,000
5,00, 000
2,40,000
1,60,000
30,00,000 Fixed Assets
Investments Closing
Stock Sundry Debtors
Bills Receivable Cashat
Bank
Preliminary Expenses 20,00,000
2,00,000
2,00,000
4,60,000
60,000
60,000
20,000
30,00,000

Summarised Profit and Loss Account is as under forth year ending on
31‐3‐'04:
Rs.
Sales(25%Cashsales) 80,00,000
Less: Cost of goods sold 56,00,000 Gross Profit 24,00,000 Net profit (Before interest andtax50%) 9,00,000

Calculate the following ratios :
(1) Rate on Return on Capital Employed (2) Proprietary Ratio
(3) Debt ‐Equi ty (4) Capital gearing Ratio (5) Debtors Ratio (365 days of
the year.) (6) Rate of Return on Share holders 'Funds (7) Rate of Return on
Equity share holders fund

Solution -4 Statement of Profitability

Sales 80,00,000
Less: Cost of goods sold 56,00,000
Gross profit 24,00,000
Less: Operating expenses (including Depreciation) 15,00,000
Earnings before Interest & Tax (EBIT) 9,00,000
Less: Interest Cost 75,000
Earnings before Tax (EBT) 8,25,000
Less: Tax liability(50%) 4,12,500
Earnings after Tax(EAT/PAT) 4,12,500
Less: Preferences are dividend 40,000
Distributional Profit 3,72,500


munotes.in

Page 27


Financial Performance Analysis Using Ratios Analysis
27

1. 6. 7.
Rate of Return
Employed On
Capital Rate of Return holders Fund on
Sha
re Ratio fretur n on Equity Shareholders Fund
= EBIT
Capital employed X100 = PAT
SHF X100 =PAT –Pref .Div.
ESHF X100
CE = Eq Sh. Cap. + Pref. Sh. Cap. + Reserves & Surplus +Debenture + Long Term Loan –Fictitious Assets SHF = Eq. Sh. Cap. + Pref. Sh.Cap.+ Reserves & Surplus–Fictitious Assets ESHF= Eq.Sh.Cap.+
Reserves & Surplus –Fictitious Assets CE= 10,00,000+4,00,000
7,00, 000+5,00,000 –20,000
= 25,80,000 SHF= 10,00,000
7,00,000 ‐20,000
=20,80,000 + 4,00,000 +ESHF=10,00,000+7,00,000 –20,000
=16,80,000
= 9,00,000
25,80,000 X100 = 4,12,500
20,80,000 X100 = 3,72,500
16,80,000 X100
=34.88% =19.83% = 22.17%

2. Proprietary Ratio Shareholders’ Funds Total Assets
SHF=Eq. Sh. Cap. + Reserves & Surplus + Preference Sh.
Cap.–Fictitious Assets
Total Assets =Total Assets –Fictitious Assets
SHF=10,00,000+7,00,000+4,00,000 ‐20,000
=20,80,000
TA=30,00,000 –20,000
=29,80,000
= 20,80,000
29,80,000
=0.70: 1










munotes.in

Page 28


Financial Management
28

3. Debt –Equity Ratio= Long Term Debt (Liabilities) Shareholders Fund
LTL=Debentures + long term loans
SHF=Eq.Sh. Cap. + Reserves & Surplus + Preference
Sh.
Cap.–Fictitious Assets
LTL=5,00,000
SHF=10,00,000+7,00,000+4,00,000 ‐20,000
=20,80,000
=5,00,000
20,80, 000
=0.24: 1










4.Capital Gearing Ratio = Fixed Interest or Dividend Securities Equity Share holders Fund
FIS=Debentures +Preference share capital
ESHF = Eq. Sh. Cap. + Reserves & Surplus – Fictitious Assets
LTL=9,00,000
ESHF=10,00,000+7,00,000 ‐20,000
=16,80,000
=9,00,000
16,80,000
=0.54: 1 munotes.in

Page 29


Financial Performance Analysis Using Ratios Analysis
29


5.Debtors Ratio

(Avg. debt collection
period) =
Debtors + Bills receivable
Credit sales
X365/360days
=4,60,000+ 60,000 X365days 60,00,000
=0.461 X365days
=31.63days
=32days(Aprox.)

Problem - 1

Two years 'Balance sheets of Jamuna Company Ltd. Areas
follows: [S.U.T.Y. -April,1999]
Liabilities 31‐3‐03 31‐3‐04 Assets 31‐3‐03 31‐3‐04 Equity share capital10%Pref. Sh. Capital General Reserve
Profi t & Loss A/c 12%Debentures Creditors
Bills payable
Bank Overdraft O/s. Expenses 1,00,000
50,000
30,000
20,000
1,00,000
30,000
10,000
10,000
5,000 1,50,000
50,000
30,000
‐‐‐‐‐
50,000
35,000
25,000
20,000
10,000 Land and Buildings
Machinery
Debtors
Bills
Receivable
Stock
Bank
Balance
Cash
Balance
Profit & Loss A/c 1,00,000
90,000
53,000
20,000
75,000
15,000
2,000
‐‐‐‐ 90,000
90,000
30,000
12,000
90,000
35,000
13,000
10,000
3,55,000 3,70,000 3,55,000 3,70,000
Additional Information:
2002 ‐'03 2003 ‐04
Rs. Rs.
(1)Sales 3,65,000 2,19,000
(2)Cost of Good s sold 2,19,000 1,46,000
(3)Net profit(Before Pref. Dividend) 35,000 47,500
(4)Stockon1‐4‐'02 71,000 ‐‐‐

Calculate following ratios and give your opinion about company position
in 2003 ‐'04 in comparison with 2002 ‐'03. Whether it is positive or
negative?
(1)Current ratio (2) Liquid ratio (3) Debtors ratio (Take 365 days for
calculations) (4) Gross profit ratio (5) Stock Turnover ratio(6) Rate of
return on equity share ‐ holders' funds.

munotes.in

Page 30


Financial Management
30
Problem - 2
The Balance Sheet as on 2002 and 2003 are as under :
Liabilities 2002 2003 Assets 2002 2003
Equity share c apital
General Reserve Profit
& Loss A/c Creditors
Bills payable O/s. Expenses Provident Fund 1,00,000
12,500
10,000
5,000
3,750
1,250
7,500 1,25,000
15,000
7,500
6,250
7,500
3,750
5,000 Land and Buildings
Plant Machinery Stock
Debtors Cash & Bank
Bills Receivable
Prelimina ry Exp. 50,000
57,500
10,000
7,500
5,000
2,500
7,500 75,000
55,000
12,500
10,000
7,500
5,000
5,000
1,40,000 1,70,000 1,40,000 1,70,000
Profit & Loss A/c.
Particulars 2002 2003 Particulars 2002 2003
To Op. Stock To Purchase To Office Exp.
To Selling exp. To Fin.
Exp.
To Net Profit 5,000
37,500
7,500
5,000
2,500
17,500 10,000
47,500
10,000
12,500
15,000
30,000 By Sales
By Closing Stock By
Profit on Sale of
Furniture 62,500
10,000

2,500 1,12,500
12,500

‐‐‐‐
75,000 1,25,000 75,000 1,25,000
Find out (1) Current Ratio (2) Stock Turnover Ratio (3) Gross Profit Ratio
(4) Liquid Ratio (5) Debtor Ratio (workingdays300) (6) Return on Equity
Capital employed (7) Ownership Ratio.

Problem - 3

Following are incomplete Trading & Profit and Loss A/c. and Balance
Sheet.
Trading A/c. Particular Rs. Particular Rs. To Op. stock To Purchase
To Purchase Return To Gross
Profit 3,50,000(?)87,000
7,18,421By Sales By Closing Stock (?) (?)

14,96,710 14,96,710
Profit & Loss A/c. Particular Rs. Particular Rs. To Office Exp. To Int.on Deb.
To Tax. Provision To Net Profit 3,70,00030,000
18,421
3,50,000By Gross Profit ByCommission 7,18,421(?)(?)(?)

munotes.in

Page 31


Financial Performance Analysis Using Ratios Analysis
31
Balance Sheet Particular Rs. Particular Rs. Paid Up Capital General Re serve
P&L a/c.
10% Debenture Current
Liabilities 5,00,000(?)
(?)
(?)6,00,000 (?)Plant & machinery Stock Debtors Bank
Other Fixed Assets 7,00,000(?)
(?)62,500
(?)(?)
Find out missing items with the help of other details are as under:
1. Current Ratio was 2:1.
2. Closing Stock is 25% of Sales.
3. Proposed Dividend was 40% of paid up capital.
4. Gross profit Ratio was 60%.
5. Amount transfer to General Reserve is same as proposed Dividend.
6. Balance of P & L Account is calculated 10% of proposed dividend.
7. Commission income is 1/7 of Net profit.
8. Balance of General reserve is twice the current year transfer amount.




munotes.in

Page 32

32 3
WORKING CAPITAL MANAGEMENT
Unit Structure
3.0 Objective
3.1 What is working capital management?
3.2 Objectives of Working Capital Management
3.3 Understanding working capital management
3.4 Classification of Working Capital
3.5 Why manage working capital management?
3.6Working capital management ratios
3.6.1 Current Ratio (Working Capital Ratio)
3.6.2 Collection Ratio (Days sales outstanding, DSO)
3.6.3 Inventory turnover ratio
3.7 Working capital investment policies / Approaches to current assets
financing
3.8 Operating Cycle / Working Capital cycle
3.9 Measurement of Operating Cycle:
3.10 Solved Problems
3.11 Problems for Practice
3.0 OBJECTIVE
After undergoing this chapter, the learner will be able to;
1. Understand the importance of working capital management.
2. Understand and evaluate the different working capital requirements
3. Understand cash cycle and operating cycle
4. Analyze working capital policies
3.1 WHAT IS WORKING CAPITAL MANAGEMENT?
A business technique called working capital management aims to make
sure a firm runs smoothly by keeping track of and making the most use of
its current assets and liabilities. Ratio analysis is a tool that can be used to
measure the effectiveness of working capital management. Working
capital management, which is calculated as current ass ets minus current
liabilities, is a method used in business to assist organisations in making
efficient use of current assets and preserving enough cash flow to achieve
short -term objectives and responsibilities. Businesses can release cash that
might othe rwise be stuck on their balance sheets by managing working
capital correctly. As a result, they could be able to cut down on the need munotes.in

Page 33


Working Capital Management

33 for outside borrowing, grow their companies, finance mergers and
acquisitions, or make R&D investments.
Every firm needs w orking capital to function, but managing it well
involves striking a balance. Companies must have enough cash on hand to
pay for both anticipated and unforeseen expenses while also making the
greatest use of the current finances. This is accomplished by ef ficient
management of cash, inventories, accounts payable, and accounts
receivable.
3.2 OBJECTIVES OF WORKING CAPITAL
MANAGEMENT
Working capital is a crucial measure for companies to monitor because it
shows how much money they have available to pay bills, cover unforeseen
expenses, and maintain corporate operations. However, working capital
management is not so straightforward, and a working capital management
programme may have a number of aims, such as:
1. Meeting business obligations - Working capital mana gement
should constantly make sure that the company has adequate cash on hand
to satisfy its immediate obligations, frequently by extending supplier
payment terms or collecting payments from clients earlier. Unexpected
expenses can also be categorised as r esponsibilities, thus they must be
taken into account when developing a working capital management
strategy.

2. Growth in business - Having said that, it's crucial to successfully
utilise your short -term resources, whether that entails funding international
expansion or contributing to R&D. Your firm may not be as successful as
it may be if its assets are locked up in inventory or accounts payable. In
other words, it is not ideal to handle working capital too cautiously.

3. Optimum utilization of capital - Optimizing the effectiveness of
capital usage, whether by lowering capital expenses or maximising capital
returns, is another goal of working capital management. For the latter, it is
necessary to make sure that the ROI of extra capital above the typical cost
of financing it. The former can be accomplished by recovering capital that
is now locked up to lessen the need for borrowing.
3.3 UNDERSTANDING WORKING CAPITAL
MANAGEMENT
Working capital management's major objective is to help the business
maintain enough cash flow to cover its short -term operational expenses
and short -term debt commitments. Working capital is the difference
between a company's current assets and current liabilities.
Anything that can be quickly converted into cash within a year is
consider ed a current asset. These are the very liquid assets of the business.
Cash, accounts receivable, inventories, and short -term investments are a munotes.in

Page 34


Financial Management
34 few examples of current assets. Any debts that are due within the next 12
months are considered current liabiliti es. These consist of the present
portion of long -term debt payments as well as accruals for operating
expenses.
Monitoring cash flow, current assets, and current liabilities through ratio
analysis of the important components of working capital, such as the
working capital, is a common aspect of working capital management.
3.4 CLASSIFICATION OF WORKING CAPITAL
Working capital can take the following forms:
(a) Gross Working Capital: Gross working capital is the sum of money
invested in different current asset components. It consists of unfinished
commodities, work in progress, debtors, raw resources, and more.
(a) Net Working Capital: Net working capital is the difference between
current assets and current liabilities. The main goal of this exercise is to
determine the kind and quantity of current assets needed to cover current
liabilities.
(c) Positive Working Capital: This is the positive difference between
current assets and liabilities.
(d) Negative Working Capital: The surplus of current liabilities over
current assets is referred to as negative working capital.
(e) Permanent Working Capital: This is the bare minimum of working
capital needed throughout the year, even at the slowest time of year.
(f) Temporary or Variable Working Capital: This category incl udes
any additional current assets needed at various points during the fiscal
year to cover things like extra inventory, extra cash, etc.
3.5 WHY MANAGE WORKING CAPITAL
MANAGEMENT?
The net operating cycle (NOC), often referred to as the cash conversion
cycle (CCC), is the lowest length of time necessary to convert net current
assets and liabilities into cash. Working capital management supports
maintaining the NOC's smooth operation.
Through the effective use of its resources, working capital management
can enhance a company's cash flow management and earnings quality.
Inventory management, accounts receivable and payable management, and
accounts payable management are all included in working capital
management.
The timing of accounts payable is another aspe ct of working capital
management (i.e., paying suppliers). A business can manage its working
capital by deciding to extend supplier payments and to make the most of
available credit, or it can spend money by making cash purchases. munotes.in

Page 35


Working Capital Management

35 3.6 WORKING CAPITAL MANAGEMEN T RATIOS
3.6.1 Current Ratio (Working Capital Ratio)
Current liabilities divided by Current Assets yields the working capital
ratio, often known as the current ratio. A company's capacity to satisfy its
immediate financial obligations is an important sign of its financial health.
A working capital ratio below 1.0, though figures differ by industry,
typically signifies that a business is having problems paying its short -term
obligations. In other words, the company's liquid assets would not be
sufficient to pay its bills due in the next year. In this situation, the business
may be forced to turn to asset sales, long -term debt borrowing, or other
forms of financing to pay off its short -term debt commitments. Working
capital ratios between 1.2 and 2.0 are prefe rred, although a ratio greater
than 2.0 suggests that the company is not using its assets efficiently. A
very high ratio is not desirable and indicates that the company is not
managing its working capital well.
Why is current ratio / working capital ratio important?
The current ratio, commonly referred to as the working capital ratio, is a
measure of liquidity that shows how well a company can satisfy its short -
term obligations. A company's finances may be in danger in the short term
if its current ratio is less than 1.00, which indicates that short -term debts
and liabilities outweigh current assets.

3.6.2 Collection Ratio (Days sales outstanding, DSO)
The efficiency with which a business manages its accounts receivable is
shown by the collectio n ratio, commonly referred to as days sales
outstanding (DSO). The average amount of outstanding accounts
receivable divided by the total amount of net credit sales throughout the
accounting period, multiplied by the number of days in an accounting
period, yields the collection ratio. The average number of days it takes for
a business to get paid following a sales transaction on credit is given by munotes.in

Page 36


Financial Management
36 the collection ratio calculation. The collection ratio will be lower if a
company's billing department is succe ssful in trying to collect debts and
clients pay their bills on time. A corporation converts receivables into cash
more quickly, the lower its collection ratio.
Why collection ratio is important?
The efficiency with which a business can collect on its acco unts receivable
is shown by the collection ratio, also known as days sales outstanding
(DSO). It may be a hint that there won't be enough money on hand to
cover immediate responsibilities if it takes a while to collect. The goal of
working capital manageme nt is to increase the speed of receivables
collection.

3.6.3 Inventory turnover ratio
Inventory management is a crucial component of working capital
management. A business must retain enough inventory on hand to meet
consumer demand while avoi ding excessive inventory that ties up working
capital in order to run as efficiently as possible and maintain a comfortable
high level of working capital. Companies often track the inventory
turnover ratio to determine how effectively that balance is kept. The cost
of goods sold divided by average balance sheet inventory yields the
inventory turnover ratio, which measures how quickly a company's
inventory is used up and replaced through sales. A significantly low ratio
when compared to other companies in th e same industry raises the
possibility that inventory levels are too high, while a comparatively high
ratio could mean that inventory levels are insufficient.
Why is inventory ratio important?
The inventory turnover ratio demonstrates how effectively a bus iness sells
its inventory. A significantly low ratio when compared to other companies
in the same industry raises the possibility that inventory levels are too
high, while a comparatively high ratio could mean that inventory levels
are insufficient.




munotes.in

Page 37


Working Capital Management

37 3.7 WORK ING CAPITAL INVESTMENT POLICIES/
APPROACHES TO CURRENT ASSETS FINANCING
The sources and quantity of working cash that a corporation should
maintain are essentially covered by working capital finance policy. The
proportions of shor t-term and long -term sources used to finance existing
assets are also important factors for a company to consider. After
considering the variability of its cash inflows and outflows as well as the
level of risk, a corporation may choose from a number of wo rking capital
investment programmes.
1. Conservative Policy: As its name implies, this strategy aims to reduce
the risk associated with financing current assets. In this case, a sizable
amount of long -term sources will be used to fund present assets. The f irm
not only matches the current assets with current liabilities but also keeps
some excess amount to meet any uncertainty.
Although this is the working capital policy with the lowest risk, it does not
guarantee the best possible use of assets. Consequentl y, it lowers the
expected returns for stockholders. Below is an illustration of this policy.
Fixed assets are shown on Line A, and permanent working capital, which
is financed by long -term sources, is shown on Line B. Line C, which is
dashed, indicates tha t a portion of the variable current assets are also
financed by long -term sources. In accordance with this policy, a portion of
volatile current assets is financed with short -term resources.

2. Aggressive policy: Aggressive working capital financing poli cy is a
risky policy that calls for the most investment possible in present assets.
This policy will use short -term debt to finance both fluctuating and long -
term current assets. According to this policy, payments to creditors are
made as late as feasible but debt is promptly recovered. The following is
an illustration of this policy. This strategy finances fixed assets,
represented by Line A, with long -term sources; nevertheless, a percentage
of permanent current assets, represented by dotted Line B, is al so financed munotes.in

Page 38


Financial Management
38 with long -term sources. Both the entire amount of fluctuating current
assets, illustrated by the curved Line D, and the remaining portion of
permanent current assets, represented by Line C, are financed by short -
term debt.

3. Extremely Aggre ssive Policy : This strategy for financing working
capital is extremely risky. This approach allows for the financing of even a
small portion of fixed assets with short -term funds. This strategy is
extremely dangerous since it relies too heavily on short -term funding. The
following is an illustration of this policy. As indicated by dotted Line A, a
significant share of fixed assets are financed.
4 Hedging Policies - The hedging strategy, also known as the
matching policy, is the method by which a company fin ances its working
capital requirements. This policy is effective in a scenario where the
business's existing assets are utilised exactly to offset its current liabilities.
According to this strategy, changing current assets are financed through
short -term sources, whereas fixed and permanent current assets are
financed through long -term sources. It's important to pay close attention to
this insurance because it carries a moderate level of risk. For instance,
even if it may or may not have enough cash right now, the company will
munotes.in

Page 39


Working Capital Management

39 make sure that there will be enough cash available to repay the loan when
it is due, say, six months from now.

3.8 OPERATING CYCLE / WORKING CAPITAL
CYCLE
Due to the slow conversion of cash into finished items, debtors, and back
into cash, any business organisation needs an adequate working capital. It
takes a while. For instance, cash is utilised in a manufacturing company to
buy raw materials. They are not eaten right away. In order to ensure
efficient production and safeguard the company against the possibility of
future raw material shortages, they are kept in storage for a period of time.
Following conversion, they are distributed from retailers to the production
facility. Additionally, this conversion typically takes some time. It
transforms into semi -finished items or work -in-progress and then finished
goods when specific costs like wages and overheads are incurred on it.
Before being sold, these final goods will need to be stored for a while. The
sale of finished items to cli ents follows, and payment may be made in cash
or in the form of receivables or debtors. Once realised, receivables and
debtors again take the form of cash, and the cycle repeats. The same is
explained in the following diagram.
munotes.in

Page 40


Financial Management
40 The working capital cycle, o ften known as the operational cycle, refers to
the continuous flow of cash from customers to suppliers, to inventory, to
accounts receivable, and back to customers. Or to put it another way, the
term "operational cycle" refers to the period of time that st arts with a
company's purchase of raw materials and concludes with the final
collection of cash from creditors.
The length of the working capital cycle affects the amount of working
capital. The demand for maintaining working capital increases as the
busin ess cycle lengthens. Consequently, the fund will be tied up in diverse
current assets for a longer length of time. The length of the operational
cycle varies from business to business and from industry to industry.
For example, if raw materials are stored for 30 days, conversion or
processing takes 45 days, completed items are stored for 30 days, and debt
collection takes 40 days. The sum of this time period (i.e., 30 + 45 + 30 +
40 or 145 days) is referred to as the gross operating cycle. Credit is given
to businesses when they buy raw materials from suppliers. The length of
working capital is shortened by this term of payment postponement. The
payment deferral time or length of credit granted by raw material suppliers
is subtracted from the gross operation cycle to determine the net -working
cycle period. If the supplier grants a 45 -day credit period, the net operating
cycle is 100 days (i.e., 145 days minus 45 days).
3.9 MEASUREMENT OF OPERATING CYCLE:
Strictly speaking, the volume of working capital depend s upon the length
of working capital cycle. So ,it is important to measure working capital
cycle for management of working capital. The financial statements i.e.,
Profit and Loss Account and Balance Sheet, can guide us to measure
working capital cycle.

The steps involved in the determination of the operating cycle
are shown be low :
Particulars Days
1.Rawmaterialsholdingperiod ***
2.Work -in-progressperiod ***
3.Finishedgoodsholdingperiod ***
4.Debtorscollectionperiod ***
Gross Operating Cycle ***
5.Less : Creditors payment period (***)
Net Operating Cycle ***
munotes.in

Page 41


Working Capital Management

41 The procedure can be summarized as below:
1. Raw Material Storage Period:
It represents the average period during which raw materials are kept
in stores.

It is calculated as :
Raw material stora ge period

AverageStock of RawMaterialAveragedailyConsumptionof RawMaterail       

Here, Average stock of raw materials
sin2Opening RawMaterialClo g RawMaterial  
sin2Opening RawMaterialClo g RawMaterial  
Average daily consumption of raw material = No.of
Average daily consumption of raw material

.(360 )No of Total MaterialConsumptionWorking daysina year days    
*Note : 365 days also can be used in place of 360 days to calculate the
average.
If consumption of raw material is not available, average daily purchase
can also be taken.

Processing Period:
Once materials are issued to production, it again involves time gap
between issue of materials and production of finished product. This time
gap is called processing period.
It is calculated as :
Processing Period

vr P r
cos PrA e ageStock of Workin ogressAveragedaily factory tof oduction     

Average stock of work -in-progress
Pr , sin2OpeningWorkin ogress Clo g workin progress    

Average daily factory cost of production

cos
360Total factory tof productiondays  
munotes.in

Page 42


Financial Management
42 *Note : 3 65 days also can be used inplace of 360 days to calculate the
average. Factory cost of production during the year = Raw materials
consumed + Direct wages + Other direct expenses + Manufactu ring over
head + Opening WIP – Closing WIP.

Finished Goods Storage Period :
Manufacturing enterprises produce the output in the expectation of future
demand. Till the demand for finished product materializes, the product
would remain in the store. This period is termed as finished goods storage
period.
This is calculated as : Finished goods storage Period
cosAveragestock of finished goodsAveragedaily tof goodssold     

Average stock of finished goods
sin2Opening stock of finished goodsClo g stock of finished goods      

Average daily cost of goods sold

cos
360Total tof goodssolddays  

2AverageDebtors Averagedailycredit salesCredit period toDebtors     

sin2Opening debtorsClo g debtorsAverageDebtors   

360360Totalcredit sales forthe year daysAveragedailycredit sales        


*Note : 365 days also can be used in place of 360 days to calculate the
average.
Cost of goods sold = Opening stock of finished goods + Factory cost of
production – Closing stock of finished goods.

Credit period Allowed to Debtors:
The business enterprises due to competitive and other reasons —extend
credit facilities to customers. The time gap between sale and realisation of
cash is known as credit or collective perio d from debtors.

It is computed as :
Credit fromSuppliers AverageCreditors Averagedailycredit purchase        sin 2AverageCreditors Opening creditorsclo g creditors    
360 Averagedailycredit purchase Totalcredit purchase forthe year days          munotes.in

Page 43


Working Capital Management

43
*Note : 365 days also can be used in place of 360 days to calculate the
average.

Illustration 1 :
The following information is availa ble for Swagat Ltd. :(`.‘000)

Averages to ckofraw materials and stores 200
Average WIP in ventory 300
Average finished goods in ventory 180
Average accounts receivable 300
Average accounts payable 180
Average raw materials and stores purchase on credit and consumed per day 10
Average WIP value of raw materials committed per day 12.5
Average cost of goods sold per day 18
Average sales per day 20

You are require to calculate :
Duration of raw material stage
Duration of WIP stage
Duration of Finishe d goods stage
Duration of accounts receivable stage
Duration of accounts payable stage, and
Duration of operating cycle. Solution:
Duration of Raw Material Stage

2002010Averagestock of Rawmaterialsand StoresdaysAveragerawmaterialsand stores purchased per day           

Duration of Work -in-progress Stage
= Average work in progress inventory = 300
Average work in progress value of raw materials committed per day
= 24 days

Duration of Finished Goods Stage 12.5

18010cos 18AverageFinished goodsinventorydaysAverage tof goodssold perday       

Duration of Accounts Receivable Stage

Re 3001520Average Accounts ceivabledaysAverageCredit Sales perday    

Duration of Accounts Payable Stage

1801810Average AccountsdaysPayble AverageCredit Purchase perday       munotes.in

Page 44


Financial Management
44
Duration of Operating Cycle
= (i) + (ii) + (iii) + (iv) – (v)
= 20 days + 24 days + 10 days + 15 days – 18 days = 51 days

Illustration 2 :
From the following information extracted from the books of a
manufacturing company, compute the operating cycle in days and the
amount of working capital required :

Period Covered 365days
Average period of credit allowed by suppliers 16days
Average Total of Debtors Outstanding 480
Raw Material Consumption 4,400
Total Production Cost 10,000
Total Cost of Sales 10,500
Sales for the year 16,000
Value of Average Stock maintained:
Raw Material
320
Work -in-progress 350
Finished Goods 260
Solution : Calculation of Operating Cycle
(i)Raw material held in stock :

Average stocks of raw materials held - 320 Average consumption
per day 4,400x365 275days
Less : Average credit period granted by Suppliers 16days
11days

Work -in-progress:

int 350 365 320 13AverageWIPma ained days      

Finished goods held in Stock:
int 260 260 365910,500 / 365 10,500AverageFinished GoodsMa aineddaysAverageCostof Good Sold Per Days        

Credit period allowed to Debtors :
tan 480 365 4801116,000 / 365 16,000AverageTotalof Outs ding debtorsdaysAverageCredit SalesPer Day        

∴Total operating cycle period : (i) + (ii) + (iii) + (iv) = 44days
∴Numbers of operating cycles in a year = 365/44=8.30times
∴Amount of working capital required

cos 10,500.1,2658.3Totaloperating tRsNumberof operating cyclesina year     
munotes.in

Page 45


Working Capital Management

45 Problem for Practice 1
Calculate the operating cycle from the following figures:
`(in Lakhs)
Annual sales 1,000
Manufacturing expenses 240
Distribution and other expenses 40
Purchase of materials 400

Opening stock:
Raw materials 80
Work -in-progress 20
Finished goods 60
Closing stock:
Raw materials 120
Work -in-progress 60
Finished goods 20
Opening balance of sundry debtors 40
Closing balance of sundry debtors 40

The company obtains accredit for 60 days from the suppliers. A ll goods
are sold for credit. Assume 360 days in the year.
(Ans : Operating Cycle=109 days)
Problem for Practice 2
From the following information, extracted from the books of a
manufacturing company, compute the operating cycle in days :
Period covered : 3 65 days Average period of credit allowed by suppliers,
16 days Other data areas follows :


(Rs.In‘000)Average debtors (outstanding) 480Raw material consumption 4,400Total production cost 10,000Total cost of sales 10,500Sales for the year 16,000Value of average stock maintained :
Raw material 320Work -in-process 350Finished goods 260
(Ans : 44 days)



munotes.in

Page 46

46 4
FINANCIAL PLANNING AND
FORECASTING
Units Structure:
4.0 Objective
4.1 Financial planning and forecasting, meaning and importance.
4.2 Approaches to financial planning
4.3 Financial statements
4.3.a Pro-forma of income statement
4.3.b Pro-forma of Balanc e Sheet
4.4 Computation of external financing requirements.
4.5 Summary
4.6 Questions
4.7 References
4.0 OBJECTIVES
After learning this topic you will be able to understand
 The concept of finance
 Importance of finance in terms of financial planning and forecasting
 Different approaches to financial planning
 Analysis of financial statement
4.1 INTRODUCTION
We have seen in the earlier topics the meaning and importance of financial
management and its importance. The business to run smoothly has to
manage its finance in proper way so that the firm is not over or under
capitalised. Financial planning is one of the important task to be
undertaken by the Chief Financial Officer (CFO) of the company. The task
of the CFO is that of a head in a family who manage s every financial
aspect for the smooth functioning of the firm. The financial requirements
of the business if to be planned to meet the requirement of the company
depending upon the stage of development, the scope of operations, the
future plans of the co mpany , etc. The financial requirement of the
business may be for the investment in fixed assets or current assets. The
finance raised may be from Owned fund or borrowed fund which together
comprises the capital structure of the business. Financial plann ing is the munotes.in

Page 47


Financial Planning And
Forecasting
47 intellectual activity which gives the clear picture of the financial status of
the business in near future. In this topic we will see all the aspects
regarding financial planning and forecasting and the different methods of
analysis of the finan cial statements.
4.1.1 FINANCIAL PLANNING
As said earlier financial planning is the process of deciding the need and
utilisation of the finance of the company in the upcoming future. Every
business needs funds to run the activities of the firm/ company in the
smooth manner. For example, a Company is in need of ten lakhs rupees
then the CFO needs to plan the fund raising strategies and accordingly the
options of investment to be made with the fund raised. If the said amount
of ten lakh rupees is raised in t ime, then the CFO can efficiently manage
the activities. On the other hand, if the said amount is not raised then the
operations of the business will be disturbed. Hence financial management
requires knowledge and it ensures required to raise finance in time when
required. Finance when raised more than requirements is also dangerous
for the business. Financial planning makes the blueprint of the business
finance for the future operations. Financial planning differs from financial
management. The latter is the wider concept while financial planning is
the narrow concept.
4.1.2 OBJECTIVES OF FINANCIAL PLANNING
1. Availability of funds: - Financial planning aims at making availability
of funds on time for the purpose of investment in fixed assets, current
assets, operations etc. Depending upon the urgency of finance, sources of
finance is also decided in financial planning.

2. Adequate finance: - Financial planning also aims at raising required
amount of fund only. As discussed earlier, excess or shortage of fund will
disturb the activities of the business. Hence, proper amount of fund is to
be raised as per the requirement of the nature of expenses.

3. Income generation: - Financial planning is to be made keeping in
mind that there should be income from the ra ised finance. For example,
borrowed funds for capital investments are not utilised fully at the initial
stages. In such cases, the excess of borrowed fund should be invested
wisely so that it generates income and the cost of borrowed capital reduces
for the firm.

4. Growth and development: - Keeping in mind the objectives and
targets of the board of directors of the company, the financial planner
should plan the finance. Financial planning should be in the direction of
growth and development of the business .



munotes.in

Page 48


Financial Management
48 4.1.3 IMPORTANCE OF FINANCIAL PLANNING
We have studied the meaning and objective of financial planning. Let us
understand the importance of financial planning in brief.
a. Facilitate rising of fund in optimum manner
b. Helps in proper utilisation of finance
c. To deve lop a best capital structure
d. Helps to absorbs shocks of the business
e. Helps in investing in appropriate project
f. Links between investment and financial decision
g. Helps in proper operation of business
h. Helps in proper financial control in future.
4.1.4 DIFFEREN CE BETWEEN FINANCIAL MANAGEMENT
AND FINANCIAL PLANNING.
Financial planning and financial management are not same as discussed
earlier. Let us understand the detail difference between the both for clear
understanding.
Sr.
No Financial Management Financial Planning
1 It refers to efficient acquisition
of fund for smooth operations
of the business. It refers to estimation of capital
requirement and deciding
sources of fund.
2 It has wider scope
3 Its objective is to manage all
the activities of the finan ce It objective is to raise funds on
time to the required level

4.1.4 FINANCIAL FORECASTING
Financial forecasting is the process of estimating future financial
requirements of a business. It is an act of deciding funds requirement and
time pattern of such req uirements. It is clear that financial forecasting
helps to know the requirements and applications of the funds raised.
Financial forecasting helps the business in different ways such as: -
Helps in planning
Dealing with uncertainties
Forecast of earning in the future course of business.
The characteristics financial forecasting are as follows
a. It is estimation of future events that would occur in the business
b. It is estimation future on the basis of past activities analysis.
c. It helps in financial planning as we get a estimate of requirement of
finance.
d. It involves various techniques for perfect estimation
e. Minimise the risk of uncertainties which will arise in future. munotes.in

Page 49


Financial Planning And
Forecasting
49 f. It predicts the possible changes in order to bring accuracy in
management decision.
Advantage s of financial forecasting
1. It helps to set the standard of performance of the business which can
be achieved in the future course of action. Such standards should be
set on the basis of the business resources so that achieving the set
standards become eas y and possible.

2. Helps in timely arrangement of fund for the business. As discussed
earlier, timely arrangement of the finance keeps the business in
operation without disturbances. Finance is also stated as blood of the
business for the same.

3. Optimum uti lisation of the available resources of the business. Proper
forecasting will help the business to operate at the maximum level and
to plan the future strategies of the business accordingly.

4. Control cash flow of the business. Financial forecasting can be used
as a parameter to control the cash flow of the business. Cash flow is
normally from the activities like capital raising, investment and
operating activities of the business.

4.2 APPROACHES TO FINANCIAL PLANNING

Financial planning is a crucial part of overall growth and development of
business. There are different approaches for financial planning at
individual level but the same can be applied for the business level as the
ultimate goal of financial planning is stability in future.

Let us understa nd the approaches of financial planning in detail.

1. Goal / Need Approach: - Financial planning can be made on the
basis of need or goal of the business. For the same the CFO should make
a list of the following points for perfect financial plan which can ai m at the
achievement of the objectives of the business. Financial planning of the
with the reference to the goals of business should be

a. Specific: - Financial planning should be specific. i.e., all the details
regarding finance of the business should be mad e available which
would make a plan proper and achievable.

b. Measurable: -Financial planning should be measurable in terms of
money. The finance to be raised and to be utilised should be giving
clear information about its cost, investment, returns on invest ment, etc.

c. Attainable: - Financial planning should be made for the task which are
attainable on the basis of available resources of the business.
munotes.in

Page 50


Financial Management
50 d. Realistic: -Financial planning should be realistic in nature.

e. Time bound: -Financial planning should be d one for the future period
stating the results and achievements of the business in the upcoming
years or a year. Short term of long term financial planning can be
done accordingly.

2. Net worth Approach: - The difference between the assets and the
liabilities of the business is nothing but Net worth of the business. Asset
includes all the fixed and current assets and also the investment made by
the business except fictitious assets. On the other hand liability includes
all the borrowed funds and current liabi lities and provisions except capital
and reserves. This approach helps a business to manage the funds to be
raised and plan the finance accordingly. Financial planning can be made
according to the assets that a business has so that they can be utilised in
future to payback the borrowed funds in future if the business suffers
losses.

3. Savings Approach: - Financial planning can be made on the basis of
saving strategies of the company/ business. The business having liberal
dividend policy will not aim at savi ng more. On the other hand, the
business having growth and development plans for the future would not
opt for more dividends to its shareholders.

4. Budget approach: - The best financial planning which a CFO can
adopt is the budget approach. Under this app roach a timely budget will be
prepared for meeting the expenses based on the income to be earned by
the business. This approach gives a clear picture of the financial
requirement of the business.

One can understand the importance of financial planning aft er
understanding different approaches of the same in the business. Financial
planning creates a clear picture of the future of the business in terms of
incomes to be earned and expenses to be made. It also helps to know the
financial position of the busine ss and to control the finance too.
4.3 FINANCIAL STATEMENTS
Financial planning is also includes estimates of the incomes and expenses
of the business and for the same some analysis is required based on past
figures of the business. Such analysis is done w ith the help of financial
statements of the business. The financial statements are normally
prepared at the end of the year and include profit and loss account and
balance sheet of the business. The statements prepared are normally
technical in nature and are with relevance to the accounting principles
coming under a special branch of accounting viz., Financial accounting.
Such financial statements are difficult for the analysis as everyone cannot
be aware with the accounting principles and procedures. A nalysis of
financial statements is done on the basis of the formats stated by another
branch of accounting viz., Management Accounting. Let us understand the
formats in details. munotes.in

Page 51


Financial Planning And
Forecasting
51
4.3.1 Vertical Income Statement
The profit and loss account which is the base of f inal accounts for the year
is also called income statement as it represents the incomes and expenses
for the particular year ended on the specified date. It is presented in a
vertical form taking sales as the base and finally revealing the profit or
loss for the year. Let us understand the pro -forma of the income statement
PRO -FORMA OF INCOME STATEMENT
Particulars Rs. Rs. Gross Sales xxx Less: - Returns xxx Net Sales XXX Less: - Cost of goods sold: - Opening Stock xxx Purchases xxx Direct expenses + manufacturing
expense xxx Factory expenses + depreciation on
factory assets xxx Less: - Closing Stock (xxx) Sale of scrap (xxx) Cost of Goods Sold (XXX) Gross Profit XXX Less: - Operating Expenses a) Administration Expenses xxx b) Selling and Distribution expenses xxx c) Finance Charges xxx Total Operating Expenses (a+b+c) XXX Net Operating Profit XXX Less: - Interest xxx Net profit after interest XXX Add: - Non operati ng income xxx Less: - Non operating expenses (xxx) Net Profit Before Tax XXX Less: - Income Tax (xxx) Net Profit After Tax XXX Add: - Opening Profit & Loss A/c
balance xxx Less: - Appropriation (xxx) Retained earnings XXX munotes.in

Page 52


Financial Management
52
 Figures in the brac ket indicates subtraction.
Illustration: -
1. Following is the profit and loss account summary of Well - balanced ltd
for the year ended 31st march 2022. You are required to prepare Vertical
income statement for the purpose of analysis.
Particulars Rs. Particu lars Rs.
To Opening Stock of raw
material 7,00,000 By Sales
To Purchases 9,00,000 Cash 5,20,000
To wages 1,50,000 Credit 15,00,000
To Factory Expenses 3,50,000 20,20,000
To office salaries 25,000 Less: - Returns 20,000
To office Rent 39,000 Net Sales 20,00,000
To postage and telegram 5,000 By Closing stock
of raw
To directors fees 6,000 material 6,00,000
To Salesman salaries 12,000 By dividend on
investment 10,000
To Advertising 18,000 By Profit on sale
of 20,000
To delivery expenses 20,000 Furniture
To Debenture Interest 20,000
To Depreciation on
furniture 10,000
To depreciation on Plant 30,000
To depreciation on
delivery van 20,000
To loss on Sale of van 5,000
To income tax 1,75,000
To Net profit 1,45,000
26,30,000 26,30,000

munotes.in

Page 53


Financial Planning And
Forecasting
53 Solution: -
Well balance Ltd.
Income statement for the year ended 31st march 2022
Particulars Rs. Rs. Gross Sales: - Cash 520000 Credit 1500000 2020000 Less: - Returns ( 20000 ) Net Sales 2000000 Less: - Cost of goods sold: - Opening Stock 700000 Purchases 900000 Wages 150000 Factory Expense 350000 Depreciation on plant 30000 Less: - Closing Stock 600000 Sale of scrap - Cost of Goods Sold (1530000) Gross Profit 470000 Less: - Operating Expenses a) Administration Expenses Office Salaries 25000 Office Rent 39000 Postage 5000 Directors Fees 6000 Depreciation of office f urniture 10000 b) Selling and Distribution
expenses Salesman salary 12000 Advertising 18000 Delivery Expenses 20000 Depreciation On Delivery Van 20000 c) Finance Charges -- Total Operating Expenses (a+b+c) 155000 Net Operating Profit 315000 Less: - Interest 20000 Net profit after interest 295000 Add: - Non operating income Dividend on Investment 10000 Profit on sale on furniture 20000 30000 325000 Less: - Non operating expenses Loss on sale o f van 5000 Net Profit Before Tax 320000 Less: - Income Tax 175000 Net Profit After Tax 145000 munotes.in

Page 54


Financial Management
54 4.3.2 Vertical Balance Sheet
The balance sheet is the statement of assets and liabilities showing the
financial position of the business on a particular date. It is not a ledger
account. The financial position or status of the company is disclosed by
the balance sheet and it is one of the basic documents required to be
presented for acquiring borrowed bund by the business. The position of
the assets owned and t he liabilities payable by the business shows its
ability to pay off the loans. The vertical balance sheet is divided in two
parts viz., Sources of Funds and Application of Funds. The sources of
Fund are discussed further in topic number 6 in detail.

Let us understand the pro -forma of vertical balance sheet

Particulars Rs. Rs. Rs. I ] SOURCES OF FUNDS A) OWNED/OWNER’S FUND 1. Capital xxx 2. Reserves xxx Less: - Losses and fictitious
assets (xx) xxx Owned/ Owner’s Fund XXX B) BORROWED FUND 1. Secured Fund / loans xxx 2. Unsecured Fund / loans xxx Borrowed Fund ( A + B) XXX Total Sources of Fund/ Capital
Employed XXX II] APPLICATION OF FUND A) FIXED ASSETS 1. Tangible assets xxx 2. Intangible assets xxx Total fixe d assets XXX B) Long term Investment XXX C) Working Capital 1. Current Assets xxx Total current assets xxx Less : 2. Current Liabilities xxx Total current liabilities (xxx) Working Capital XXX Total Application of Fund XXX

munotes.in

Page 55


Financial Planning And
Forecasting
55 *the above format / pro -forma changes from business to business as
per the requirements and the financial plan.
Illustration: -
2. Following is the summary balance sheet of Abhijeet Ltd. As on 31st
March 2015.
Liabilities Amo unt
(rs) Assets Amount
(rs)
Equity Share Capital 3,90,000 Cash in hand 15,000
10% Preference
Share 2,00,000 Cash at Bank 90,000
9% Debenture 2,50,000 Preliminary
Expenses 20,000
General Reserve 60,000 Goodwill 1,00,000
Capital Reserve 50,000 Building 3,00,000
11% Bank Loan 1,00,000 Investment (long
term) 2,00,000
Creditors 1,25,000 Furniture 2,50,000
Bank Overdraft 1,35,000 Plant and Machinery 3,00,000
Provision for tax 1,40,000 Debtors 1,50,000
Proposed Dividend 30,000 Prepaid Expenses 50,000
Profit & Loss A/c 1,40,000 Stock 2,00,000
Depreciation
provision 80,000 Calls in
arrears(equity) 10,000
Commission on
issue of Shares 15,000
17,00,000 17,00,000

Present the above balance sheet in vertical form and show Net worth,
Borrowed Fund, Capi tal Employed, Net Block, Working Capital and
Fictitious assets.



munotes.in

Page 56


Financial Management
56 Solution: -
Particulars Rs. Rs. Rs.
I ] SOURCES OF FUNDS
C) OWNED/OWNER’S
FUND
Capital
Equity share capital 390000
Less: - calls in arrears (10000) 380000
Preference shar e capita 200000
Reserves
General reserves 60000
Capital reserves 50000
Profit & Loss account 140000
Less: - preliminary expenses (20000)
Commission on issue
of shares (15000) 215000
Total owned fund / Net
worth 795000
D) BORROWED FUND
Secured Fund / loans
9% Debentures 250000
Loans from bank 100000
Unsecured Fund / loans ---
Borrowed Fund ( A + B) 350000
Total Sources of Fund/
Capital Employed 1145000
II] A PPLICATION OF
FUND
D) FIXED ASSETS
Tangible assets
Land and building 300000
Furniture 250000
Plant and machinery 300000
Less: - Depreciation ( 80000
) 770000
Intangible assets munotes.in

Page 57


Financial Planning And
Forecasting
57 Goodwill 100000
Total f ixed assets 870000
E) Long term
Investment 200000
F) Working Capital
Current Assets
Cash 15000
Bank 90000
Debtors 150000
Inventory 200000
Prepaid expenses 50000
Total current
assets 505000
Less : Current Liab ilities
Creditors 125000
Provision for Tax 140000
Outstanding
expenses 300000
Total current
liabilities 430000
Working Capital 75000
Total Application of Fund 1145000

ANALYSIS OF FINANCIAL STATEMENTS:
Financial statements are further analysed for the business decisions in
different forms. The balance sheet and the profit and loss account
presented vertically are further presented in different format depending
upon the goals, requirement, Operations etc. of the business. Each of its
analysis has its own importance and can be studied independently.
Analysis of financial statements is important to take timely decisions for
the growth and development of the business. Apart from this such analysis
is also helpful to avoid losses for the business.
The different types of presentations for analysis of financial statements are
1. Trend analysis
2. Comparative analysis
3. Common size analysis
4. Ratio analysis
However, the above are not dealt in detail as students have learnt the
above in previous standards. munotes.in

Page 58


Financial Management
58 4.4 COMPUTATION OF EXTERNAL FINANCING
REQUIREMENTS

External finance of the business is in the form of borrowed funds are they
are to be paid back after the maturity. External Financial source s are paid
high interest and so at times the cost of raising such funds is more for the
company. Only the share capital and retained earnings are the internal
sources for the company. Even though shareholders are outsider i.e, from
general public, they ar e the owner of the company. Hence they are treated
internal source of finance. The company needs to be very wise in selecting
external financing requirements as one wrong decision may affect the
operation of the company

Usually external finance is less t han the internal finance of the company.
The company needs to find the proportion of external finance to the
internal finance. This is done with the help of ratio analysis and vertical
statements too. We have already seen the different types of vertical
statement and its application. The relationship between internal and
external finance can be very clearly understood with the help of ratio
analysis. There are different types of ratios as stated below.
 Balance sheet ratios
 Revenue ratios
 Combined / comp osite ratios
Composite ratios are more reliable to find the ratio of external and internal
finance. This is because composite ratios includes the amount of profit
from profit and loss account on one hand and the amount of owned and
borrowed fund on other hand. The profit earned by the firm is paid to the
borrowed fund in the form of interest and then dividend is paid to the
shareholders. Let us understand the composite ratios with reference to
external finance in the form of mathematical formulas.
a. Return on investment = Profit before interest and tax
Capital employed

b. Return on proprietor fund = Net profit after tax
Proprietor fund

c. Debt Service Ratio = Net profit before interest and tax
Interest

d. Debt service coverage rati o = Cash Profits
Interest + instalments
(The above formulas can be applied by the learner for the sums given later
in the exercise)

munotes.in

Page 59


Financial Planning And
Forecasting
59 4.5 SUMMARY
 Financial planning is important for every business in order to manage
its cash requirements
 Financial plann ing and Financial management are different from each
other
 Financial forecasting is one of the important aspect to know the flow
of cash in the business
 Financial statements include profit and loss account, balance sheet and
all relevant records of account s.

4.6 EXERCISE

A. Answer the following questions.
1. What do you mean by financial planning? Explain the objectives of
financial planning.
2. Explain the concept of financial forecasting. State the advantages of
financial forecasting.
3. What are the different approach es of financial planning?
4. State different types of financial statements and state its components.

B. Solve the following : -
1. From the following Balance Sheet , Prepare a vertical balance sheet for
the year ending 31st march 2015.
Particulars Rs.
Cash and Bank 6000
Land and Building at cost less
Depreciation 40000
Prepaid Expenses 10000
Stock 30000
Creditors 8000
General Reserve 14000
Debtors 18000
Preliminary Expenses 3000
Plant and machinery less depreciation 52000
Term Loan from bank 35000
Bank Overdraft 18000
Capital 80000
Profit and Loss account 16000
Marketable investments 10000
Advance payment of tax 18000
Provision for tax 16000
munotes.in

Page 60


Financial Management
60 2. Prepare a vertical balance sheet for M/s Laxman Ltd for the year
ending 31st march 2015.
Particulars Rs. Part iculars Rs.
Sundry Debtors 2,00,000 Creditors 1,50,000
Trade Investments 2,50,000 Capital Reserve 1,50,000
Bank Overdraft 1,00,000 Short term
investments 50,000
Public Deposits 3,00,000 Plant and
Machinery 12,00,000
Bills Payable 7,90,000 Outstanding
expenses 1,20,000
General Reserve 1,00,000 Cash and bank 7,,00,000
Bills Receivables 2,00,000 Profit and loss a/c
(cr) 4,00,000
Vehicles 9,00,000 Stock 5,00,000
10% Preference Share
Capital 8,00,000 Land and building 12,00,000
Commission on issue of
shares 40,000 Equity share
capital 16,00,000
Provision for tax 1,00,000 Preliminary
expenses 10,000
Bank Loan 3,00,000 Debenture 5,00,000
Advance Tax 3,00,000 Proposed Dividend 3,00,000
Prepaid Expenses 1,00,000 Advance to
suppliers 60,000














munotes.in

Page 61


Financial Planning And
Forecasting
61
3. Re-arrange the following information in the suitable form for
analysis for the year ending 31st March 2015.
Particulars Rs. Particulars Rs.
Sales 20,00,000 Return inwards 50,000
Opening stock
of raw material 1,10,000 Purchases of Raw
materials 5,00,0 00
Staff salaries 1,50,000 Commission allowed 5,000
Salesman
salaries 25,000 Proposed dividend 1,50,000
Bank charges 10,000 Exhibition expenses 35,000
Freight inwards 40,000 Repairs of Computers 5,000
Office Rent and
Insurance 45,000 Closing stock of w-i-p 40,000
Debenture
interest 50,000 Wages 70,000
Loss on sale of
machinery 10,000 Purchases of finished
goods 80,000
Printing and
Stationery 5,000 Interest received ,40,000
Direct
Expenses 50,000 Provision for income
tax 2,00,000
Profit and Loss
account 2,40,000 Closing stock of
material 80,000
Depreciation
on patterns 10,000 Sale of scrap 20,000
Depreciation
on Machinery 20,000






munotes.in

Page 62


Financial Management
62 4. From the following information, prepare vertical income statement and
balance sheet.
Particulars Rs. Particulars Rs.
Equity Share Capital 225000 Sales 855000
Plant and Machinery 45000 Debentures 50000
Purchases 655000 Interim Dividend
Paid 15000
Wages 85000 Depreciation 15000
Bank Overdraft 20000 Office Salaries 15000
Office Rent 5000 Dividend Received 5000
Advertisement 20000 Goodwill 25000
Finance Expenses 8000 Land and Building 48000
Income tax 15000 Creditors 25000
Preliminary
Expenses 5000 Trade Investment 75000
Bills payable 15000 Returns to suppliers 5000
Net Profit (opening) 13000 Debtors 65000
Opening Stock 75000 Cash 42000

Closing Stock was valued at Rs.80,000.
5 From the following financial statements for the year ended 31st
March 2015 submitted to you, prepare the vertical statements: -
Dr. Trading and Profit and Loss account for the year ending
31/03/15 Particulars Rs. Particulars Rs.
To Opening Stock 70000 By Sales 1660000
To Purchase
1530000 By Closing
Stock 160000
Less: - Returns
30000 1500000
To Gross Profit 250000
1820000 1820000
To Depreciation 36000
To Admin Overheads 50000
To Selling &
Distribution 24000
To Provision of Tax 40000
To Proposed Dividend 16000
To Profit Balance 94000
260000 260000
munotes.in

Page 63


Financial Planning And
Forecasting
63 Balance sheet as on 31/03/15
Liabilities Rs. Assets Rs
Share Capital 300000 Goodwill 20000
P & L Account 180000 Cash in hand 8000
Proposed
Dividend 16000 Stock in trade 160000
Bank Overdraft 38000 Sundry Debtors 178500
Sundry Creditors 26000 Land & Building 92150
Provision for
Depreciation 55750 Plant & Machinery 128600
Provision for Tax 40000 Prepaid Expenses 1500
Expenses on issue
of shares 7000
Short term
investment 60000
655750 655750

4.7 REFERENCES
1) Management Accountancy J.Betty.
2) Management Accounting Manmohan Goyal.
3) Princip les o f Management Accounting S.N.Maheshwari
4) Management Account ing Principles & Practices M.A.Saraf.
5) Management Accounting Hingorani.
6) Manage ment Accounting I.M.Pandey.
7) Management Accounting M.Y.Khan , K.P.Jain .
8) Management Accounting M.G. Patkar.

munotes.in

Page 64

64 5
INVESTMENT ( PROJECT )
IDENTIFICATION AND CAPITAL
BUDGETING
Unit Structure:
5.0 Objective
5.1 Introduction
5.2 Investment (Project) identification, feasibility analysis with
sensitivities, constraints and long term cash projection.
5.3 Capital Bu dgeting and Investment decision analysis
5.4 Illustrations
5.5 Summary
5.6 Exercise
5.7 References
5.0 OBJECTIVE
After learning this topic, students will be able to understand
 Project and its feasibility analysis
 Capital budgeting and its computation
 Application of various approaches under capital budgeting.
5.1 INTRODUCTION
A business is not a one -time activity and so it involves number of
processes which are done sequentially. It is very natural that the business
which operates smoothly aims at investin g amount in different projects.
The word ‘Project’ means sequence of tasks which are done to achieve the
desired outcome. In the words or Turner, project means ‘an endeavour in
which human, material and financial resources are organized in a novel
way, to undertake a unique scope of work, of given specification, within
constraints of cost and time, so as to achieve beneficial change defined by
quantitative and qualitative objectives.’ It can be summarized that
projects are involving huge investment and in volvement of human
resource too and is undertaken with a desired objective.
5.2.1 INVESTMENT / PROJECT IDENTIFICATION: -
Investment / Project is not a routine activity of the business. It is done
with a view to achieve something or to reach a new height in the business munotes.in

Page 65


Investment ( Project )
Identification and capital
budgeting
65 and industry. Accordingly, project can be classified on the basis of size
viz. Mini project or Mega project. Project or investment can also be
classified on the basis of time viz. short term and long -term investment or
project. Every inves tment basically has three basic characteristic which are
limited, unique and risky. Speaking in detail, every investment should be
limited in terms of money and operation. As the success of project is only
after completion, it is always advisable to be li mited. Secondly
investment should be unique in nature. Every business tries to do
something different as compared to others. So that the uniqueness in
investment may bring maximum returns on investment in future. Thirdly
investment or project is risky. A s said earlier, the success of the project or
investment is known only after completion of the project.
5.2.2 FEASIBILITY ANALYSIS
Feasibility analysis in project management knows the practicality of the
project to be invested in. Such analysis is helpfu l to know the practical
working of the project in which the investment is or is to be made. It is
done to know the feasibility of the operation and technical application of a
project with keeping in mind the economical aspect. Such analysis helps to
take d ecision regarding the durability of the project. The normal procedure
of feasibility analysis is done as follows: -
 Conduct a preliminary analysis to describe the market conditions
 Create an income statement to estimate the income from the project
 Conduct m arket research to know the applicability of the project
 Plan business organisation and operation to accomplish the project
successfully
 Create opening balance sheet to understand the pre project financial
status of the business
 Review and analyse the proje ct success rate
 Decide YES / NO for the project
Feasibility study can be classified as: -
 Legal feasibility: - to understand the legal framework regarding a
project
 Economic feasibility: - to understand the economic aspects of a
project
 Technical feasibilit y:- to know the technical knowhow of a project to
be undertaken.
 Operational feasibility: - to check the practical application of the
project
 Scheduling feasibility: - to know the duration of the project
5.2.3 CASH FLOW PROJECTION: -
Cash is very well know n as BLOOD of any business and hence it becomes
important to manage cash of the business wisely. Proper care must be
taken while taking a project i.e investing in a particular project keeping the
cash requirement and the cash earnings in long run. As long run munotes.in

Page 66


Financial Management
66 investments are huge and made for capital assets, the return on investment
is slow and risky in nature. Hence, the Cash Outflow – the initial
investment and the Cash Inflow – the return on investment should be
appropriately estimated. The technique o f estimating the cash in future is
Cash Flow projection. One should remember that Cash flow projection in
project management is not the same as Cash Flow Statements prepared in
day-to-day accounting of the business operations. Long term cash flow
projecti on is a practice of management that helps to understand the impact
of investment in future which is made today.
5.3 CAPITAL BUDGETING AND INVESTMENT
DECISION ANALYSIS
Capital budgeting involves decision making regarding the investment in
capital assets wh ich are one of the crucial decisions to be taken in the
business as such decision is irreversible. The amount invested in capital
assets is huge and long term in nature. Some of the examples of capital
investments are purchase of machinery, purchase of lan d and building,
replacement of plant and machinery, selecting proper alternative of plant
to purchase, expenditure on research and development, etc.
Some of the important features of Capital Budgeting are: -
 Large amount is involved to be invested.
 Long period investment
 Uncertainties of outcome in futures for the decision taken at present.
 Decision taken by top level management as it is for the growth,
development and existence of the business.
 Irreversible as decision once taken cannot be taken back.
Capital budgeting involves some process which is normally followed by
every business which is as follows: -
 Project Generation :- It involves making of proposal for capital
investment. Such proposal may be for increasing the revenue in future
or reducing the cost and it may be done by any level of management.

 Project Evaluation :- Reviewing the proposal in terms of cash
flow to be generated, operational benefits to be arrived after
investment etc. are to be done at this stage.

 Project selection :- Selecting t he appropriate project of investment
depending upon the needs and aim of the business in near future is
done at this point and it can be done by any level of management even
though the execution is done by top level of management.

 Project Execution :- Investment of the amount actually in the selected
project is project execution. Proper care should be taken in allocation
of fund at this stage as the huge amount is to be blocked for long term
period. munotes.in

Page 67


Investment ( Project )
Identification and capital
budgeting
67  Follow up :- Follow up is required to check the gap between the
estimates and actual performance of the project selected. A system
should be generated to check the performance of the selected project in
daily operations of the business.
Cash Flow in Capital budgeting: -
Capital budgeting involves huge cash investm ent as discussed above
which is to be compared with the incomes to be generated in future. On
one hand the cash invested results in outflow of cash and the income to be
generated in future shows the inflow of cash. Normally, the inflow is an
estimate and it will help to take the decision of investment in capital
assets. Hence, the inflow of cash should be more than the outflow of cash
which will result in profitability and financial stability along with growth
and development of the business. Let us under stand the elements of cash
inflow and cash outflow.
Cash Inflow: -
Cash Inflow includes: -
 Operating Inflows : -The cash generated from the operations of the
business which is the net profit in real sense. The mathematical
formula for operating cash inflow is NPAT + Depreciation.

Depreciation is a non -cash expense hence added back to the profit
 Salvage: - Salvage is the realisable value of the asset at the completion
of its economic life. The amount of Salvage should be considered as
inflow in the year of co mpletion of economic life.

 Working capital: - Capital investment also results in increase in
working capital to be set aside for the smooth working of business. It
is an outflow of cash in beginning but the amount of working capital is
added to the total inflows at the end of the project life.

 Terminal inflows: - sum total of salvage and working capital recovered
in the last year are together called terminal inflows.
Cash outflow: -
Cash outflow includes: -
 Cost of new project: - Initial cost of investment to be made.
 Cost of installation to be made
 Working capital : - The amount of increased working capital required
due to investment in new project is locked up till the end of the project
life. Gradually it is adjusted against the inflows and not considered as
the capital cost of the asset. In the last year, after completion of the life
of asset, such working capital is treated as cash inflow.
 Proceeds from old asset: - If the new investment is for replacing an old
asset, the proceeds from selling the old asset are usually utilised to
invest in the new project. munotes.in

Page 68


Financial Management
68 METHODS OF EVALUATIONS: -
we have seen that investment decision are taken by evaluating the cash
inflow and cash outflow. However there are different methods of
evaluating the investment decision which are as follows: -
1. Payback period method
2. Payback Profitability Method
3. Average/ Accounting Rate of Returns Method
4. Net Present Value Method
5. Discounted Payback Period Method.
6. Profitability Index Method.
7.
Let us understand each method in detail.

1. Payback Period Me thod: -
In simple words Payback period is the period taken by the investment to
recover the amount of initial investment of the project. For example, The
initial investment of the machinery is Rs.10 lakhs and the estimated inflow
the cash is Rs. 2 lakh ann ually, then it will take 5 years to recover the cost
of investment. This period of 5 years is called Payback Period. Higher the
amount of cash inflow, lesser is the period of Payback and vice versa. The
mathematical formula for calculating Payback period is as follows
Payback Period = Cost of investment / cash inflow p.a
Incase where investment is to be made in exclusive projects one can
compare the payback period and select the project having the lesser
payback period.

2. Payback Profitability Method: -
Incase of Payback period method, the time of recovery of initial
investment was the only constraint which was given importance. The
overall amount of cash inflows till the end of economic life of the asset
was not considered. To avoid this limitation Payback Profitability method
came into existence which considered the profitability of the investment
made in the project. Profitability is the total of surplus income earned over
the cost of investment during the lifetime of the asset.

The mathematical formula for calculating payback profitability is as
follows

Payback Profitability = Total Cash inflows – Cost of investment.

3. Average / Accounting Rate of Returns (ARR): -
Under this method, the average or accounting profit is considered as the
base of evaluatio n. The profit is presented in the form of percentage which
it bears to the amount invested.

The mathematical formula for calculating average/ accounting rate of
return is

munotes.in

Page 69


Investment ( Project )
Identification and capital
budgeting
69 ARR = average profit after tax / average capital invested where
Average pro fit = total profit after tax / no. of years and
Average Capital invested = total capital invested / 2
OR
Average Capital invested = total capital invested / 2 + salvage +
working capital

4. Net Present Value Method (NPV): -
Under this method, the esti mated cash inflows of each year are converted
into the discounted rate. Such rate is normally the interest rate to be treated
as discount rate. Net present value is the difference between the
discounted cash inflows at a specified rate and the amount inve sted in the
project. If the difference is positive, normally the project is accepted and
investment is made in the desired project. The mathematical formula is as
follows
NPV = PV of cash inflows – Cost of investment where
PV is the present value 1 /


5. Discounted Payback Period Method: -
This method is combination of the payback period method and the Net
Present Value Method. The initial amount of investment is divided by the
discounted cash inflows instead of total cash inflows.
The mathema tical formula to find the discounted payback period is
Discounted Payback Period Method = Cost of Investment /
Discounted Cash Inflows per annum

6. Profitability Index Method: -
The above stated methods are useful to select the projects having similar
cost of investment. But if there are projects having different level of
investments then it becomes difficult to select the appropriate project on
the basis of profitability index or Net Present Value. To overcome this
shortcoming, the profitability index meth od is useful when the level of
investments differs in different projects.

The mathematical formula for calculating the Profitability Index Method
is as follows
Profitability Index = PV of Cash Inflows / PV of Cash Outflows.











munotes.in

Page 70


Financial Management
70 5.4 ILLUSTRATIONS
1. The mana gement of P Limited is considering selecting a machine
out of two mutually exclusive machines. The company’s cost of capital
is 12%. Details of the machines are as follows: -
Machine A Machine B
Cost of machine Rs.10,00,000 Rs.15,00,000 Expected life 5yrs 6yrs Annual cash flow Rs.3,01,500 Rs.3,93,500
You are required to calculate the discounted pay-back period and net present
value.
Solution: -
For Machine A : - Cost Rs,10,00,000
Year Cash
inflow Discount
@12 Present
value Cumulative
prese nt value 1 3,01,500 0.893 2,69,240 2,69,240 2 3,01,500 0.797 2,40,296 5,09,536 3 3,01,500 0.712 2,14,668 7,24,204 4 3,01,500 0.636 1,91,754 9,15,958 5 3,01,500 0.567 1,70,951 10,86,909 10,86,909
Discounted payback period = 4 years + 1000000 -915958
170951
= 4 years + 0.49
= 4.49 years
Net Present Value = 10,86,909 – 10,00,000
= Rs.86909
For Machine B: - Cost 15,00,000
Year Cash
inflow Discount
@12 Present
value Cumulative
present value 1 3,93,500 0.893 3,51,396 3,51,396 2 3,93,500 0.797 3,13,620 6,65,016 3 3,93,500 0.712 2,80,172 9,45,188 4 3,93,500 0.636 2,50,266 11,95,454 5 3,93,500 0.567 2,23,115 14,18,569 6 3,93,500 0.507 1,99,505 16,18,074 16,18,074 munotes.in

Page 71


Investment ( Project )
Identification and capital
budgeting
71 Discounted Payback period = 5years + 1500000 – 1418569
199505
= 5 years + 0.41
= 5.41 years
Net Present Value = 1618074 – 1500000
= Rs. 118074
2. A company has to make a choice between two projects namely A
and B. the initial capital outlay of two projects are Rs.1,35,000 and
Rs.2,40, 000 respectively for project A and B. There will be no scrap
value at the end of the life of both the projects. The opportunity cost of
capital of the company is 16%. The annual incomes are as under: -
Years Project A Project B discounting factor
1 -
- 60,000 0.862
2 30000 84,000 0.743
3 132000 96,000 0.641
4 84000 1,02,000 0.552
5 84000 900,00 0.476

You are required to calculate the Profitability Index and Net present Value of both
the projects.
Solution: -
For Project A: - Cost Rs.1,35,000
Year Cash
inflow Discount
@12 Present
value Cumulative
present value 1 -- 0.862 -- 2 30000 0.743 22290 22290 3 132000 0.641 84612 106902 4 84000 0.552 46368 153270 5 84000 0.476 39984 193254 193254
munotes.in

Page 72


Financial Management
72 Profitability Index = Total discounted cash inflow
Initial Cash outflow
= 193254
135000
= 1.43
Net Present Value =Total discounted cash inflow – total cash outflow
= Rs. 193254 – Rs.135000
= Rs. 58254
For Project B: - Cost Rs. 2,40,000
Year Cash
inflow Discount
@12 Present
value Cumulative
present value 1 60000 0.862 51720 51720 2 84000 0.743 62412 114132 3 96000 0.641 61536 175668 4 102000 0.552 56304 231972 5 90000 0.476 42840 274812 274812
Profitability Index = Total discounted cash inflow
Initial cash outflow
= 274812
240000
= 1.15
Net Present Value = Total Discounted cash inflow – total
cash outflow
= Rs. 274812 – Rs.240000
= Rs.34812




munotes.in

Page 73


Investment ( Project )
Identification and capital
budgeting
73 3. PQR Ltd is considering to select a machine out of two mutually
exclusive machines. The company’s cost of capital is 12%. Other
information relating to both machines is as follows: -
Particulars Machine I Machine II Discounting factor for
5 years
Cost of
machine Rs.1500000 Rs.2000000 1 0.893
Expected life 5yrs 5yrs 2 0.797
Annual cash
flow Rs.527500 Rs.732500 3 0.712
4 0.636
5 0.567

You are required to calculate Discounted payback period, Net Present Value and
Profitability Index.
Solution: -
Statement of Cash flow
Year
Annual Cash
Inflow Discounting
factor
@12% Discounted cash
inflow Cumulative
discounted inflow
Machine
I (Rs) Machine
II (Rs.) Machine
I (Rs.) Machine
II(Rs.) Machine I (Rs.) Machine II (Rs.)
1 5,27,500 7,32,500 0.893 471058 654123 471058 654123 2 5,27,500 7,32,500 0.797 420418 583803 891476 1237926 3 5,27,500 7,32,500 0.712 375580 521540 1267056 1759466 4 5,27,500 7,32,500 0.636 335490 465870 1602546 2225336 5 5,27,500 7,32,500 0.567 299093 415328 1901639 2640664 1901639 2640664
Cost of investment (Outflow of cash) 1500000 2000000
Net Present Value 401639 640664



munotes.in

Page 74


Financial Management
74 i. Discounted Payback Period:
For machine I = 3years + ( 1500000 – 1267056)
527500
= 3years +0.44
= 3.44 years
For Machine II =3 years + (2000000 – 1759466 )
732500
= 3 years + 0.33
= 3.33 yea rs
ii. Net Present Value = Total discounted cash inflow - Cash outflow
For Machine I = Rs.401639
For Machine II = Rs.640664

iii. Profitability Index = Total discounted cash inflow
Total cash outflow
For Machine I = 1901639
1500000
= 1.27
For Machine II = 2640664
2000000
= 1.32
4. A firm has an investment opportunity costing Rs.50,000 with
following expected net cash flows after taxes and before depreciation.
Year Net Cash Flows (Rs.)
1 8000
2 9000
3 10000
4 13000
5 12000
6 10000
7 8000
8 2000

munotes.in

Page 75


Investment ( Project )
Identification and capital
budgeting
75 Using 10% as the cost of capital, evaluate the project taking 10% discount factor
for NPV by :- Payback Period, Pay back Profitability, Net Present Value and
Profitability Index.
Solution: -
Year Cash
inflow Discount
@10 Present
value Cumulative
present value 1 8000 0.909 7272 7272 2 9000 0.826 7434 14706 3 10000 0.751 7510 22216 4 13000 0.683 8879 31095 5 12000 0.621 7452 38547 6 10000 0.564 5640 44187 7 8000 0.513 4104 48291 8 2000 0.467 934 49225 82000 49225
Payback Period (on the basis of cash inflow) = 4 years + 8000
12000
= 4.67 years
Payback Profitability = Total cash Inflow – Total Cash outflow
= 82000 – 50000
= Rs.32000
The discounted payback profitability is negative
Net Prese nt Value = Discounted Cash inflow – total cash outflow
= Rs.49225 – Rs.50000
=(Rs.775)
Profitability index = Total Discounted cash inflow
Cash outflow
= 49225
50000
= 0.9845


munotes.in

Page 76


Financial Management
76 5.5 SUMMARY
 Investment is done in differ ent level to keep the business in operation.
 Such investment may be for growth, development, diversification of the
business
 Feasibility study is the study of a particular investment project in terms
of its actual application in near future.
 Capital budget ing is useful in decision making process for investment of
fixed assets
 Capital budgeting is based on certain assumptions and certain methods
on the basis of returns to be earned on the investment.
 NPV is the present value of the income to be generated in future at the
fixed interest rate.
 Future value is the value of current investment at a future date at a fixed
rate of interest.
 Discounted rate is the rate of Re.1 at an interest rate with reference to a
particular period.
 Profitability is the capacity o f the project to earn profit in future against
the investment made at present.
5.6 EXERCISE
A. Write short answers
1. What do you mean by capital budgeting? Explain different types of
investment projects.
2. Explain the process of feasibility analysis of project i nvestment.
3. Explain the concept of cash flow in capital budgeting.
4. State different methods of decision making in capital budgeting. Or
evaluating investment proposals.

B. Select the most appropriate answer

1. Payback period = ___________________
Annual Cash Inflow
a. Cash
Flow b. Cash Inflow c. Cash
Outflow d. Total Cash
Inflow

2. If the average rate of return of a project is _________ 1, it
should be accepted.
a. Less
than b. More
than c. equal d. none of
the above
3. _________ affe cts cash flow
a. Depreci
ation b. Interest c. Capital d. Investment

munotes.in

Page 77


Investment ( Project )
Identification and capital
budgeting
77 4. Capital budgeting involves ________ term investment.
a. short b. long c. medium d. all the above

5. The excess of PV of cash inflow over cash outflow is called
___________
a. ARR b. Discou
nted Payback c. IRR d. Net
Present Value

6. Financial position of the business is provided by
__________
a. Balance
Sheet b. Reven
ue Statement c. Cash
Flow d. Fund flow


7. _____________ decision are taking by capital budgeting
evaluation.
a. Day to
day activities b. Acquis
ition of fixed
assets c. Both
the above d. None of
the above


8. The techniques of Capital Budgeting are ___________
a. Payback
Period b. ARR c. NPV d. All the
above

9. Payback period is the period required to ____________
a. Recover
the original
investment b. Deprec
iate asset c. Pay the
creditors d. Recover
from debtors

10. The amount spent on fixed assets is expected to give return
over ______ years
a. one b. two c. three d. many









munotes.in

Page 78


Financial Management
78 C. Practical Problem
Q.1. DB ltd. is producing articles mostly by annual labour and its
consideration to replace it by a new machine. There are two alternative
models D and B of the new machine. Prepare a statement of profitability
showing the pay –back period from the following information:
Machine D Machine B
Estimated life of machine 5 Years 6Years
Cost of machine Rs.10,000 Rs.24,000
Estimat ed saving in scrap Rs.1,000 Rs.1,000
Estimated saving in direct
wages Rs.7,000 Rs.10,000
Additional cost of
maintenance Rs.750 Rs.1,500
Additional cost of
supervision Rs.1,500 Rs.2,000

Q.2 A concern is considering two projects A and B. Following ar e the
particulars in respect of them:
Project A Project B
Cost(Rs.) 2,00,000 2,00,000
Economic life (in year’s) 10 10
Estimated scrap (in Rs.) 15,000 20,000
Annual saving 40,000 30,000

Ignoring income tax, recommend the best of these p roject using (a) pay –
back period, (b) post pay -back profit, and (c) Index of post pay back
profit.





munotes.in

Page 79


Investment ( Project )
Identification and capital
budgeting
79 Q.3 The following are the particulars relating to a project.
Rs.
Cost of the project 1,00,000
Operating Savings:
1st Year 10,000
2nd year 40,000
3rd year 60,000
4th year 60,000
5th year 20,000

Calculate (a)pay - back period ignoring interest factor and (b) discount
pay-back period taking into account interest factor at 10%.
Q.4 Calculate the average rate of return for project X and Y from the follo wing:
Project X Project Y
Investment
Rs.30,000 Rs.50,000
Expected Life (no salvage v alue) 3 year 5Year
Project Net Income, after interest, depreciation and taxes:
Year Project X (Rs.) Project Y (Rs.)
1 4,000 6,000
2 3,000 6,000
3 3,000 4,000
4 2,000 2,000
5 ---- 2,000
Total 12,000 20,000

Q.5 A Company is considering the purchase of the two machine the
following details.
Machine I Machine II
Estimated life 5 Year 5 Year
Capital Cost (Rs.) 1,00,000 1,00,000
Net earnings after tax: munotes.in

Page 80


Financial Management
80 (Rs.)
1st year 80,000 20,000
2nd year 60,000 70,000
3rd year 40,000 1,00,000

You are required to suggest which machine should be preferred.
Q.6 Initial Investment of the asset is Rs.6,000 and the economic life is
4years. The Estimated net annual cash flows for next 4 years are:
Rs.1,5 00, Rs.2,000, Rs.3,000 and Rs.2,000. Calculate Internal Rate of
Return.

Q.7 The initial cash outlay of a project is Rs.1,00,000. The estimated
cash inflows:
1st year Rs.40,000
2nd year Rs.30,000
3rd year Rs. 50,000
4th year Rs. 20,000
Compute profitabi lity Index.
Q.8 Z Ltd. is considering the purchase of a machine. Two machine are
available, X and Y. The cost of each machine is Rs. 1, 00,000. Each
machine has an expected life of 5 year. Net profit before tax but after
depreciation during the expected l ife of the machine is given below:
Year Machine X (Rs.) Machine Y(Rs.)
1 20,000 10,000
2 25,000 20,000
3 30,000 25,000
4 20,000 35,000
5 15,000 25,000

Following the method of return on investment ascertain which of the
alternatives will be more prof itable. The average rate of tax may be taken
at 50%.

munotes.in

Page 81


Investment ( Project )
Identification and capital
budgeting
81 Q.9 D Ltd. company is considering the purchase of a new machine which
will carry out some operation performed by labour. A and B are
alternative models. From the following information, you are required t o
prepare a profitability statement and work out the pay - back period for
each model.
Models A (Rs.) Models B
(Rs.)
Estimated Life 5 year 6 year
Cost of Machine 2,00,000 3,00,000
Cost of indirect material 56,000 58,000
Estimated savings in scrap 60,00 0 65,000
Additional cost of maintenance 69,000 67,000
Estimated saving in direct wages:
Employee not required 250 300
Wages per employee 900 900

Taxation to be regarded 50% of profit before charging depreciation.
Which model would you recommend?
Q.10 A company proposing to expand its production can go either for an
automatic machine costing Rs.3,24,000 with an estimated life of 5 ½ years
or an ordinary machine costing Rs.1,00,000 having an estimated life of 8
years.
The annual sales and
cost are e stimated as
follows : Automatic Machine
(Rs.) Ordinary Machine
(Rs.)
Sales 3,00,000 3,00,000
Cost:
Materials 1,00,000 1,00,000
Labour 24,000 1,20,000
Variable Overhead 42,000 40,000
Compute the comparative profitability under pay back metho d.



munotes.in

Page 82


Financial Management
82 Q.11 The Pari Ltd. is considering a proposal for the investment of
Rs.8,00,000 on product development which is expected to generate net
cash inflows for 6 year as under:
Year Net cash Flows (‘000)
1 Nil
2 160
3 240
4 300
5 600
6 700

The following are the present value factors @15% p.a
Year 1 2 3 4 5 6
Factors 0.87 0.76 0.66 0.57 0.50 0.43

Q.12 The financial manager of a company has to advise the Board of
Directors on choosing between two compelling project proposal which
required an equal investment of Rs. 2,00,000 and are expected to generate
cash flows as under:
Year Project I (Rs.) Project II (Rs.)
1 96,000 40,000
2 64,000 48,000
3 40,00 72,000
4 Nil 96,000
5 48,000 32,000
6 24,000 16,000

Which project proposal sho uld be recommended and why ? Assume the
cost of capital to be 10% p.a . The following are present value factors at
10% p.a.
Year 1 2 3 4 5 6
Factors 0.909 0.826 0.751 0.683 0.621 0.564


munotes.in

Page 83


Investment ( Project )
Identification and capital
budgeting
83 Q.13 From the following, calculate the net present value of the two project
and suggest which of the two profits should be accepted assuming a
discount rate of 10%.
Profit A (Rs.) Profit B (Rs.)
Initial Investment 30,000 45,000
Estimated Life 5 Year 5 Year
Scrap value 1,500 3,000

Profit before depreciation and after taxex are as follows:
Year Profit A (Rs.) Profit B (Rs.)
1 7,500 30,000
2 15,000 15,000
3 15,000 7,500
4 4,500 4,500
5 3,000 3,000

Q.14 A firm is considering the purchase of a machine. Two machine X
and Y are available, each costing Rs. 1,00,000. I n comparing the
profitability of those machines a discount rate of 10% is to be uded.
Earning after taxation are expected to be as follows:
Year Machine X Cash
Inflows Machine Y Cash
Inflows
1 30,000 10,000
2 40,000 30,000
3 50,000 40,000
4 30,000 60,000
5 20,000 40,000

You are also given the following data:
Year PV Factors @ 10%
discount
1 0.909
2 0.826
3 0.751
4 0.683
5 0.621
munotes.in

Page 84


Financial Management
84 Evaluate the project using:
a) The pay back period
b) The accounting rate of return
c) The net present value
d) The profitabilit y index
5.7 REFERENCES
RECOMMENDED BOOKS
1) Management Accountancy J.Betty.
2) Management Accounting Manmohan Goyal.
3) Principles of Management Accounting S.N.Maheshwari
4) Management Accounting Principles & Pr actices M.A.Saraf.
5) Management Accounting Hingorani.
6) Management Accounting I.M.Pandey.
7) Management Accounting M.Y.Khan , K.P.Jain.
8) Management Accounting M.G. Patkar.


munotes.in

Page 85

85 6
SOURCES OF FINANCE
Unit Structure
6.0 Objective
6.1. Introduction
6.2 Needs and Sources of finance
6.3 Sources of finance based on period
6.4 Long term sources of finance
6.5 Short term sources of finance
6.6 Summary
6.7 Questions
6.8 References
6.0 OBJECTIVES
After learning this topic, students will be able to understand
 Meaning of finance for the business and its importance
 Different types of sources finances of the business
 Features of different types of sources of finance
6.1 INTRODUCTION
Finance play an important role in business right before it starts till its day
to day operations. It is a very crucial decision to manage finance for the
business and hence effective evaluation is necessary to arrange finance for
the same. Normally the factor s to be considered while choosing a source
of finance are cost of finance, term of finance, leverage of the company,
risk factors involved, future plans of the business etc.
6.2 NEEDS AND SOURCES OF FINANCE
Finance is required by the business for differen t purposes. The need of
finance by the business may be for: -
 Growth
 Development
 Diversification
 Investment either initial or to acquire new assets
 daily operations i.e., working capital
munotes.in

Page 86


Financial Management
86 The business opts different types of finance which can be classi fied as
follows:
.1 On the basis of source of generations: -
a. Internal sources like retained earnings (reserves), depreciation fund
etc.
b. External sources like debentures, bonds, loans etc.
2. On the basis of Ownership: -
a. Owners/ Owned Capital i.e. Share capit al and retained earnings
b. Borrowed Capital i.e. loans, debentures, public deposits
3. On the basis of Period: -
a. Long term sources
b. Medium Term sources
c. Short term sources
After learning the different types of sources of finances, let us study in
detail the sou rces of finance based on period.
6.3 SOURCES OF FINANCE BASED ON PERIOD
As seen above, the different types of finance based on period may be long
term, medium term or short term. Let us understand the concept of these
sources of finance of the business.
 Long term finance: - As the name indicates, this finance is for the
long term period and usually utilised to procure fixed assets like plant and
machinery, land and building, furniture etc. The investment is huge in
nature and usually for 5 to 10 years. T he cost of investment on such
sources of finance may be high if it is borrowed from external source.
Such long term finance is also known as permanent working capital

 Medium term finance: - Such finance is arranged to meet the
expenditure which are deferre d revenue in nature. The finance is usually
for a period less than 5 years and the benefit of such finance may be
written off over a period of time. Huge advertisement expenses made by
the company, share issue expenses etc are the examples of such deferre d
revenue expenditures.

 Short term finance: - Such finance, as the name indicates is
arranged to meet the day to day expenses against the current assets like
stock, debtors, cash etc. It is also known as working capital which is
normally raised for a peri od of one year only.
After understanding the different types of finances based on period, let us
discuss the above in detail to gain thorough knowledge of different types
of finance.
munotes.in

Page 87


Sources of finance
87 6.4 LONG TERM SOURCES OF FINANCE
Long term sources of finance can be ar ranged in the following: -
i. Equity capital / owners capital
ii. Preference capital
iii. Retained earnings
iv. Debentures
v. Bonds
vi. Loans from banks and financial institutions
vii. Bridge finance ( short term loans arranged till the disbursement of
long term loan from banks and financial institutions)
Let us discuss the above in detail
I Equity capital / Owners capital: -
The capital raised by issue of equity shares is called equity capital and
holders of such shares are called equity shareholders. Equity shares have
no fixed div idend as a return on their investment and also they bear the
risk of investment. Hence they are the real owners of the company. They
are involved in every decision of the business and they do vote in the
annual general meetings of the business. As they ar e the real risk bearers
(as the owner of business bears) of the company their capital is also called
owners capital. The equity shares do not have any types. Let us understand
the features of equity shares to develop the concept more thoroughly
a. Permanent capital: - Equity share capital is permanent in nature as
it is irredeemable. The amount raised by issue of equity shares remains
with the company for its life time. Such capital is refundable only at the
time of winding up of the company or at the time of buyback.

b. Fluctuating return on investment: - The return on equity share
capital is not fixed. The rate of dividend on equity share capital depends
upon the profitability and the development plans of the company. At
times, even if the company earns huge p rofit no dividend is paid if the
company opts for development projects for the growth of the business.

c. Rights: - The equity share holders have special rights as they are the
owners of the business. The equity share holders have the rights like, right
to vote, right to participate in profit, right to check the books of the
accounts, right to transfer the shares.

d. Controlling power: - Being the owners of the company, equity
share holders enjoy the power to control the business decisions. Decisions
regarding the management of the company are taken on the basis of the
votes of equity share holders.
e. Residual Claimant: - Equity share holders do not have preference
to claim their investment in case of closure of the business. At the time of
winding up of the comp any, the equity shareholders have the last
preference of payment i.e. firstly all the outsiders secured and unsecured munotes.in

Page 88


Financial Management
88 liabilities are paid, then the outstanding expenses, then other liabilities if
any. Hence equity share holders are also called last claima nt of the
earnings of the company.

f. Bonus and Right Issue: - Equity shares holders enjoy the right to
get bonus shares as well as right issues of the company. Bonus shares are
issued free of cost to the existing shareholders and right issue is made to
the existing shareholders at the rate lower than market rate. Both the above
are issued in the proportion of the shares held by the equity shareholder.

g. Values: - Equity shares have different values viz., face value or
nominal value or registered value and mark et value. The face value
remains constant but the market value changes as per the market
conditions. Market fluctuations results in increase of decrease in the profit
of the company which further results in increase or decrease in the price of
equity share s in the market respectively.
II Preference Capital: -
The capital raised by issue of preference shares is called preference share
capital. As the name indicates, preference shares holders have the
preference of getting dividend as returns on their inve stment and also their
initial investment at the end of the agreed term end or at the time of
winding up of the company. The status of preference shares holders is that
of the creditors of the company and hence they can’t take part in the
management of the company. There are different types of preference
shares which are stated below with its brief explanation.
1. Redeemable Preference shares : - These shares are redeemed at the
maturity and hence they are called redeemable preference shares.

2. Irredeemable Pref erence Shares : - As the name indicates, such
preference shares are not redeemed till the winding up of the company.
However, Section 55(1) of Indian Companies act 2013 does not permit
any company to issue irredeemable preference shares.

3. Cumulative prefere nce shares : - The amount of dividend payable to
these shareholders is accumulated over a period of time if the company
is unable to pay dividend in any year due to loss. Such unpaid
dividend is to be paid in future when the company earns profits.

4. Non-cumulative preference shares: - These types of shares are just
opposite to cumulative preference shares. The shareholders of such
preference shares will not get dividend for a particular year in which
the company makes losses.
5. Convertible Preference shares: - As the name indicates, these
preference shares can be converted into equity shares after completion
of certain time.
munotes.in

Page 89


Sources of finance
89 6. Non-Convertible Preference shares: - Such Preference shares cannot
be converted into equity shares even if the holders of such preferen ce
shares wish to convert them.

7. Participating Preference Shares: - Sometimes when the company
earns huge profit, the holders of participating preference shares, get
additional share in the profits apart from the fixed dividend.

8. Non-Participating prefere nce share : -Such preference shares do not
have right to enjoy additional dividend even if the company earns huge
profits.
Let us understand the features of preference shares
a. Nature of Capital: - Preference capital is not permanent capital of
the compan y. It is to be redeemed at the end of pre -determined term fixed
at the time of issue of preference shares. Hence a company cannot issue
irredeemable shares. Usually, preference shares are issued by the well -
established company in the later stage of its bu siness life.

b. Fixed return: - The rate of dividend on preference shares is fixed. It
does not change and hence preference shareholders cannot claim extra
income in case if the company earns huge profits.

c. Preference of repayment : - The amount raised by is sue of
preference share by the company is to be paid back compulsorily at the
end of the term. Similarly, the payment of dividend is also to be made
even if the company suffers losses.

d. No voting rights: - Preference shareholders are mere Creditors of
the company. As they are paid back the amount invested along with the
dividend at the predetermined rate, they don’t enjoy voting rights similar
to equity shareholders. Preference shareholders cannot interfere in the
management of the company.

e. Risk: - Investment in preference shares is risk free. Usually the
investor who is cautious about getting returns on the investment opts for
purchasing preference shares of the company. There is no risk for the
investors as the amount of investment and the amount of d ividend is
received back at the term end.
III. Retained Earnings: -
A good business always follow a habit of keeping some part of profit aside
annually rather than giving out as a dividend to the owners. Such
accumulated profit of the company over a perio d of years is called retained
earnings of the company. Retained earnings are normally used for re -
investment in the business for its expansion and development. Retained
earnings of the business depend upon the factors like total earnings of the
company, t axation policy, dividend policy, government control etc. munotes.in

Page 90


Financial Management
90 IV Debenture : -
Debentures are normally issued to raise long and medium term finance of
the company. Debenture is a borrowed capital which also includes bonds
and other instruments showing evidence o f debt of the company which
may or may not have charge on the assets of the business. Debentures
have a fixed rate of interest and the amount of investment is payable at the
end of the term agreed at the time of issue. Different types of debentures
are iss ued by the business / company depending upon its financial position
and need of finance for its operations. Debenture holders are the creditors
of the company.
Features of Debentures: -
a. Promise: - It is a promise by the company to pay the holder the sum
specified at the time of maturity of the term.
b. Face Value: - Debentures are issued normally in the face value of 100
or its multiple.
c. Time of Repayment: - The date of repayment is fixed at the time of
issue by the company. The company has to compulsorily rep ay the
amount of debentures.
d. Priority of Repayment: - Debentures are to be paid before paying the
amount of shareholders. The amount of debentures is paid along with
others secured liabilities like bank loan.
e. Interest: - Debenture holders are paid fixed inte rest on their
investment.
f. Issuing authority: - The board of directors of the company have the
authority to issue the Debentures with the consent of shareholders as
per the sec 179(3) of Indian Companies Act 2013.
g. Voting Rights: - Debenture holders have No Vo ting Rights. They are
just the creditors of the company which are secured in nature.
V. Bonds
Bond is a formal and legal contract between the investor and the company
taking finance. The amount borrowed by the company through bonds is
repayable in cash at the end of the term with a fixed amount of interest
too. Normally bonds are made for a period of 5 to 50 years.
Features of Bonds: -
a. Nature: - The amount issued through bonds is a debt finance and has
to be repaid on maturity of term.
b. Status : - The bond holders are mere creditors of the company where
they have invested their funds.
c. Return on investment: - The return on investment in bonds is called
interest which is similar to any other borrowed fund of the business.
The rate of interest on bonds is fixed and such interest can be paid
regularly or at the time of maturity along the principal amount
invested.
d. Repayment: - Being a borrowed fund, the amount raised through
bonds is to be repaid compulsorily by the company on the maturity. munotes.in

Page 91


Sources of finance
91 VI. Loans from banks a nd financial institutions: -
Long term finance can also be obtained by the company from banks and
financial institutions in the form of loans. These loans may be taken for
investment purpose or for the operations of the business. The rate of
interest in fi xed on such borrowings and is to be repaid in the agreed term
of period. Such loans can also be taken from financial institutions which
have developed in our country since 1948 as the result of industrial
development. Financial institutions may be of diff erent types such as
Development banks, Financial institutions, Investment institutions or state
level institutions.
6.5 SHORT TERM SOURCE OF FINANCE: -
Short terms finance plays an equally important role as of long term finance
in the business management. Short term finance can be raised through
i. Trade Credit
ii. Accrued expenses and deferred incomes
iii. Advances from customers
iv. Commercial papers
v. Bank advances
vi. Public deposits
Let us understand the above finances in details
i. Trade Credit : - Credit plays an importan t role in running any
business. The manufacturer, wholesaler, retailer are called the creditors of
the business as they provide goods tangible or intangible to the business.
Majority business runs on credit from the above said creditor which helps
a busine ss to manage its operations smoothly. The goods purchased on
credit creates a burden of payment on one hand and also creates an
assurity of income to the creditors. Trade credit is a non -cash loan of the
business.

ii. Accrued expenses and deferred incomes: - Accrued expenses are
the expenses which are payable in future. It clearly means that such
expenses are payable for the goods or services taken by the business at
present and the amount is payable in future normally within a year. Hence
it is treated as sho rt term finance and shown as current liability in the
books of accounts. On the other hand, deferred incomes are the incomes
receivable in the future for the service provided at present. Such amount
is receivable in future and is treated as short term fi nance which is shown
on asset side of the balance sheet.

iii. Advances from customers: - The amount received as advance from
the customers can be treated as short term source of finance as the
business is bonded to do a certain pre decided task for the amoun t received
from the customer. It gives short term loan on one hand for the operations
of business and on the other hand it has to be treated as a liability till the munotes.in

Page 92


Financial Management
92 date of fulfilling the required service from the business receiving such
advances.

iv. Commer cial papers: - Commercial papers are the sources of short
term finance of the business which is unsecured in nature bearing a low
rate of interest as compared to banks interest. Commercial papers are
issued by huge corporates in multiple of lakhs and bank h as no role to play
in commercial papers. The different types of commercial papers are
promissory notes, drafts, cheques, etc.

v. Bank advances: - Banks do provide short term finances in the form
of overdraft, cash credit, cash loans, discounting of bills, et c. Short term
finance from banks are useful for the business entities to manage their
working capital for smooth operations. Bank advances like overdraft and
cash credits can be secured and discounting of bill may be unsecured in
nature.

vi. Public deposits: - Similar to the function of bank, some companies
accept deposits from the general public. Such Public Deposit is normally
accepted for the period of minimum 6 months and maximum 36 months
which bears a fixed rate on interest. Deposit holders get a “Depo sit
Receipt” as a proof of the investment done. Public deposit may be secured
or unsecured in nature.
6.6 SUMMARY
 Finance is the blood of the business
 Finance can be raised from different sources and can be classified as
Owned fund or Borrowed funds
 Owned funds remain with the business for its lifetime and include
share capital and reserves or retained earnings.
 Borrowed funds are the debts of tee business and are to be repaid on
the maturity.
 On the basis of time the business finance can also be classifie d as
Long term, Medium term and short term depending upon the period of
the issue or raising of finance.











munotes.in

Page 93


Sources of finance
93 6.7 EXERCISE
A. Select the appropriate option and complete the statement given: -
1. _____________ refers to capital made up of equity and preference
shares.
a. Share
capital b. Debt capital c. Reserves d. All the
above

2. The maximum term of accepting public deposit is ________ months.
a. 12 b. 24 c. 36 d. 60

3. Public deposit is a _____________ borrowed fund
a. secured b. unsecured c. both the
above d. none of the above

4. _________ are the creditors of the company.
a. Equity
shareholders b. Preference
share holders c. Debenture
holders d. Any of the
above

5. ________ is the smallest unit of the share capital of the company.
a. Share b. Debenture c. bond d. public
deposit

6. The share given out to the existing shareholders free of cost are
__________
a. Right issues b. Bonus issue c. Initial
issue d. Subsequent
issue

7. The preference shares which can be converted into equity shares are
called _________ preference shares
a. Redeemable b. irredeemable c. converti ble d. non-
convertible

8. The return on borrowed capital is _________
a. profit b. dividend c. interest d. bonus

9. Retained earnings is the __________ source of finance
a. additional b. subsequent c. internal d. external

10. ____________ has the right to check the books of the co mpany
a. Debenture
holder b. Preference
shareholder c. Equity
shareholder d. All the above


B Distinguish between
1. Share and Debenture
2. Equity shares and Preference Shares
3. Owned fund and Borrowed fund. munotes.in

Page 94


Financial Management
94 C. Answer the following: -
1. what is an equity shar e? State its features.
2. State different types of preference shares.
3. What do you mean by borrowed fund? Explain different types of
borrowed funds.
4. State different types of sources of finance based on time/ period.
6.8 REFERENCES:
1) Management Accountancy J.Betty.
2) Management Accounting Manmohan Goyal.
3) Principles of Management Accounting S.N.Maheshwari
4) Management Accounting Principles & Practices M.A.Saraf.
5) Management Accounting Hingorani.
6) Management Accounting I.M.Pandey.
7) Management Accounting M.Y.Khan , K.P.Jain.
8) Management Accounting M.G. Patkar.


munotes.in

Page 95

95 7
THEORY OF CAPITAL STRUCTURE
Unit Structure :
7.0 Objectives
7.1 Introduction
7.2 Sources of Capital employed
7.3 Pattern of Capital Structure:
7.4 Earnings of Capital Structures:
7.5 Theories of Capital Structure:
7.6 Summary
7.7 Questions
7.8 References
7.0 OBJECTIVES:
 To understand the concept of Capital Structure
 To explore different Capital Structure Theories
 To Examine pattern of Capital Structure
 To study Net Income Approach
 To overview Operating Net Income Approach
 To demonstrate Traditional Theory
7.1 INTRODUCTION:
Capital structu re theory attempts to explain the financial mix used to
finance corporation and firms. Capital structure decisions are essential for
business survival and prosperity. Wrong decision may lead to financial
distress, bankruptcy, or even liquidation. The rec ommended textbook
strategy that management should follow to lower the weighted average
cost of capital (WACC), will lead to increased profitability and returns,
which will eventually translate in increased shareholders wealth.
Financial decisions are crit ical in maximizing the firm’s value. Therefore,
to achieve the “best”, capital structure is critically important task of
management decision.

munotes.in

Page 96


Financial Management
96 7.2 SOURCES OF CAPITAL EMPLOYED:
Following are components of Capital Structure
A] Own fund or Shareholders fund or Net Worth
B] Loan fund or Borrowed Fund or Owed fund
A. Own Fund:
1. Equity Share Capital
2. Preference Share Capital
3. Reserves and surplus/ ploughing back profit/ Retained earning
1. Equity Share Capital: Equity Share Capital is also known as “Risk”
capital. Risk capital means which has chances of loss to the extent of
amount invested in the business. Possessors of equity shares are called as
‘Equity share Holders’ they are ‘deemed owners’ of the firm. Equity share
Holders have complete control on the assets of organization but without
complete rights on cash flow that is generated by total assets. The debt
holders have the first right on cash flow in the form of interest
(Bhattacharya 2012).
2. Preference Share Capital: Prefere nce Share Capital is also known as
“Quasi Risk” capital. They are committed with the fixed rate of dividend
only in case of profit of the concern. Preference shares are ‘Quasi Debt’ in
case of profit, due to fixed rate of dividend. Whereas it is ‘Quasi Equ ity’,
when there is no profit, due to no obligation to pay dividend in case of
loss. Preference shareholders claims are settle before Equity shareholders
claim but after debt claim.
3. Reserves and surplus: Reserves and surplus are the undistributed
profits of the business, which are transferred to various Reserves for
specific or unspecific reasons. This is also called as Retained Earnings
which are ploughed back to the capital employed. Reserves and Surplus
belong to Equity shareholders only.
B. Loan Fund:
1. Secured Loan
2. Unsecured Loan
1. Secured Loan: Amount of borrowing against the collateral security is
called as ‘secured loan’. In case of collateral security the Debt holders has
specific claim on Assets of the concern. Mortgage, hypothecate, charge
pledge or lien on asset is covered for Secured Loan. Interest is obligatory
even in case of loss. Debentures or Bonds issued by company are few
examples of secured loans.
2. Unsecured Loan: Amount of borrowing without the collateral security
is called as ‘ unsecured loan’. In case of unsecured loan the Debt holders munotes.in

Page 97


Theory of Capital Structure

97 has no specific claim on Assets of the concern. Interest is obligatory even
in case of loss. Claims will be settled before share capital, in case of
liquidation of firm. Public deposits are unsecu red loans.
Table 7.1 COMPPARISION OF ELEMENTS OF CAPITAL
STRUCTURE
Types Of Funds Cost of
Capital Dilution
of
control Risk Restriction on
managerial
autonomy
Equity capital High Yes Nil No
Preference Capital High No Negligible No
Retained Earning High No Nil No
Debentures Low No High Some
Term Loans Low No High Moderate

7.3 PATTERN OF CAPITAL STRUCTURE:
Corporate firm does have to plan a suitable blend of different securities in
total capitalization in such a way so as to abate the cost of capital a nd
amplify the earnings per share to the equity shareholders. There can be
four basic patterns of capital structure as follows:
(i) Equity capital only (including Reserves and Surplus)
(ii) Equity and preference capital
(iii) Equity, preference and long t erm debt i.e. debentures, bonds and loans
from financial institutions etc.
(iv) Equity and long term debt.
Some authors use capital structure and financial structure interchangeably.
But, both are different concepts. Financial structure refers to the way i n
which the total assets of a firm are financed. In other words, financial
structure refers to the entire liabilities side of the Balance Sheet. But,
capital structure represents only long term sources of funds and excludes
all short term debt and current liabilities. Thus, financial structure is a
broader one and capital structure is only a part of it.



munotes.in

Page 98


Financial Management
98 7.4 EARNINGS OF CAPITAL STRUCTURES:
7.4.1 Concept of Scarce Returns on Capital: Over Capitalization:
According to Gerstenberg, “A company is over-capitalized when its
earnings are not large enough to yield a fair return on the amount of stock
and bonds that have been issued, or when the amount of securities
outstanding exceeds the current value of the assets” .
Over -capitalization means no proper use of investe d funds. Over -
capitalization arises when the earnings of the company are not adequate to
give a normal rate of return on capital employed. If, the value of real
assets is less than the sum total of paid up value of shares and debentures.
Over -capitalizatio n means there is existence of fictitious and less earning
as compared to normal rate earn by other companies from the same
industry.
Causes of Scarce Returns on Capital of a company are Promotion,
formation or development during inflation. This is beca use in inflation,
prices are high and value is less, it will be a cause of over -capitalization.
Obtaining loan funds at high rate of interest results in over -capitalization.
Excess -issue of capital. If promoters buy assets of lower values at higher
prices, and provide inadequate depreciation, it will result in over -
capitalization. High rates of taxation may leave little in the hands of the
management to deliver for depreciation, replacements and dividends. If a
company’s products record a continuous decline , it will bring down the
earnings of the company and as a result, normal rate of returns on capital
employed will be reduced which represents over -capitalization.
It wills also reduction in face value of shares and loss to its owners, due to
writing off t he accumulated losses.Over -Capitalization affects not only the
company and its owners but also the society as a whole. Over -capitalized
organization goes to increase prices and decreased in quality of its
products. It’s been affecting the entire industry a nd the society, and
ultimately leads to recession of economy.
Management must take remedial actions to resolve the situation as soon as
the over -capitalization is experienced by the firm to avert losses. Various
remedial measures such as reduction of high interest bearing funded debt,
increasing the efficiency of management, redemption of preference shares,
reduction in face value and number of shares, etc., have been suggested.
7.4.2 Concept of Prolific Returns on Capital: Under Capitalization:
Bonneville and D ewey observe that when a corporation is earning an
extraordinary large return on its outstanding stock it is said to be
undercapitalized. Under -capitalization is the contrary to over -
capitalization. A company is said to be under -capitalized when it is
earning remarkably higher profits as evaluated to other companies or the
value of its assets is substantially higher than the capital raised. For
instance, the average rate of return of the industry is 12%. But the munotes.in

Page 99


Theory of Capital Structure

99 company is earning 25% on the capital employe d; it is a case of under -
capitalization.
Following are the indicators of Prolific Returns on Capital :
(a) There is a surprising increase in earnings of the company.
(b) Future earnings of the company were under - approximated at the time
of endorsement.
(c) Assets might have been acquired at very low cost.
A company may have huge past accumulated secret reserves, and follows
a cautious dividend policy. This would increase the earning capacity of the
company. High profitability of the company may be very hig h. This will
result in the higher rate of earnings per share. With the high value of its
equity share in the market will be high. The financial reputation of the
company will also increase in the market. The shareholders can anticipate
higher dividends rec urrently.
Under -capitalization may be remedied by increasing the number of equity
shares by fresh issue. This will dilute the earnings per share. Dilute the
earnings per share by issuing bonus shares to the equity shareholders. If
under -capitalization is d ue to insufficiency of capital, issue more shares
and debentures to the public,
7.5 THEORIES OF CAPITAL STRUCTURE:
The existence of an optimum capital structure is not accepted by all. There
are two extreme considerations regarding the existence of an op timum
capital structure.
Different theories of capital structure have been developed. The main
contributors to the theories are David Durand, Ezra Solomon, Modigliani
and Miller.
The important theories of capital structure are:
 Net Income Approach
 Net Oper ating Income Approach
 The Traditional view
 Modigliani and Miller hypothesis
Assumptions Underlying the Theories:
With the following assumptions, there is a clear understanding of these
theories and the relationship between capital structure and value of th e
firm or cost of capital,
(i) There are only two sources to employ by firms, i. e. debt and equity.
(ii) The total assets of the firm are given. munotes.in

Page 100


Financial Management
100 (iii) The firm‘s total capital employed remains constant. The degree of
leverage can be altered by vending deb t to repurchase shares or
vending shares to redeem debt.
(iv) The firm has 100% payout ratio, i.e., it pays 100% dividends out of its
earnings.
(v) The operating earnings (EBIT) of the firm are not likely to grow.
(vi) The business risk is assumed to be c onstant and not dependent of
capital structure and financial risk.
(vii) Investors have the same subjective probability distribution of
expected future operating earnings for a given firm.
(viii) There is absence of corporate and personal taxes. This assum ption it
relaxed later.
In analyzing the capital structure theories the following basic definitions
are used:
S = Market value of common shares
D = Market value of debt
V = S + D = Market value of the firm
NOI = X = Expected net operating income, i.e.,
Earnings before interest and taxes (EBIT)
NI = NOI - Interest = Net Income or shareholders earning.
Ko

Ko
Kd [
] + K e [
]
Kd

Ke

Kd = Cost of Debt [cheap source of finance],
Ke = Cost of Equity [costly source of finance] and
Ko = Overall Cost of Capital / Weighted average cost of Capital (WACC)
7.5.1. Net Income Approach
This approach was identified by David Durand. According to his
approach, capital structure has consequence. A f irm can upsurge the value
of the firm and curtail the overall cost of capital by hiring debt capital in munotes.in

Page 101


Theory of Capital Structure

101 its capital structure. As per this theory, greater the debt capital hired, lesser
shall be the overall cost of capital and greater the value of the firm .
This theory is subject to the following assumptions:
 The cost of debt (K d) is always cheaper source of capital than cost of
equity (K e).
 The risk assessment of investors is not pretentious by the use of
debt. As a result, the cost of equity (K e) and the cost of debt (K d) don‘t
change with leverage.

 There is no existence of corporate taxes.
According to the above assumptions, cost of debt is cheaper than cost of
equity and they remain constant irrespective of the degree of leverage. If
maximum debt capita l is hired because of its relative inexpensiveness, the
overall cost of capital decreases and the value of the firm upsurges.
According to this approach:
V = S + D
Where,
V= Total value of firm
S= Market value of equity shares
D= Market value of debt.
And, overall cost of capital or weighted average cost of capital can be
calculated as:
Overall cost of Capital (K o)









Figure 7.1: Leverage and Cost of Capital (NI Approach)
(Source: Chandra, 2011) Kd Ke
Ko Cost
of
Capit
al ()

Degree of Leverage 0 munotes.in

Page 102


Financial Management
102 Above diagram 7.1 is evidence that when there is no debt fund means
degree of leverage is null, only equity capital is employed then overall
cost of capital is equal to cost of equity (K o = K e ). Debt identical fund is
relatively inexpensive as compared to cost of equity, if debt capital is
hired more and more, the overall cost of capital declines, and it becomes
identical to cost of debt (K d ) when the firm is fully financed by debt.
Thus, corresponding to this theory, the firm‘s capital structure will be
optimum, when degree of leverage is one.
7.5.2. Net Operating Income Approach
This net operating income (NOI) approach is also suggested by David
Durand. This represents another diverge opinion that capital structure and
value of the firm are independent means irrelevant. NOI methodology
does not influence cost of capital and value of the firm. The value of the
firm (V) is determined as follows:
V = S + D

Weighted Average Cost of Capital (WACC) means K o depends on the
business risk of the firm. It is not influenced by financing mix i.e.
Leverage. Value of firm (V) will not be alter with the alteration in capital
structure.
Rendering to this theory, higher degree of leverage creates risky situation
for equity investors, hence they pursue higher reimbursement. The use of
less expensive debt fund is offset by an increase equity capitalization rate.
Thus, the overall Capitalization rate (K o) remains unaffected and
subsequently the value of the firm does not change. There is no existence
of an optimum capital structure. As per NOI Approach any capital
structure is optimum. Following are the critical assumptions of this theory:
 The market capitalizes the value of the firm as a whole. Thus, the
split between debt and equity is not important.
 Weighted Average Cost of Capital (K o) depends on the business risk
of the firm. If the business risk remains constant at every level of debt -
equity mix, then K o is constant.
 There is no existence of corporate taxes.
 There is no alteration in Cost of Debt (K d).






Degree of Leverage (D/E)
Figure 7.2: Leverage and Cost of Capital (NOI Approach)
(Source: Chandra, 2011) 0 Ke
Kd Ko Cost
of
Capita
l () munotes.in

Page 103


Theory of Capital Structure

103 NOI Approach:
The above 7.2 diagram sho ws that K o and K d are constant and K e increases
with leverage continuously. The increase in cost of equity (K e) exactly
counterbalances the benefit of cheap cost of debt, so that overall cost of
capital (K o) remains constant, at every degree of leverage. I t denotes that
every capital structure is optimum and there is no exceptional optimum
capital structure.
7.5.3. THE TRADITIONAL APPROACH
This approach, which is also known as midway approach, has been
universalized by Ezra Solomon. It is a transitional bet ween the two
extreme Approaches of Net Income Approach and Net Operating Income
Approach. Rendering to this approach, cost of capital (K o) can be
decreased or the value of the firm can be increased with a prudent mix of
debt and equity. This theory says th at WACC falls with upsurge in debt
capital up to a judicious level and later it upturns with a further escalation
in debt capital.
Following are assumptions of Traditional Approach:
 The K d remains unchanged up to reasonable degree of leverage but
upsurge thereafter at an increasing rate.
 The K e, also remains unaltered or rises only progressively up to
reasonable degree of leverage and rises suddenly thereafter.
 Behaviour of K d and K e varies the K o as initially it decreases up to a
certain point, remain mor e or less constant or moderate and upsurge
beyond a certain point.
The way in which the WACC (K o) reacts to changes in capital structure
can be alienated into three stages under traditional position.
Stage I: Decreasing WACC and Increasing Value of Firm:
In the stage I, the cost of equity (K e) and the cost of debt (K d) are constant
and cost of debt is less than cost of equity. The employment of debt capital
upto a reasonable level will cause the overall cost of capital to decline and
increasing value of fir m due to the cheap cost of debt.
Stage II: Optimum Capital Structure:
As and when the firm has reached a judicious level of leverage, additional
escalation in debt will have no effect on the value of the firm (V) and the
WACC (K o). The further surge in deb t capital increases the risk to equity
shareholders which leads to a rise in equity Capitalization rate (K e). This
rise in cost of equity exactly counterbalances the cheap cost benefit of debt
capital so that the WACC (K o) remains constant.
Stage III: Incr easing WACC and Decreasing Value of Firm:
If the firm increases debt capital extend beyond reasonable level, it will
cause intensification in risk to both equity shareholders and debt – holders, munotes.in

Page 104


Financial Management
104 because of which both K d and K e start escalating in this stag e. This will
result in an increase in the WACC (K o) and ultimately decreasing value of
firm.
If the overall effect of all the three stages is taken, it is evident that cost of
capital declines and the value of the firm increase with a rise in debt
capital upto a certain reasonable level.
If debt capital is further increased beyond this level, the overall cost of
capital (K o) tends to rise and as a result the value of the firm will decline.









Figure 7.3: Leverage and Cost of Capital (Tra ditional Approach)
(Source: Chandra, 2011)
Traditional View
It is evident from above graph that the overall cost of capital declines with
an increase in leverage upto Stage I and it increases with rise in the
leverage after Stage III. Hence, the optimum c apital structure lies in
Stage II.
7.5.4. Modigliani – Miller (MM) Hypothesis
The Modigliani – Miller hypothesis is matching with the Net Operating
Income Approach.
Modigliani and Miller (1958) proposed the capital structure irrelevance
theory, which states that under the assumption of a perfect capital market,
the choice of bonds or stocks makes no difference to firm value; in other
words, capital structure has no influence on firm value. A perfect capital
market does not have corporate tax or transaction costs, and when
information asymmetry is not a concern, a firm’s value is determined by
its ability to create value, no matter whether the capital it uses is from
internal or external sources. Ke
Ko
Kd Co
st
of
Capit Degree of Leverage 0 Stage I Stage II
Stage III munotes.in

Page 105


Theory of Capital Structure

105 Modigliani and Miller explored that, if there a re no taxes the cost of
capital and the value of the firm are not affected by the alterations in
capital structure. In other words, capital structure decisions are irrelevant
and value of the firm is autonomous of debt – equity combination.







0 Degree of Leverage
Figure 7.4: Leverage and Cost of Capital (M -M Approach)
Assumptions of the MM Approach:
 There is a perfect capital market, where investors are unrestricted to
buy and sell s ecurities, they can borrow funds free at the same terms as
the firms do, they behave rationally, they are cognizant, and there are
no transaction costs.
 Firms can be classified into identical risk classes. All the firms in the
same risk class will have the same degree of financial risk.
 All investors have the same anticipation of a firm‘s net operating
income (EBIT).
 There are no retained earnings, as the dividend payout ratio is 100%.
 There is no existence of corporate taxes. This assumption has been
detached later.
Basic Propositions: M -M Hypothesis can be explained in terms of two
propositions of Modigliani and Miller. They are:
(i) The WACC (K o) and the value of the firm (V) are autonomous of debt
– equity combination. The total market value of the fir m is given by
capitalizing the expected net operating income by the rate appropriate for
that risk class.
(ii) The financial risk increases with more debt gratified in the debt –
equity combination. As a result cost of equity (K e) increases in a manner
to counterbalance exactly the cheap cost benefit of debt.
Hence, overall cost of capital remains the same.
Proposition I
According to M – M Approach, for the firms in the same risk class, the
total market value is independent of the debt – equity combination and is Ke
Ko
Kd Co
st
of
Capitmunotes.in

Page 106


Financial Management
106 determined by capitalizing net operating income by the rate appropriate to
that risk class. Proposition I can be expressed as follows:
V=

Where, V = The market value of the firm
S = The market value of equity
D = The market value of deb t
EBIT = Earnings before Interest and Tax
Ko = Weighted Average Cost of Capital
According the Proposition I the average cost of capital is not affected by
degree of leverage and is determined as follows:







Degree of Leverage (D/E)
Figure 7.5: Leverage and Cost of Capital (M -M Proposition
I)
Arbitrage Process
According to M –M’s Proposition I, two firms alike in all respects except
their debt – equity combination cannot have d issimilar market values or
diverse cost of capital. In case, these firms have different market values,
the arbitrage will take place and equilibrium in market values is restored in
no time. Arbitrage process refers to switching of investment from one firm
to another. Arbitrage will take place to enable investors to engage in
personal leverage as against the corporate leverage to maintain
equilibrium in the market; hence the capital structure decision is
irrelevant.
Proposition II
Proposition II, defines the cost of Equity K e =

Where, 0 Ko Cost of
Capital
()
Ke
munotes.in

Page 107


Theory of Capital Structure

107 S = The market value of equity Shares
D = The market value of debt
EBIT = Earnings before Interest and Tax
Ke = Cost of Equity
Ko = Cost of Debt
According the Proposition II, for any firm in a given risk class the Cost of
Equity (K e) is identical to the constant WACC (K o), plus a premium for
the financial risk, which is equivalent to Debt -Equity ratio times spread
between K o and K d. Hence, Leverage will result in more earning per share.
The benefit of leverage is e xactly compensate by the boosted cost of
equity so firm’s market value will be irrelevant.
7.6 SUMMARY
Different theories of capital structure have been developed. The main
contributors to the theories are David Durand, Ezra Solomon, Modigliani
and Miller .
The important theories of capital structure are:
 Net Income Approach
 Net Operating Income Approach
 The Traditional view
Modigliani and Miller hypothesis
7.8 QUESTIONS
Q.01. Explain Theory of Net Income Approach?
Q.02 Evaluate Theory of Operating Net Inc ome Approach?
Q.03 Examine Traditional Theory Approach?
Q.04.Write Short note on Over Capitalisation?
Q.05 Write Short note on Under Capitalisation?
7.9 REFERENCES:
1. Carles W. Gesternberg, “Financial Organisation and Management of
Business”, Asia Publishin g House, IV ed., p.72
2. Prasanna Chandra, “Financial Management - Theory & Practice”, Mc
Graw Hill Education (India) Private Limited, New Delhi, 2015, 9th
Edition, p.465
munotes.in

Page 108


Financial Management
108 3. Durand David, “Costs of debt and equity funds for business: trend and
problems of measur ement”, Reprinted in the management of corporate
capital, Ezra Solomon, the free press, 1959, p.91 -116

4. Modigliani and Miller. (1958). The cost of capital, Corporation
Finance and the Theory of Investment. The American economic
Review, XLVIII(3).

5. Myers, S.C. (1984). The capital structure puzzle, Journal of Finance,
39, pp. 575 -592.

6. Akerlof, G. “The Market for Lemons.” Quarterly Journal of
Economics 84 (Aug., 1970): 488 -500.

7. Spence M. “Job Market Signaling.” The Quarterly Journal of
Economics 87(3) (1973 ) 355 -374.

8. Ross, S.A. “the Determination of Financial Structure: the Incentive -
Signaling Approach.” the Bell Journal of Economics 8 (1977): 23 -40.



munotes.in

Page 109

109 8
CAPITAL STRUCTURE: EBIT, EPS MPS
APPROACH
Unit Structure
8.0 Objectives
8.1 Introduction
8.2 Definition
8.3 Characteristics of Capital Structure:
8.4 Factors Affecting Capital Structure
8.5 EBIT
8.6 EPS
8.7 MPS
8.8 Summary
8.9 Questions
8.10 References
8.0 OBJECTIVES:
 To understand the concept of E BIT
 To Study the concept of EPS
 To understand the concept of MPS
 To Examine the Factors Affecting Capital Structure
8.1 INTRODUCTION:
Capital structure includes proportion of debt and equity in the total capital
of a firm. In term “Capital Structure” capit al refers to long terms funds
and structure refers to the proportion of debt and equity in capital. One
should study financial leverages to check their effect on returns on equity.
Leverages may increase or decrease returns on equity in different
situati on. Capital structure planning helps to maximize the profit of the
owner by considering securities carrying less cost of capital. Capital
structure decision is of critical importance to the organizations, because it
affects firm’s value and stockholders we alth. There is a significant
relationship between firm value and its capital structure, which implies
that capital structure affects financial performance and shareholders’
value. munotes.in

Page 110


Financial Management
110 Capital structure is the blend of funds employed in the corporation. This
blend includes Equity share capital, Preference share capital, Retained
earnings and secured, unsecured borrowed funds. Equity share capital
holders are the deemed owners of an organization. Preference shares the
deemed debt which have fixed rate of dividend . Retained earnings are the
undistributed profit which can be ploughed back to the employed capital.
Borrowed funds or Loan funds or Owed funds are further classified in
Secured debts or and Unsecured debts.
8.2 DEFINITION:
According to Gerestenberg, ― “C apital structure of a company refers to
the composition or make -up of its capitalization and it includes all long
term capital resources, viz, loans, bonds, shares and reserves”. Thus
capital structure is made up of debt and equity securities and refers t o
permanent financing of a firm.
8.3 CHARACTERISTICS OF CAPITAL STRUCTURE:
A capital structure will be considered to be appropriate if it possesses
following Characteristics:
1) Conservatism: Conservatism means the debt substance in the total
capital struc ture does not exceed the proportionate limit which the
company can bear. It should be commensurate with the company‘s ability
to harvest future cash flows.
2) Solvency: The pattern of capital structure should be so formulated so as
to ensure the firm’s sol vency. If there is disproportionate rise in debts over
equity, it will lead to insolvency of the company. Debt constituent should
not be the cause that results in increased risk beyond acceptable limits.
3) Flexibility: Capital structure should be flexible to meet the
requirements of changing conditions. Moreover, it should also be possible
for the company to provide funds whenever needed to finance its
profitable activities.
4) Profitability: The capital structure of the company should be utmost
profitable . The most profitable capital structure is one that tends to
minimize weighted Average Cost of Capital (WACC) and maximize
earnings per equity share.
5) Control: The capital structure should be so strong that Owner of the
company should have complete contr ol and risk of losing control to be
minimised. Debt fund should not be able to dominate the company’s
policies.

munotes.in

Page 111


Capital Structure: EBIT, EPS
MPS Approach
111 8.4 FACTORS AFFECTING CAPITAL STRUCTURE
DECISION:
The capital structure of a firm depends on a number of factors and each
factor is of equ al importance. Generally, the following factors affecting
the capital structure decisions of a company.
 Leverage effect:
Capital gearing ratio helps in understanding leverage effect, the long term
debt and preference share capital are the funds bearing fi xed rate of
interest and dividend, if they are employed with equity share capital is
called financial leverage or trading on equity. Cost of preference capital is
less than cost of equity share capital, which leads high Earning Per Share
(EPS).The use of l ong term debt capital also increases the EPS, because of
its lower cost as compared to Equity and Preference. Debt is also
important for tax benefit.

The possible fluctuations in Earnings Before Interest and Taxes (EBIT)
and their impact on EPS can help in analyzing different financing plans.
Financial leverage is one of the important considerations in planning the
capital structure of a company, because of its effects on the earnings per
share, Financial leverage increases EPS, along with financial risk to
shareholders. Hence, the firm should employ debt in such way that
financial risk does not hamper the leverage effect.

 Cost of Capital:
Cost of capital is another important factor that should be kept in mind
while designing the capital structure of a firm. The capital structure should
be designed in such a way that the firm‘s overall cost of capital (k o) is the
minimum. Cost of capital is the minimum return expected by its suppliers.
Of all the sources of capital, equity capital is the costliest as the equity
shareholders bear the highest risk. Cost of Equity (k e) > Cost of Debts (k d)
On the other hand, debt capital is the cheapest source because the interest
is paid on it by the firm whether it makes profits or not. Moreover, interest
on debt capital i s tax deductible which makes it further cheaper. Cost of
Preference share capital (k p) is also cheaper than equity capital as the
dividends are paid at a fixed rate on preference shares. Retained earnings
(kr) do not require any flotation cost. So, the ov erall cost of capital
depends on the proportion in which the capital is mobilized from different
sources of finance. Hence, capital structure should be designed carefully
so that overall cost of capital is minimized.

 Dilution of Control:
Sometimes, the d esigning of capital structure of a firm is influenced by the
desire of the existing management to retain control over the firm.
Whenever supplementary funds are mandatory, the management of the
firm desires to raise the funds without any damage to control over the
firm. If equity shares are issued for raising funds, the control of the
existing shareholders is diluted. This is why; they may raise the funds by
allotting fixed charge bearing debt and preference share capital, as munotes.in

Page 112


Financial Management
112 preference shareholders and deb t holders do not have any voting right. The
Debt financing is advisable from the point of view of no dilution of
control.

But overdependence on debt capital may result in heavy burden of interest
and fixed charges and may lead to liquidation of the compan y in extreme
cases.

 Fluctuation of sales:
This is another important factor which influences the capital structure of a
firm. Consistency of sales guarantees constant earnings, so that the firm
will not face any exertion in meeting its fixed commitments o f interest
payment and repayment of debt. So the firm can raise a higher level of
debt. In the same way, the rate of growth in sales also affects the capital
structure decision. Usually, greater the rate of growth of sales, greater can
be the use of the de bt in the financing of a firm. On the other hand, the
firm should be very careful in employing debt capital if its sales are highly
fluctuating and declining.

 Utilisation of Source of Finance:
The purpose for which funds are raised should also be conside red while
determining the sources of capital structure. If funds are raised for prolific
purpose, loan fund is suitable as the interest can be paid out of profits
engendered from the investment. But, if it is for infertile purpose, equity
should be favored .

 Flexibility:
Flexibility means the firm‘s competence to acclimate its capital structure
to the needs of the changing circumstances. Capital structure should
flexible enough to raise additional funds whenever required, without much
delay and cost. The c apital structure of the firm must be designed in such a
way that it is possible to substitute one form of financing for another to
economies the use of funds. Preference shares and debentures offer the
highest flexibility in the capital structure, as they can be redeemed at the
discretion of the firm.

 Issue Expenses:
Flotation costs are not a very significant factor in the determination of
capital structure. These costs are incurred when the funds are raised
externally. They include cost of the issue of p rospectus, brokerage,
commissions, etc. Generally, the cost of flotation for debt is less than for
equity. So, there may be a temptation for debt capital. There will be no
floatation cost for retained earnings. As is said earlier, flotation costs are
not a significant factor except for small companies.

Flotation costs can be an important consideration in deciding the size of
the issue of securities, because these costs as a percentage of funds raised,
will decline with the size of the issue. Hence, greater the size of the issue
more will be the savings in terms of flotation costs. However, a large issue
affects the firm‘s financial flexibility. munotes.in

Page 113


Capital Structure: EBIT, EPS
MPS Approach
113  Market conditions:
Capital market conditions may change from time to time. Sometimes there
may be depression and at times there may be boom condition in the
market. The firm should decide whether to go for equity issue or debt
capital by taking market sentiments into consideration. In the case of
bearish conditions in the share market, the firm should not issue equit y
shares but go for debt capital. On the other hand, under boom conditions,
it becomes easy for the firm to mobilise funds by issuing equity shares.

The internal conditions of a firm may also determine the marketability of
securities. For example, a high ly levered firm may find it difficult to raise
additional debt. In the same way, a firm may find it very difficult to
mobilise funds by issuing any kind of security in the market merely
because of its small size.

 Statutory requirements:
The various guide lines issued by the Government from time to time
regarding the issue of shares and debentures should be kept in mind while
determining the capital structure of a firm. These legal restrictions are very
significant as they give a framework within which capi tal structure
decisions should be made.

8.5 EBIT:
Earnings before interest and taxes (EBIT) is an indicator of a company's
profitability. EBIT can be calculated as revenue minus expenses
excluding tax and interest. EBIT is also referred to as operating earni ngs,
operating profit, and profit before interest and taxes.
 EBIT (earnings before interest and taxes) is a company's net income
before income tax expense and interest expenses are deducted.
 EBIT is used to analyze the performance of a company's core
operations without the costs of the capital structure and tax expenses
impacting profit.
 EBIT is also known as operating income since they both exclude
interest expenses and taxes from their calculations. However, there
are cases when operating income can dif fer from EBIT.
8.6 EPS:

Earnings per share (EPS) is calculated as a company's profit divided by
the outstanding shares of its common stock. The resulting number serves
as an indicator of a company's profitability. It is common for a company
to report EPS that is adjusted for extraordinary items and potential share
dilution.
Earnings per share (EPS) is a company's net profit divided by the number
of common shares it has outstanding. munotes.in

Page 114


Financial Management
114 EPS indicates how much money a company makes for each share of its
stock and is a widely used metric for estimating corporate value.
A higher EPS indicates greater value because investors will pay more for
a company's shares if they think the company has higher profits relative
to its share price.1
EPS can be arrived at in several fo rms, such as excluding extraordinary
items or discontinued operations, or on a diluted basis.
Like other financial metrics, earnings per share is most valuable when
compared against competitor metrics, companies of the same industry, or
across a period of time.
8.7 MPS:

Market price per share simply refers to the most recent price of a single
share in a publicly -traded stock. This is not a fixed price —it fluctuates
throughout the trading day as various market forces push the price in
different directions. Unli ke the book value per share, the market price per
share has no specific relation to the value of the company's assets or any
other balance sheet information.
 Market price per share tells you the latest price for which a single
share of a company's stock wa s sold.
 Forces of supply and demand push market prices up and down
throughout the trading day.
 Market price per share is used to determine a company's market
capitalization.
8.8 SUMMARY:

Capital structure is the blend of funds employed in the corporation. Thi s
blend includes Equity share capital, Preference share capital, Retained
earnings and secured, unsecured borrowed funds. Equity share capital
holders are the deemed owners of an organization. Preference shares the
deemed debt which have fixed rate of divi dend. Retained earnings are the
undistributed profit which can be ploughed back to the employed capital.
Borrowed funds or Loan funds or Owed funds are further classified in
Secured debts or and Unsecured debts.

8.9 QUESTIONS:
Illustration: 01: Indian industr ies Ltd. is a profit -making company with a
paid-up capital of 100 lakhs consisting of 10 lakh ordinary shares of 10
each. Company is earning an annual pre -tax profit of 60 lakhs. The
company's shares are listed and are quoted in the range of 50 to 80. The
management wants to diversify production and has approved a project
which will cost 50 lakhs and which is expected to yield a pre -tax income munotes.in

Page 115


Capital Structure: EBIT, EPS
MPS Approach
115 of 40 lakhs p.a. To raise this additional capital, the following options are
under consideration of the management:
(a) To issue equity capital for the entire additional amount. It is expected
that the new shares (face value of 10) can be sold at a premium of 15.
(b) To issue 16% non -convertible debentures of 100 each for the entire
amount.
(c) To issue equity capita l for? 25 lakhs (face value of 10) and 16%non -
convertible debentures for the balance amount. In this case the
company can issue shares at a premium of 40 each. Advise the
management as to how the additional capital can be raised. Show
workings. Note : The m anagement wants to maximise the earning per
share to maintain its goodwill. The company is paying income -tax at
50%. (Oct. 2013, adapted)
Solution :
Indian industries Ltd.
Earnings Per Share under the Three Options (in lakhs)
Particulars Option I
(Issue of
Equity
only) Option II
(Issue of
Debenture
only) Option III
(Issue of equity
Debentures only) No. of equity shares:
Existing
New issued
Total numbers

16% debentures
Estimated total
income: From current
operations
From new projects
Less: Interest on 16%
debenture
Profit before tax
Tax at 50%

Profit after tax

Earnings per share 10
2 10
- 10
0.50
12 10 10.50
Nil
60

40 50
60

40 25
60

40
100
- 100
8 100
4
100
50 92
46 96
48
50 46 48
4.17 4.60 4.57 munotes.in

Page 116


Financial Management
116 Option II , namely issue of 16 debentures is most suitable to maximize
earning per share.
Working:
I
Additional amount of 50 lakhs entirely
Total amount 50 lakhs
Issue price including share premium 25
No. of shares to be issued 2 lakhs
II
Additional amount of 50 lakhs issued as equity and debenture in the ratio
of 1:1 on equity
Total amount of equity (new issue)? 25 lakhs Issue price including share
premium? 50 lakhs No. of shares to be issued 0.50 lakh
Illustration: 02: Captain Ltd. has Equity share capital of 25 lakhs, divided
into shares of 100 e ach. It wishes raise further 2 10,00,000 for expansion
programme. The company plans to the following Alternatives.
1. By issuing Equity Share Capital only.
2. 5,00,000 by issuing Equity Shares and 5,00,000 by 10% Debentures.
3.5,00,000 Equity Shares, 3,00, 000, 8% Preference Share Capital and
2,00.000, 10% Debentures.
4. 5,00,000, 10% Debentures and 5,00,000, 8% Preference Share Capital.
You are required to suggest the best alternative giving your comments,
assuming that the estimated EBIT after expansion is 5,00,000 and
Corporate Tax rate is 30%. (April 2014, adapted)
Solution:
Captain Ltd.
Financing Alternatives
Particulars Financing Alternatives
I II III IV
Equity Existing
Equity New
8% Pref Shares
Capital
9% Debentures 2500000
1000000
-
- 250000 0
500000
-
500000 2500000
500000
300000
200000 2500000
-
500000
500000
3500000 3500000 3500000 3500000

munotes.in

Page 117


Capital Structure: EBIT, EPS
MPS Approach
117 Calculation of EPS
Particulars Financing Alternatives
I (Rs.) II (Rs.) III (Rs.) IV (Rs.)
Pref Dividend
Int on Deb No. of Equity Shares (A) -
-
35000 -
50000
30000 24000
20000
30000 24000
50000
20000
EBIT
Less: Interest 500000
- 500000
50000 500000
20000 500000
50000
EBT
Less: Tax 30% 500000
150000 450000
135000 480000
144000 450000
135000
EAT
Less: Pref Dividend 350000
- 315000
- 336000
24000 315000
40000
Earnings available
for Equity
shareholder (B)
350000
315000
312000
275000
EPS
(B /A) 10 10.5 10.4 11

Analysis: EPS is highest in alterative IV. It is suggested 5,00,000, 10%
Debentures and 5,00,00 0, 8% Preference Share Capital offers best sources
for raising finance.
Illustration: 03: The existing capital structure of BAC Ltd. Is as follows:
The company earns a return before interest and tax at 12% and the tax on
income is 50%. Company wants to rai se 25.00,000 for its expansion
programme, for which it is considering following alternatives
(a) Issue of 20,000 Equity Shares at a premium of 25 per share.
(b) Issue of 10% Preference Shares
(c) Issue of 9% Debentures.
It is forecasted that the price -earning ratio i n case of these alternatives are
(a) 20 (b) 17 and (c) 16. Which alternative would you consider to be the
best?
Give reasons for your choice.
Also calculate expected market price in case of three alternative financing
proposals. munotes.in

Page 118


Financial Management
118 Solution:
BAC Ltd.
Capital structure
Source of
finance Alternatives
Existing A (Rs.) B (Rs.) C (Rs.)
Equity shares
Securities
premium
Retained
earnings
9% Pref shares
10% Pref capital
7% debentures
9% Debentures 4000000
-
1000000
2500000
-
2500000
- 6000000
500000
1000000
250000 0
-
2500000
- 4000000
-
1000000
2500000
2500000
2500000
- 4000000
-
1000000
2500000
-
2500000
2500000
Total 10000000 12500000 12500000 12500000

Calculation of EPS and Market price per shares
Source of finance Existing
(Rs.) A (Rs.) B (Rs.) C (Rs.)
EBIT (@ 12%)
Less: Int on Deb 1200000
175000 1500000
175000 1500000
175000 1500000
400000
EBT
Less: Tax @ 50%
Less: Pref Dividend 1025000
512000
225000 1325000
662500
225000 1325000
662500
475000 1100000
550000
225000
1.Earnings available
to Equity
sharehold ers
2.No. of equity
shares (No.) 287500
40000 437500
60000 187500
40000 325000
40000
3. Eps (i + ii)
4. P/E ratio
forecasted 7.19
- 7.29
20 4.69
17 8.12
16
Expected market
price per equity
shares (iii) * (iv) - 145.80 79.73 129.92

Alternative 'a' finan cing is the best, because market value per share of
Equity shares is the highest
Illustration: 04: Seven -up Ltd. has Equity Share Capital of 5,00,000
divided into shares of 100 each. It wishes: raise further 3,00,000 for
expansion -cum-moderation scheme. Th e company plans the following
financing alternatives
munotes.in

Page 119


Capital Structure: EBIT, EPS
MPS Approach
119 (I) By issuing Equity Shares only.
(II) 1,00,000 by issuing Equity Shares and 2,00,000 through Debentures or
term loan @ 10%per annum.
(III)By raising term loan only at 10% per annum.
(IV) 1,00,000 by i ssuing Equity Shares and 2,00,000 by issuing 8%
Preference Shares: You are required to suggest the best alternative giving
your comments assuming that the estimated 'Earning Before Interest and
Taxes (EBIT)' after expansion is 1,50,000 and corporate rate o f tax 35%
(April 2011, adapted
Solution:
Seven Up Ltd.
Calculation of EPS
Particulars Financing alternatives
I II III IV
Equity
-Existing
-New
8% Preference Shares
10% Term Loan/Debenture
500000
300000
-
-
500000
100000
-
200000
500000
-
-
300000
500000
100000
200000
-
800000 800000 800000 800000

Calculation of EPS
Particulars Financing alternatives
I II III IV
Preference dividend
Int on term loan or
debentures
No. of Equity shares (A)
EBIT
Less : Interest
EBT
Less : 30% tax
EAT
Less : Pref. dividend
Earnings available for equity
shareholder (B)
EPS (B/ A) -
-
8000
150000
-
150000
52500
97500
-
97500
12.19 -
20000
6000
150000
20000
130000
45500
84500

84500
14.08 -
30000
5000
150000
30000
120000
42000
78000
-
78000
15.60 16000
-
6000
150000
-
150000
52500
97500
16000
81500
13.58
munotes.in

Page 120


Financial Management
120 Analysis: From the above analysis, it is observed that EPS is highest with
the 3rd alternative, i.e. further financing of * 3.00.00 can be done by
raising term loan onl y at 10% per annum.
Illustration: 05: Amount Products Ltd. wants to raise 100 lakhs for a
diversification project. Current estimated earnings before interest and
taxes (EBIT) from the new projects * 22 lakhs per annum. Cost of debt
will be 15% for amounts up to and including 40 lakhs, 16% for additional
amounts up to and including 50 lakhs and 18% for additional amounts
above 50 lakhs. The equity shares (face value 10) of the company have a
current market value of? 40. This is expected to fall to 32 if debt s
exceeding 50 lakhs are raised. The following options are under
consideration of the company:
OPTION. Equity. Debt
I 50%. 50%
II 60%. 40%
III. 40%. 60%
Determine the earning per sha re (E.P.S.) for each option and state which
option the company should exercise. Tax rate applicable to the company is
50%. (ICWA, Dec. 1997, adapted)
Solution:
Statement Showing Comparative Earnings Per Share
Particulars Option I
lakhs Option II
lakhs Option III
lakhs
Equity
Equity shares (to issue) Nos.
Debt
EBIT
Less: interest on debt
EBT
TAX@50%
PAT
EPS 50
125000
50
22. 00
7.60
14.40
7.20
7.20
5.76 60
150000
40
22.00
6.00
16.00
8.00
8.00
5.33 40
125000
60
22.00
9.40
12.60
6.30
6.40
5.04

Option I be ing the



munotes.in

Page 121


Capital Structure: EBIT, EPS
MPS Approach
121 Working note:
Option / Debt in Rs. Lakhs Interest Rate Amount
I. 50

II. 40 III. 60 First 40 lakhs
Next 10 lakhs

On 40 lakhs
First 10 lakhs
Next 10 lakhs
Next 10 lakh's 15
16

15
15
16
18 6.00
1.60
7.60
6.00
6.00
1.60
1.80
9.40

8.10 REFERENCES:

1. Sharma and Gupta, Financial Management.
2. Khan and Jain, Financial Management.
3. Pandey, I M, Financial Management.
4. Hampton, John J., Financial Management and Policy.
5. Solo man, Ezra & Pringle, John J., An Introduction to Financial
Management,
6. Weston J., Fred, & Drigham, Managerial Finance,


munotes.in

Page 122

122 9
CAPITAL STRUCTURE: PLANNING AND
LEVERAGE ANALYSIS
Unit Structure
9.0 Objectives
9.1 Introduction
9.2 Meaning of Leverage
9.3 Operating Leverage
9.4 Financial Leverage
9.5 Degree of Combined Leverage
9.6 Significance of Financial and Operating Leverages
9.7 Cost of Capital
9.8 Self Assessm ent Test
9.9 References
9.0 OBJECTIVES:
After studying this chapter you will be able to:
 Define, discuss, and quantify―busin ess risk and―financial risk.
 Explain in detail operating and financial leverage and identify causes
of both.
 Understand how to calculate and interpret firm’s leverage?
 Calculate a firm‘s operating break -even (quantity) point and break -
even(sales) p oint
 Understandwhatisinvolvedindeterminingtheappropriateamountoffinanc
ialleverageforafirm?
9.1 INTRODUCTION
The main objective of this lesson is to make the students learn about the
basic concepts of leverages with reference to operating, financial and
composi te leverages. The financing or capital structure decision is of
fabulous significance for the management, as it influences the debt -equity
blend of the company, which ultimately affects shareholders return and
risk.
munotes.in

Page 123


Capital Structure: Planning
and Leverage Analysis
123 If it is not obligatory for a firm to p ay fixed cost or fixed return, there will
be no leverage. Since fixed cost or return has to be paid or incurred
irrespective of the level of output or sales, the size of such cost or return
has substantial influence over the amount of profits available for the
shareholders. When the level of sales changes, leverage helps in
quantifying such influence, It may therefore be defined as relative change
in profits due to a change in sales. A high degree of leverage implies that
there will be a stout change in pro fits due to a relatively minute change in
sales and vice versa. Thus, higher is the leverage, higher is the risk and
higher is the expected return.
9.2 MEANING OF LEVERAGE
Thedictiona rysenseofthete rmlev eragerefersto―anincreasedmeansofaccompl
ishingsome purpose. For example, leverage helps us in lifting heavy
objects, which may not be otherwise feasible. Nevertheless, in the area of
finance, the term leverage has a special connotation. It isused to describe
the firm‘s ability to use fixed cost assets or funds to blow up the return to
it sowners.
JamesVanHornehasdefinedlevera geas―theemploymentofanassetorfundsfor
whichthe firm pays a fixed cost or fixed return. A high degree of leverage
implies that there will be a stout change in profits due to a relatively
minute change in sales and vice versa.Thus, higher is the leverage, higher
is the risk and higher isthe expected return.
Types of Leverages
Leverages are of three types: (i) Operating leverage, (ii) Financial leverage
and (iii) Composite leverage . Let us discuss these leverages taking one by
one.


Types of Leverages

munotes.in

Page 124


Financial Management
124 9.3 OPERATING LEVERAGE
Theoperatingleveragemaybedefinedasthetendencyoftheoperatingprofittova
rydisproportionately with sales.It is assumed to exist when a firm has to
pay fixed cost regardless of level of output or sales.The firm is said to
have a high degree of operating leverage if its employs a greater amount
of fixed costs and a small amount of variable costs. On the other hand, a
firm will have a low operating leverage when it employs a greater am ount
of variable costs and a smaller amount of fixed costs. Thus, the degree of
operating leverage depends upon the amount of fixed elements in the cost
structure. Operating leverage in a firm is a function of three factors:
1. The amount of fixed costs.
2. The contribution margin.
3. The volume of sales.

Of course, there will be no operating leverage, if there are no fixed
operating costs. Computation of Operating Leverage : The operating
leverage can be calculated by the following formula:

Operating profit here means―Earnings Before Interestand Tax (EBIT).
Operating leverage may be favourable or unfavourable. In case the
contribution (i.e., sales less variable cost) exceeds the fixed cost, there is
favourable operating leverage.In a reverse case, the operating lever age
will betermed as unfavourable.
Degree of operating leverage : The degree of operating leverage may be
defined as percentage change in the profits resulting from a percentage
change in the sales i t.
May be put in the form of following formula :
munotes.in

Page 125


Capital Structure: Planning
and Leverage Analysis
125
Operat ing leverage is directly proportional to business risk. More
operating leverage leads to more business risk, for then a small sales
decline causes a big profit. This can be illustrated graphically as:

Illustration1 : A Company produces and sells 10,000 shirts. The selling
price per shirtis Rs 500. Variable cost is Rs. 200 per shirt and fixed
operating cost is Rs. 25,00,000.
(a) Calculate operating leverage.
(b) If sales are up by10%, then what is the impact on EBIT?
Solution
(a) Statement of Profit ability Rs.
Sales Revenue(10,000× 500) 50,00,000
Less : Variable Cost(10,000 ×200) 20,00,000
Contribution 30,00,000
Less: Fixed Cost 25,00,000
EBIT 5,00,000
munotes.in

Page 126


Financial Management
126
Solution:


The operating leverage exists only when there are fixed costs. In the case
of firm D, ther e is no magnified effect on the EBIT due to change in sales.
A 20 per cent increase in sales has resulted in a 20 per cent increase in
EBIT. In the case of other firms, operating leverage exists. It
ismaximuminfirmA,followedbyfirmCandminimuminfirmB.Theinte rceptio
nofDOLof7is that 1 per cent change in sales results in 7 per cent change in
EBIT level in the direction of the change of sales level of firm A.

munotes.in

Page 127


Capital Structure: Planning
and Leverage Analysis
127 9.4 FINANCIAL LEVERAGE
Financial leverage (FL) maybe defined as ‗the use of funds with a fixed
cost in order to increase earnings per share.‘ In other words, it is the use of
company funds on which it pays a limited return. Financial leverage
involves the use of funds obtained a t a fixed cost in the hope of increasing
the return to common stockholders.
Degree of financial leverage is the ratio of the percentage increase in
earnings per share (EPS) to the percentage increase in earnings before
interest and taxes (EBIT).
Favourable and unfavourable financial leverage: Financial leverage
may be favourable orunfavourable depending upon whether the earnings
made by the use of fixed interest or dividend bearing securities exceeds
the explicit fixed cost, the firm has to pay for the empl oyment of such
funds, or not. The leverage will be considered to be favourable if the firm
earns more on assets purchased with the funds than the fixed costs of their
use. Unfavourable or negative leverage occurs when the firm does not earn
as much as the funds cost.
Trading on equity and financial leverage : The Financial leverage is
sometimes termed as
―trading on equity . However, most of the scholars on financial
manag ement are of the opinion that the term trading on equity should be
used for the term financial leverage only when the
financial leverage is favourable. The company resorts to trading on equity
with the objective of giving the equity shareholders a high rate of return
than the general rate of earning on capital employed in the company, to
compensate them for the risk that they have to swallow.
Computation of Financial Leverage : The compu tation of financial
leverage can bed one according to the following methods:
(i) Where capital structure consists of equity shares and debt: In
such a case, financial leverage can be calculated according to the
following formula:
Financial leverage=OP/PBT
Wher e, OP = Operating profit or earnings before interest and tax.(EBIT)
PBT = Profit before tax but after interest.
Illustration 3 : A company has any choice of the following three financial
plans. You are required to calculate the financial leverage in each case and
interpret it.

munotes.in

Page 128


Financial Management
128 X Y Z
Equity Capital 2000 1000 3000
Debt 2000 3000 1000
Operating Profit (EBIT) 400 400 400
Interest @ 10%ondebtin all cases and tax rate 50% only.
Solution:
The financial leverage will be computed as follows incase of each of these
financial plans :
X Y Z
Operating Profit(OP) 400 400 400
Interest(10%ondebt) 200 300 100
Profit before tax PBT 200 100 300
Financial leverage 400/200 400/100 400/300
=2 4 1.33

Financial leverage, as explained earlier, indicates the change th at will take
place in the tax able income as a result of change in the operating income.
For example, taking Financial Plan X as the basis, if the operating profit
decreases to Rs.200, its impact on taxable income will be as follows:
Operating Profit (OPor EBIT) Rs. 200
Less : Interest Rs. 200
Profit before tax PBT NIL
Financial leverage in case of plan X is 2. It means every 1% change in
operating profit will result in 2% change in the taxable profit. In the above
case operating profit has decreased from Rs .400to Rs.200 (i.e. 5%
decrease), as a result the taxable profit has decreased from Rs.200 to
zero100%decrease.
Degree of financial leverage is the ratio of the percentage increase in
earnings per share (EPS) to the percentage increase in earnings before
inter stand taxes (EBIT). munotes.in

Page 129


Capital Structure: Planning
and Leverage Analysis
129

Illustration 4 : A Company has the following capital structure. Rs.
Equity share capital 1,00,000
10% Prof. share capital 1,00,000
8% Debentures 1,25,000
The present EBIT is Rs.50,000. Calculate the financial leverage assuring
that the company is in 50% tax bracket.
Solution : Statement of Profit Rs.
Earning Before Interest and Tax (EBIT) 50,000 (or) Operating Profit
Less : Interest on Debenture1,25,000×8×100 -10,000
Earning before Tax (EBT) 40,000
Income Tax 50% of 40,000 -20,000
Profit 20,000
Financial leverage = Operating Profit (OP) =50,000
Profit Before Tax (PBT) 40,000 =1.25
Financial leverage helps to examine the relationship between EBIT and
EPS. Financial leverage measures the percentage of change in taxable
income to the percentage change in EBIT.
Financial leverage locates the correct profitable financial decision
regarding capital structure of the company. Financial leverage is one of the
important devices which is used to measure the fixed cost proportion with
the total capital of the company.
If the firm acquires fixed cost funds at a higher cost, then the earnings
from those assets, the earning per share and return on equity capital will
decrease.
(ii) Where the capital structure consists of preference shares and
equity shares . The formula for computation of financial leverage can
also be applied to a financial plan having preference shares. Of course, the
amount of preference dividends will have to be grossed up (as per the tax
rate applicable to the compan y) and then deducted from the earnings
before interest and tax. Illustration 5 : The capital structure of a company
consists of the following securities: munotes.in

Page 130


Financial Management
130 10 % Preference share capital Rs.100000
Equity share capital (Rs.10/ -shares) Rs.100000
The amount of o perating profit is Rs. 60,000. The company is in 50% tax
bracket. You are required to calculate the financial leverage of the
company. What would be new financial leverage if the operating profit
increase to Rs.90000 and read between the lines your results ?
Solution: Computation of the present financial leverage
Operating profit (OP or EBIT) Rs.60, 000
Less : Preference dividend (after grossing up) Rs.20, 000
PBT Rs.40, 000
Present Financi al Leverage =OP/PBT =60000/40000=1.5
Computation of new financial leverage
New operating Profit Rs.90000
Less : Preference Dividend (after grossing up) Rs.20000
PBT Rs.70000
Financial Leverage =OP/ PBT=90000/70000=1.286
The existing financial leverage is 1.5. It means 1% change in operating
profit (OP or EBIT) will cause in taxable profit (PBT) in the same
direction. For example, in the present case operating profit has increased
by50% (i.e. from Rs. 60000 to Rs. 90000).
This has resulted in 75% increase in the tax able profit (i.e. from Rs.40000
toRs.70000).
(iii) Where th e capital structure consists of equity shares,
preferences, shares and debt: In such a case the financial leverage is
calculated for deducting from operating profit both interest and preference
dividend on a before tax basis.
Illustration 6 : A company has the following capital structure:
Equity share capital Rs. 1,00,000
10%Preference share capital Rs. 1,00,000
8% Debentures Rs. 1,25,000
The present EBIT is Rs .50,000. Calculate the financial leverage assuming
that company is in 50 % tax bracket.

munotes.in

Page 131


Capital Structure: Planning
and Leverage Analysis
131 Solutio n: Operating Profit Rs.50,000
Less: Interest on debenture Rs.10,000
Pref. dividend (pre-tax basis) Rs. 20,000
Profit before tax Rs. 20,000
Financial leverage = OP/ PBT = 50,000/ 20,000 = 2.5
Illustration 7: A company has the following capital structure;10 ,000
Equity shares of Rs.10 each Rs.1,00,000
2,00010 % Prof. shares of Rs.100 each Rs. 2,00,000
2,00010 % Debentures of Rs.100 each Rs.2,00,000
Calculate the EPS for each of the following levels of EBIT:
(i)Rs. 1,00,000 (ii)Rs. 60,000 (iii) Rs. 1,40,000.
The company is in 50 % tax bracket. Calculate also the financial leverage
EBIT level under (i) as base
Solution: Computation of earnings per share

(i)Rs. (ii)Rs. (iii)Rs.
EBIT 1,00,000 60,000 1,40,000
Less: Interest on debenture -20,000 -20,000 -20,000
PBT 80,000 40,000 1,20,000
Less: Income Tax -40,000 -20,000 -60,000
PAT 40,000 20,000 60,000
Less: Preference dividend -20,000 -20,000 -20,000
Earnings available for equity
Shareholders(EAS) 20,000 ———
40,000 Earningspersha re(EPS) 2
Nil 4
The above table shows that (a) Incase (ii) the EBIT has decreased by 40%
(i.e. from Rs.1,00,000 to Rs. 60,000 while the earning per share has
decreased by 100% (from Rs. 2 per share to nil); (b) In case (iii) the EBIT
has increased by 40% (from Rs. 1,00 ,000 to Rs. 1,40,000 as compared to
case(i), while the earning per share has increased by100%(from Rs. 2 to
Rs. 4).
The degree of financial leverage can there fore be computed as follows
:DFL=Percentage change in EPS/Percentage change in EBIT munotes.in

Page 132


Financial Management
132 Financial Leverage in between(i) and (ii)=100/40
=2.5Financia
lL everage in between(i) and(iii)=100/40 =2.5
The same result can be obtained by using the equation OP/PBT as shown
below.
Computation of Financial Leverage
(i)Rs. (ii)Rs. (iii)Rs.
OP 1,00,000 60,000 1,40, 000
Less: Interest 20,000
Preference Dividend 40,000 -60,000 -60,000 -60,000
(Grossed up)
PBT 40,000 -- 80,000
Financial leverage =OP/PBT =2.5
This means that with every 1% change in operating profit (OP), profit
before tax (PBT) will chan ge (in the same direction) by 2.5%. For
example, in situation (ii) OP has decreased by 40%.This has resulted in
decrease of PBT by 100% (i.e., 40 x 2.5). In situation (iii) OP has
increasedby40%. This has resulted of PBT by100%(i.e.,40 x2.5).
Usefulness: Financial leverage helps considerably the financial manager
while devising the capital structure of the company. A high financial
leverage means high fixed financial costs and high financial risk. A
financial manager must plan the capital composition in a w ay that the
firmisinapositiontomeetitsfixedfinancialcosts.Increaseinfixedfinancialcosts
requiresindispensable increase in EBIT level. In the event of collapse to do
so, the company may be in principle forced into insolvency.
9.5 DEGREE OF COMBINED LEVERAGE
Comb inedleveragemaybedefinedasthepotentialuseoffixedcosts,bothoperati
ngandfinancial, which magnifies the effect of sales volume change on the
earning per share of the firm. Degree of combined leverage (DCL) is the
ratio of percentage change in earning per shar e to the percentage change in
sales. It indicates the effect the sales changes will have on EPS.

munotes.in

Page 133


Capital Structure: Planning
and Leverage Analysis
133 Illustration 8: A company has sales of Rs. 1,00,000.The variable costs are
40% of the sales while the fixed operating costs amounts to Rs.
30,000.The amount of interest on long -term debt is Rs. 10,000. You are
required to calculate the composite leverage and illustrate its impact if
sales increaseby5%.
Solution:
Statement showing computation of composite leverage
Sales 1,00,000
Less: Variable costs(40%ofsale s) -40,000
Contribution © 60,000
Less: Fixed operating costs 30,000
Earnings before interest and tax (EBIT) or Operating
profit (OP) 30,000
Less: Interest 10,000
Taxable Income(PBT) 20,000
Composit eleverage=C/PBT=60000/20000=3.
The composite lever age of‗3 ‘indicates that with each
increase ofRe.1insales,thetaxable
Income will increase by Rs. 3 (i.e. 1×3).
This can be verified by the following computations when the sales increaseby5%.
Sales 1,05,000
Less: Variable costs -42,000
Contribution( C ) 63,000
Less: Fixed operating costs -30,000
Earnings before interest and tax (EBIT)or Operating
profit (OP) 33,000
Less: Interest -10,000
Taxable Income (PBT) 23,000
It is clear from the above computation that on account of increase in sales
by 5% the profit before tax has increasedby15%. This can be verified as
follows:
Increase in percentage profits=[Increase in profit/Base Profit]×100
=[3,000/20,000]×100 =15% munotes.in

Page 134


Financial Management
134 9.6 SIGNIFICANCE OF FINANCIAL AND
OPERATING LEVERAGES
The operating leverage and the fina ncial leverage are the two quantitative
tools used by the financial experts to measure the return to the owners
(viz., earning per share) and the market price of the equity shares. The
financial leverage is considered to be superior of these two tools, since it
focuses the attention on the market price of the shares, which the
management always stories to increase by in creasing the net worth of the
firm.
The management for this purpose resorts to trading on equity because
when there is increase in EBIT the n there is corresponding increase in the
price of the equity shares. However, low operating leverage and a high
financial leverage is well thought -out to be an ideal situation for the
maximization ofthe profits with bare minimum of risk.
9.7 COST OF CAPITA L:
Cost of capital is another important factor that should be kept in mind
while designing the capital structure of a firm. The capital structure should
be designed in such a way that the firm‘s overall cost of capital (k o) is the
minimum. Cost of capital is the minimum return expected by its suppliers.
Of all the sources of capital, equity capital is the costliest as the equity
shareholders bear the highest risk. Cost of Equity (k e) > Cost of Debts (k d)
On the other hand, debt capital is the cheapest sour ce because the interest
is paid on it by the firm whether it makes profits or not. Moreover, interest
on debt capital is tax deductible which makes it further cheaper. Cost of
Preference share capital (k p) is also cheaper than equity capital as the
dividen ds are paid at a fixed rate on preference shares. Retained earnings
(kr) do not require any flotation cost. So, the overall cost of capital
depends on the proportion in which the capital is mobilized from different
sources of finance. Hence, capital struc ture should be designed carefully
so that overall cost of capital is minimized.
1. Cost of Debt =a) k d= I (1 -t)
b)k d = I(1 -t) / NP [ NP = FV + Premium( -discount) - Flotation Costs ]
c) k d = I(1-t) + (RV - NP)/N / (RV + NP / 2)
RV= Redeemable value
NP = Net Proceeds = FV + Premium( - discount) - Flotation Costs

2. Cost of Preference = k p= Preference Dividend / Net Proceeds X
100 [ PD/NP X 100 ]
kp= PD + (RV - NP / N) / (RV +NP /2)

3. Cost of Equity = k e = Dividend / Price X 100 + Grow th = [Ke =
D/P X 100 + g]
k e = E / P X 100 + G [Earning / Price X 100 +
Growth]
munotes.in

Page 135


Capital Structure: Planning
and Leverage Analysis
135 4. Weighted Average Cost of Capital = WACC = Weights X Cost of
Capital
Q. 01. A ltd issued 12,000 debentures, 10% of Rs 100 each at par. Th e Tax
rate is 50%. Calculate after tax cost of debt.
Solution: I = 100 x 10% = 10, T= 50% 0.5
kd = I(1 -t)
= 10 (1 -0.5)
kd = 5%
I = 12,000 x 100 x 10% = 60,000, T = 50%
kd = I(1 -t) = 120,000 (1 - 0.5)
kd = Rs 60,000
Q.02. V ltd issued Rs 300,000, 8% Debentures of Rs 100 each at a
premium of 10%. The Flotation costs are 2% on the net proceeds. The tax
rate is 50%. Ascertain Cost of Debt after tax.
Solution: FV = 300,000, Np = 300,000 + 10% - 2% = Rs 323,400 Interest
= Rs 24,000, T= 50%
kd = I(1 -t) / Np
= 24,000(1 - 0.50) / 323,400
kd = 3.71 %
Q.03. Calculate Cost of Capital, Rs 20,00,000 14% Debentures of Rs 100
each issued at 95Rs each, Redeemable at 120 Rs Each After 7 years. Tax
rate applicable is 40%.
Solution:
FV= Rs 100, NP= 95, RV= Rs 120, N= 7yrs, T=0.40, I= 14
kd = I(1 -t) + (RV - NP)/N / (RV + NP / 2) kd = 11.33% kd = 11.33% kd = 11.33% kd = 11.33%

kd = 11.33%
munotes.in

Page 136


Financial Management
136 Q.04 D Ltd issued 9% Preference shares of Rs 100 each. The issue
expenses were Rs 3 per share. Calculate Kp in following cas es:
a. Issued at par b. Issued at premium of 10% c. Issued at discount of 10%
Solution:
kp= PD / NP X 100
a. Issued at par
PD = 100 X 9% = 9Rs , NP = 100(par) - 3=97Rs
= 9 / 97 x 100 = 9.28%

b. Issued at premium of 10%
PD = Rs 9, NP = 100 + 10% - 3 = 107Rs
= 9 / 107 x 100 = 8.41%

c. Issued at discount of 10%
PD= 9 , NP = 100 -10%(DISC) - 3(Exp)
= 9 / 87 x 100 = 10.34%
Q.05. K ltd issued 8800 10% Preference Shares of Rs 100 each at par. Shares are
redeemable after 5 years at premium of 10%. The Issue expenses are 4% of Face
value. Calculate k p ?
Solution:
kp = PD + (RV - NP / N) / (RV + NP /2) X 100
PD = 100 X 10% =Rs 10,
NP = Rs100(par) - 4% =96Rs,
RV = 100 +10%(premium) =Rs110,
N= 5 yrs
kp = 10 + (110 - 96 / 5) / (110+96/2) x 100
kp = 12.8 / 103 x 100
kp = 12.43%

munotes.in

Page 137


Capital Structure: Planning
and Leverage Analysis
137
Q.06. For T ltd, Share is quoted at Rs 4, Dividend Paid is Rs 1 per share,.
The growth rate expected is 6%p.a. Calculate k e ?
Solution:
ke = D / P X 100 + G
ke = 1 / 4 X 100 + 6
ke = 25 + 6 = 31%

Q.07. For S ltd Earnings P er Share is Rs 3, Market price is Rs 12 &
expected growth rate is 10% Calculate Cost of Equity?
Solution:
ke = E / MP X 100 + G
ke = 3/12 X 100 +10
ke = 25 + 10 = 35 %

Q.08
Source of Capital Capital Amt Cost of Capital
Equity 3,00,000 8%
Debt 6,00,0 00 13%
Preference Shares 4,00,000 5%
Determine WACC of Moon ltd?

Solution:
Source of Capital Capital Amt Weights (W) Cost of Capital (C) W X C
Equity 3,00,000 0.2307 8 1.8456
Debt 6,00,000 0.4615 13 5.9995
Preference Shares 4,00,000 0.3076 5 1.538 13,00,000 1 WACC = 9.38% munotes.in

Page 138


Financial Management
138 Q. 09.
Following are the details of ABC Ltd
10% Debentures (Rs 100 per debenture) -Rs 10,00,000
8% Preference shares (Rs 100 per share) - Rs 500,000
Equity Shares (Rs 10 per share) - Rs 20,00,000
Dividend expected at the end of the year Rs 3 per share, growth rate in dividend in 10% & tax rate is 40%
Calculate WACC by considering above information?
Solution:

10% Debentures, Kd = I(1 - t)
Kd = 10( 1 - 0.40) Kd = 0.6 ~ 6%

8% Prefernce Shares Kp = PD / NP X 100
kp = 8 / 100 x 100 kp = 8%

Equity Shares = K e = D/P X 100 + G
Ke= 3 / 10 X 100 +
10 Ke = 40%

Source of Capital Capital Amt Weights
(W) Cost of
Capital
(C) W X C
10% Debentures 10,00,000 0.286 6 1.7142
8% Preference
Shares 5,00,000 0.143 8 1.144
Equity Shar es 20,00,000 0.571 40 22.856
35,00,000 WACC = 25.712 munotes.in

Page 139


Capital Structure: Planning
and Leverage Analysis
139 9.9 PRACTICAL QUESTIONS:
Solved:
Statement of Income
Particulars Rs Rs
Sales
- Variable Cost
Contribution
- Fixed Cost
EBIT
- Interest
EBT
- Tax
EAT / NPAT
- Prefere nce Dividend
Profit for Eq. Shareholders
/ No of Equity Shares
EPS

Q.01. From the following, Calculate, OL, FL & CL Sales= Rs. 300,000, Variable Cost= Rs. 72,000, Interest= Rs. 100,000 @ 10%
Fixed Cost= Rs. 50,000 , Tax @ 50%
Solution:
Particulars
Sales 300,000
-Variable Cost -72,000
Contribution 228,000
-Fixed Cost -50,000
EBIT 178,000
- Interest 100,000 x 10% -10,000
EBT 168,000
- Tax @ 50% -84,000
EAT 84,000 munotes.in

Page 140


Financial Management
140 OL= 228,000 / 178,000 = 1.28 times
FL = 178,000 / 168,000 = 1.06 ti mes
CL = 1.28 X 1.06 = 1.36 times



Determine Leverages:
Solution:
Particulars
Income Stat ement
Particulars Rs
Sales(100,000U X 120Rs) 1,20,00,000
-Variable Cost(100,000U X 90Rs) (90,00,000)
Contribution 30,00,000
-Fixed Cost* 15,00,000
EBIT 15,00,000
-Interest* -300,000
EBT 12,00,000
-Tax @30% x 12,00,000 -360,000
EAT 840,000

Q.2
Sales 100,000 units
Variable Cost Rs 90 p.u Fixed Cost Including Interest 18,00,000
Selling Price p.u 120Rs
10% Debentures Rs 30,00,000
Tax Rate 30%
munotes.in

Page 141


Capital Structure: Planning
and Leverage Analysis
141 Fixed including Interest 18,00,000
-Interest (30,00,000 X 10%) -300,000
Fixed Cost 15,00,000

i) Operating Leverage= Contribution / EBIT 30,00,000/15,00,000 2times

ii) Financial Leverage= EBIT / EBT 15,00,000/12,00,000 1.25times
iii) Combined Leverage= Contribution / EBT 30,00,000/12,00,000 2.5times
CL= OL X FL
Q.03 A firm has sales of Rs 40 lacs, Variable cost of Rs 25 lacs, Fixed Cost of Rs 6 lacs 10% Debentures of Rs 30 lacs & Equity Capital of Rs 45 lacs of Rs 10 Each.
Calculate Lev erages & EPS
Solution:
Statement of Income
Particulars Rs.
Sales 40,00,000
-Variable Cost (25,00,000)
Contribution 15,00,000
-Fixed Cost -600,000
EBIT 900,000
-Interest(30,00,000x10%) -300,000
EBT 600,000

No of Equity shares=45,00,000 / 10Rs
450,000 Shares
munotes.in

Page 142


Financial Management
142 i) Operating Leverage= Contribution / EBIT 15,00,000/900,000 1.67times

ii) Financial Leverage= EBIT / EBT 900,000/600,000 1.5times

iii) Combined Leverage= Contribution / EBT 1.67 x 1.5 2.5times
CL= OL X FL

iv) Earnings Per Share=Profit Available to Equity Shareholders / No of Equity Shares Rs 600,000 / 450,000 Shares 1.33Rs


Q.04
Sales 90,00,000
Variable Cost @60%
Annual Fixed Cost 10,00,000
(Excluding Interest)
Tax Rate @ 40%
400,000 Equity shares of Rs 10 each
12% Debentures 40,00,000
Preference Shares @ 10% 30,00,000
Calculate Leverages & EPS
Solution:
Statement of Income
Particulars Rs
Sales 90,00,000
- Variable Cost @60% (54,00,000)
Contribution 36,00,000
-Fixed Cost (10,00,000)
EBIT 26,00,000
-Interest(12% x 40,00,000) -480,000 munotes.in

Page 143


Capital Structure: Planning
and Leverage Analysis
143 EBT 21,20,000
-Tax @ 40% x 21,20,000 -848,000
EAT 12,72,000
-Preference Dividend -300,000
(10% x 30,00,000)
Profit Available to
Equity share holders 972,000
/No of Equity Shares 40,000 EPS 24.3Rs


i) Operating Leverage= Contribution / EBIT 36,00,000/26,00,000 1.38times

ii) Financial Leverage= EBIT / EBT 26,00,000/21,20,000 1.22times

iii) Combined Leverage= Contribution / EBT 1.38 x 1.22 1.68times


iv) EPS=Profit Available to Equity Shareholders /No of Equity Shares 972,000/400,000 24.3Rs

No of Equity Shares = 400,000 / 10 = 40,000 shares

Q.05 LS Ltd
Operating Leverage 3:1
Financial Leverage 2:1
Interest Charges 20,00,000
Tax Rate 50%
Variable Cost Percentage 60% munotes.in

Page 144


Financial Management
144 Prepare Income Statement

OL= C/EBIT
FL= EBIT / EBT
EBT = EBIT – Interest
Sol:
WN 1
Financial Leverage= EBIT / EBT
2 = EBIT / EBIT - Int
2 = EBIT / [EBIT - 20,00,000]
2(EBIT - 20,00,000)= EBIT
2EBIT - 40,00,000 = EBIT
2EBIT -1 EBIT = 40,00,000
EBIT = 40,00,000

WN 2
Operating Leverage = C/EBIT
3 = C / 40,00,000......WN.1
Contribution = 1,20,00,000

Income Statement
Particulars Rs.
Sales 3,00,00,000 S 100 ? 3,00,00,000 1,20,00,000 / 40 x 100
-VC (60%) 1,80,00,000 V.C 60 ?
Contribution 1,20,00,000 C 40 1,20,00,000 munotes.in

Page 145


Capital Structure: Planning
and Leverage Analysis
145 -FC (b.f) 80,00,000
EBIT 40,00,000
-Int (Given) (20,00,000)
EBT 20,00,000
-Tax(50%) (10,00,000)
EAT 10,00,000

Q.06
Operating Leverage=4:1
Financial Leverage=2:1
Annual Interest paid=Rs 10,00,000
Contribution / Sales =0.4
Tax Rate= 40%
Prepare Income Statement

Sol:
Working note: 01
Financial Leverage = EBIT / EBT
2 = EBIT / EBIT – Int
2 = EBIT / EBIT - 10,00,000
2 X (EBIT - 10,00,000) = EB IT
2EBIT - 20,00,000 = EBIT
2EBIT - EBIT = 20,00,000
EBIT = 20,00,000

munotes.in

Page 146


Financial Management
146 Working note: 02
Operating Leverage = C / EBIT
4 = C / 20,00,000
C = 80,00,000

C / Sales = 0.4 i.e 40%
100 Sales 2,00,00,000 80,00,000 / 40 X 100
60 -VC 120,00,000
40 Cont ribution 80,00,000

Practice Questions;
01. Calculate Ke for Sanjana Ltd in Following cases:
1. Market price Rs 25 per share, Dividend is Rs 9 per share
2. Net proceeds being 125Rs, Dividend being Rs 50, growth rate being 8%
3. Earnings per share being Rs 6.6, Market proceeds being 60, growth rate
being 10%
4. Earnings per share being 10, market price being 100 Rs
9.10 EFERENCES
1. Sharma and Gupta, Financial Management.
2. Khan and Jain, Financial Management.
3. Pandey, IM, Financial Management.
4. Hampton, John J., Financial Management and Policy.
5. Soloman, Ezra & Pringle, John J., An Introduction to Financial
Management,
6. Weston J.,Fred, & Drigham, Managerial Finance,


munotes.in

Page 147

147 10
DIVIDEND POLICY
Unit Structure
10.0 Objectives
10.1 Introduction
10.2 Meaning of Dividend
10.3 Types of dividend/ form of dividend
10.4 The Dividend Payment Time Line
10.5 Dividend Decision
10.6 Relevance of dividend policy
10.7 Determinants of dividend policy
10.8 Questions
10.9 References
10.0 OBJECTIVES
After studying this chapter you will be able to:
 Define dividend and its various forms
 The concept of Dividend policy.
 Know the objectives and determinants of Dividend policy.
 Various approaches of Dividend policy.
10.1 INTRODUCTION
Financing an enterprise thro ugh its internal sources is known as internal
financing. Such internal resources comprise of earnings retained by the
company in the form of income left over after meeting all expenses. Such
funds are available to an enterprise which has been carrying on i ts
business successfully and thereby has been in a position to set aside a
portion of its earnings for future needs. This retaining of earnings is
technically termed as ploughing back of profits.
Dividend decision relates to how much of the company’s net p rofit is to be
distributed to the shareholders and how much of it should be retained in
the business for meeting the investment requirements.
This decision should be taken, keeping in view the overall objective of
maximising shareholders’ wealth. Every com pany has to decide whether munotes.in

Page 148


Financial Management
148 to return cash to its stockholders and, if yes, how much in the form of
dividends. This is the dividend decision, and we begin this chapter by
providing some background on three aspects of dividend policy. One is a
purely procedu ral question about how dividends are set and paid out to
stockholders. The second is an examination of widely used measures of
how much a firm pays in the dividends. The third is an empirical
examination of some patterns that firms follow in dividend polic y.
10.2 MEANING OF DIVIDEND
Dividend refers to the business concerns net profits distributed among the
shareholders. It may also be termed as the part of the profit of a business
concern, which is distributed among its shareholders.
According to the Institute of Chartered Accountant of India , dividend
is defined as "a distribution to shareholders out of profits or reserves
available for this purpose".
The term dividend refers to that portion of profit (after tax) which is
distributed among the owners/shareholder s of the company and the profit
which is not distributed is known as retained earnings. A company may
have preference share capital as well as equity share capital and dividends
may be paid on both types of capital. However, there is as such, no
decision i nvolved as far as the dividend payable to preference
shareholders is concerned. The preference dividend is more or less, a
contractual liability and is payable at a fixed rate. On the other hand, a
firm has to consider a whole lot of factors before decidin g for the equity
dividend. The expected level of cash dividend, from the point of view of
equity shareholders, is the key variable from which the shareholders and
equity investors determine the share value.
10.3 TYPES OF DIVIDEND/ FORM OF DIVIDEND
Dividend may be distributed among the shareholders in the form of cash
or stock. Hence, Dividends are classified into:
A. Cash dividend
B. Stock dividend
C. Bond dividend
D. Property dividend
A. Cash Dividend
If the dividend is paid in the form of cash to the shareholders, it is call ed
cash dividend. It is paid periodically out the business concerns EAIT
(Earnings after interest and tax). Cash dividends are common and popular
types followed by majority of the business concerns.
munotes.in

Page 149


Dividend Policy

149 B. Stock Dividend
Stock dividend is paid in the form of the company stock due to raising of
more finance. Under this type, cash is retained by the business concern.
Stock dividend may be bonus issue. This issue is given only to the
existing shareholders of the business concern.
C. Bond Dividend
Bond dividend is also known as script dividend. If the company does not
have sufficient funds to pay cash dividend, the company promises to
pay the shareholder at a future specific date with the help of issue of
bond or notes.
D. Property Dividend
Property dividends are paid in th e form of some assets other than cash. It
will be distributed under the exceptional circumstance. This type of
dividend is not published in India.
10.4 THE DIVIDEND PAYMENT TIME LINE
Dividends in publicly traded firms are usually set by the board of directors
and paid out to stockholders a few weeks later. There are several key dates
between the times the board declares the dividend until the dividend is
actually paid.
1. The first date of note is the dividend declaration date, the date on
which the board of direct ors declares the dollar dividend that will be
paid for that quarter (or period). This date is important because by
announcing its intent to increase, decrease, or maintain dividend,
the firm conveys information to financial markets. Thus, if the
firm chang es its dividends, this is the date on which the market
reaction to the change is most likely to occur.
2. The next date of note is the ex -dividend date, at which time investors
have to have bought the stock in order to receive the dividend. Since
the dividend is not received by investors buying stock after the ex -
dividend date, the stock price will generally fall on that day to reflect
that loss.
3. At the close of the business a few days after the ex -dividend date, the
company closes its stock transfer books and makes up a list of the
shareholders to date on the holder -of-record date. These shareholders
will receive the dividends. There should be generally no price effect
on this date.
4. The final step involves mailing out the dividend checks on the
dividend paymen t date. In most cases, the payment date is two to
three weeks after the holder -of-record date. While stockholders may
view this as an important day, there should be no price impact on this
day either. The following Figure presents these key dates on a time
line: munotes.in

Page 150


Financial Management
150

Dividend Policy
What happens to the value of the firm as dividend is increased, holding
everything else (capital budgets, borrowing) constant. Thus, it is a trade -
off between retained earnings on one hand, and distributing cash or
securities on t he other. The establishment and determination of an
effective dividend policy is therefore, of significant importance to the
firm's overall objective. However, the development of such a policy is not
an easy job. A whole gamut of considerations affecting the dividend
decision is thee. The dividend decision may seem to be simple enough, but
it evokes a surprising amount of controversy.
The dividend decision is one of the three basis decisions which a financial
manager is required to take, the other two being the investment decisions
and the financing decisions. In each period any earning that remains after
satisfying obligations to the creditors, the Government, and the preference
shareholders can either be retained, or paid out as dividends or bifurcated
between retained earnings and dividends. The retained earnings can then
be invested in assets which will help the firm to increase or at least
maintain its present rate of growth. The dividend decision requires a
financial manager to decide about the distribu tion of profits as dividends.
It may be noted that the profits may be distributed either in the form of
cash dividends to shareholders or in the form of stock dividends (also
known as bonus shares).
In dividend decision, a financial manger is concerned to decide one or
more of the followings:
1. Should the profits be ploughed back to finance the investment
decisions?
2. Whether any dividend be paid?
3. How much dividends be paid?
4. When these dividends be paid?
5. In what form the dividends be paid?
All these decisions are inter-related and have bearing on the future growth
plans of the company. If a company pays dividends, it affects the cash
flow position of the firm but earns goodwill among the investors who
therefore, may be willing to provide additional funds for the financing of
investment plans of the firm. On the other hand, the profits which are not
distributed as dividends become an easily available source of funds at no munotes.in

Page 151


Dividend Policy

151 explicit costs. However, in the case of ploughing back of profits, the firm
may loose the goo dwill and confidence of the investors and may also defy
the standards set by other firms. Therefore, in taking the dividend
decision, the financial manage has to consider and analyze various factors.
Every aspects of dividend decision is to be critically e valuated. The most
important of these considerations is to decide as to what portion of profit
should be distributed.
This is also known as the dividend payout ratio. While deciding the
dividend payout ratio the firm should consider the effect of such poli cy on
the objective of maximization of shareholder's wealth. If payment of
dividend is expected to increase the market value of the share (i.e. increase
in the wealth of the shareholders) the dividend must be paid, otherwise,
the profits may be retained an d used as an internal source of finance. So,
the firm must find out and establish a relationship between the dividend
policy and the market value of the share.
10.5 DIVIDEND DECISION
Dividend decision of the business concern is one of the crucial parts of the
financial manager, because it determines the amount of profit to be
distributed among shareholders and amount of profit to be treated as
retained earnings for financing its long term growth. Hence, dividend
decision plays very important part in the financi al management. Dividend
decision consists of two important concepts which are based on the
relationship between dividend decision and value of the firm.
Irrelevance of Dividend: According to professors Soloman, Modigliani
and Miller, dividend policy has no effect on the share price of the
company. There is no relation between the dividend rate and value of the
firm. Dividend decision is irrelevant of the value of the firm. Modigliani
and Miller contributed a major approach to prove the irrelevance dividend
concept. Modigliani and Miller's Approach: According to MM, under
a perfect market condition, the dividend policy of the company is
irrelevant and it does not affect the value of the firm. "Under conditions
of perfect market, rational investors, absence of tax discrimination
between dividend income and capital appreciation, given the firm's
investment policy, its dividend policy may have no influence on the
market price of shares". MM approach is based on the following
important assumptions:
1. The first assum ption is the existence of a perfect market in which all
investors are rational. In perfect market condition there is easy access
to information and the floatation and the transaction costs do not exist.
The securities are infinitely divisible and hence no single investor is
large enough to influence the share value.
2. Secondly, it is assumed that there are no taxes implying that there are
no differential tax rates for the dividend income and the capital gain.
3. Thirdly, a firm has a given investment policy whic h does not change.
The operational implication of this assumption is that financing of new munotes.in

Page 152


Financial Management
152 investments out of retained earnings will not change the business risk
complexion of the firm and, therefore, there would be no change in the
required rate of return .
4. Finally, it was also assumed that the investors are able to forecast the
future earnings, the dividends and the share value of the firm with
certainty. This assumption was however, dropped out of the model.
In order to testify their model, MM has started with the following
valuation model:
Step I: The market price of a share in the beginning of the period is
equal to the present value of dividends paid at the end of the period plus
the market price of share at the end of the period. Symbolically,

Where P0 = Prevailing market price of a share Ke = Cost of equity capital
D1 = Dividend to be received at the end of period 1 P1 = Market price of a
share at the end of period 1
Step 2: Assuming no external financing, the total capitalised value of the
firm woul d be simply the number of shares (n) times the price of each
share (P0). Thus,

Step 3: If the firm's internal sources of financing its investment
opportunities fall short of the funds required, and Δn is the number of new
shares issued at the end of year 1 at price of P 1, Eq. 2 can be written as:

Equation 3 implies that the total value of the firm is the capitalized value
of the dividends to be received during the period plus the value of the
numbe r of shares outstanding at the end of the period, considering new
shares, less the value of the new shares. Thus, in effect, Eq. 3 is equivalent
to Eq. 2.
Step 4: If the firm were to finance all investment proposals, the total
amount raised through new shares issued would be given in Eq. 4
munotes.in

Page 153


Dividend Policy

153
According to Equation 4, whatever investment needs (I) are not financed
by retained earnings, must be financed through the sale of additional
equity shares.
Step 5: If we substitute Eq. 4 into Eq. 3 we derive Eq. 5

Step -6: Since dividends (D) are not found in Eq. 6, Modigliani and
Miller conclude that dividends do not count and that dividend policy has
no effect on the share price.
MM's approach to irrelevance of dividend to valuation is illustrated here:
Illustrati on 1: A company belongs to a risk class for which the
approximate capitalisation rate is 10 per cent. It currently has
outstanding 25,000 shares selling at Rs. 100 each. The firm is
contemplating the declaration of a dividend of Rs. 5 per cent share at the
end of the current financial year. It expects to have a net income of Rs.
2,50,000 and has a proposal for making new investments of Rs. 5,00,000.
Show that under the MM assumption, the payment of dividend does not
affect the value of the firm.
munotes.in

Page 154


Financial Management
154 Solut ion (a) Value of the firm, when dividends are paid:
Thus, whether dividends are paid or not, value of the firm remains the
same.
The above example clearly demonstrates that the shareholders are
indifferent between the retention of profits and the payment of dividend.
Criticism of MM approach
MM approach consists of certain criticisms also. The following are the
major criticisms of MM approach:
1. MM approach assumes that tax does not exist. It is not applicable in
the practical life of the firm.
munotes.in

Page 155


Dividend Policy

155 2. MM approach assumes that, there is no risk and uncertain of the
investment. It is also not applicable in present day business life.
3. MM approach does not consider floatation cost and transaction
cost. It leads to affect the value of the firm.
4. MM approach considers only s ingle decrement rate, it does not exist
in real practice.
5. MM approach assumes that, investor behaves rationally. But we
cannot give assurance that all the investors will behave rationally.
10.6 RELEVANCE OF DIVIDEND POLICY
The relevance approach propounds that the dividend policy has an effect
on the market value of the share and the value of the firm. The market
price of the share will increase if the firm pays dividends, otherwise it may
decrease. A firm therefore, must pay a dividend to shareholders to fulfill
the expectations of the shareholders in order to maintain or increase the
market price of the share. Two models representing this argument may be
discussed here:
A. Walter Model
The dividend policy given by James E Walter considers that dividends are
relev ant and they do affect the share price. In this model he studied the
relationship between the internal rate of return
(r) and the cost of capital of the firm (k), to give a dividend policy that
maximizes the shareholders' wealth.
According to Walter a firm can maximize the market value of its share and
the value of the firm by adopting a dividend policy as follows:
(i) If r > ke, the payout ratio should be zero (i.e., retention of 100%
profit).
(ii) If r < ke, the payout ratio should be 100% and the firm should not
retain any profit, and
(iii) If r = ke, the dividend is irrelevant and the dividend policy is not
expected to affect the market value of the share.
Assumptions: The relevance of the dividend policy as explained by the
Walter's Model is based on a few assumptions , which are as follows:
a. A retained earnings is the only source of finance available to the firm,
with no outside debt or additional equity used.
b. r and k are assumed to be constant and thus additional investments
made by the firm will not change its risk an d return profiles.
c. Firm has an infinite life. munotes.in

Page 156


Financial Management
156 d. For a given value of the firm, the dividend per share and the
earnings per share remain constant.
In order to testify, Walter has suggested a mathematical valuation model
i.e.

Where
P = Market price of Equity share
D = Dividend per share paid by the Firm
r = Rate of return on Investment of the Firm ke = Cost of Equity share
capital, and
E = Earnings per share of the firm
As per the above formula, the market price of a share is the sum of two
components i.e.,
(i) The present value of an infinite stream of dividends, and
(ii) The present value of an infinite stream of return from retained
earnings.
Thus, the Walter's formula shows that the market value of a share is
the present value of the expected stream of dividends and capital
gains. The effect of varying payout ratio on the market price of the
share under different rate of returns, r, have been shown in Example
Illustration 2: Given the following information about ABC Ltd., show
the effect of the dividend policy on the market price of its shares, using the
Walter's model:
Equity capitalization rate (ke) = 12%, Earnings per share (E) = Rs.
8, Assumed return on investments (r) as follows: (i) r = 15%, (ii) r =
10%, (iii) r = 12%
Solution: To show the effect of the diffe rent dividend policies on the
share value of the firm for the three levels of r let us consider the
dividend pay-out (D/P) ratios of zero, 25%, 50%, 75% and 100%.
munotes.in

Page 157


Dividend Policy

157


Interpretation: From the above calculations it can be observed that when
the return on investment is greater than the cost of capital, there is an
inverse relation between the value of the share and the pay -out ratio.
Thus, the value of ABC Ltd. is the highest when the D/P ratio is zero
(P=Rs.83) and this goes on declining as the D/P ratio inc reases. Hence the
optimum dividend policy for a growth firm is a zero dividend pay-out
ratio.
(ii) r < ke (r=10%, ke = 12%)
a. D/P ratio = 0 ; dividend per share = zero

munotes.in

Page 158


Financial Management
158

Interpretation: When the return on investment is less than the cost of
equity capital, the calculations reveal that the firm's value will enhance as
the D/P ratio increase. Due to this positive correlation between the share
price and the dividend pay -out ratio, firms which have their return on
investment less than the cost of equity capital should prefer a higher
dividend pay-out ratio in order to maximize the share value.



munotes.in

Page 159


Dividend Policy

159 Interpretation: In the final case where the firm has its' return on
investment equal to the cost of equity capital, the dividend policy does
not affect the share price of the firm. The price of the firm remains Rs. 67
for all the given levels of the D/P ratio. However, in actual practice r and k
will not be the same and it can only be hypothetical case. Excepting the
hypothetical cases of r
= k e in other cases where rk e, according to Walter model the
dividend policy of a firm, as shown above is relevant for maximizing the
share price of the firm.
Limitations of the Walter's Model
Most of the limitations for this model arise due to the assumptions made.
The first assumption of exclusive financing by retained earnings is to
make the model suitable only for all-equity firms.
Secondly, Walter assumes the return on investments to be constant. This
again will not be true for firms making high investments.
Finally, Walte r's model on dividend policy ignores the business risk of
the firm which has a direct impact on the value of the firm.
Thus, k cannot be assumed to be constant.
B. Gordon's Dividend Capitalization Model
Yet another model that has given importance to the divid end policy of the
firm is the Gordon Model. Myron Gordon used the dividend capitalization
approach to study the effect of the firm's dividend policy on the stock
price. The model is however, based on the following assumptions:
1. The firm will be an all -equit y firm with the new investment proposals
being financed solely by the retained earnings.
2. Return on investment (r) and the cost of equity capital (ke) remain
constant.
3. Firm has an infinite life.
4. The retention ratio remains constant and hence the growth rate also is
constant (g=br).
5. k > br i.e. cost of equity capital is greater than the growth rate.
Gordon's dividend capitalization model gives the value of the stock as
follows:

munotes.in

Page 160


Financial Management
160 Where, P = Share price, E = Earnings per share, b = Retention ratio, (1 -b)
= Div idend pay -out ratio, k e = Cost of equity capital (or cost capital of
firm), br = Growth rate (g) in the rate of return on investment.
This mode shows that there is a relationship between payout ratio (i.e., 1 -
b), cost of capital k e, rate of return, r, and the market value of the share.
Illustration 3: The following information is available in respect of XYZ
Ltd: Earnings per share = Rs. 10 (Constant)
Cost of Capital, ke, = .10 (Constant)
Find out the market price of the share under as per Gordon Model
diffe rent rate of return, r, of 8%, 10% and 15% for different payout ratios
of 0%, 40%, 80% and 100%.
Solution: The market price of the share as per Gordon's model may be
calculated as follows:
If r =15% and payout ratio is 40%, then the retention ratio b is .6 (i.e. 1-
.4) and the growth rate, g=br=.09 (i.e., 6×.15) and the market price of the
share is:


Similarly, the expected market price under different combinations of 'r'
and dividend payout ratio have been calculated and placed in Table given
here:
Market Price under Gordon's Model for Different Combinations of 'r'
and Payout Ratio


munotes.in

Page 161


Dividend Policy

161 It is clear from the table that if the firm adopts a zero payout then the
investor may not be willing to offer any price. For a growth firm (i.e., r>ke
>br), the market pric e decreases when the payout ratio is increased. For a
firm having rincreased.
If r = ke, the dividend policy is irrelevant and the market price remains
constant at Rs. 100 only. However, in his revis ed model, Gordon has
argued that even if r = ke, the dividend payout ratio matters and the
investors being risk averse prefer current dividends which are certain to
future capital gains which are uncertain. The investors will apply a higher
capitalization rate i.e., ke to discount the future capital gains. This will
compensate them for the future uncertain capital gain and thus, the market
price of the share of a firm which retains profit will be adversely affected.
Thus, Gordon's conclusion about the relat ionship between the dividend
policy and the value of the firm are similar to that of Walter's model. The
similarity is due to the reason that the underlying assumptions of both the
models are same.
10.7 DETERMINANTS OF DIVIDEND POLICY
The payment of dividend de cision has to be taken considering the special
circumstances of an individual case. The factors which determine the
dividend policy are as follow:
1. Dividend Payout (D/P) Ratio: A major aspect of the dividend policy
of a firm is its dividend payout (D/P) rat io, that is, the percentage
share of the net earnings distributed to the shareholders as dividends.
In other words, the dividend policy of the firm affects both the
shareholders' wealth and the long -term growth of the firm. The
optimum dividend policy shou ld strike the balance between current
dividends and future growth which maximises the price of the firm's
shares. The D/P ratio of a firm should be determined with reference to
two basic objectives – maximising the wealth of the firm's owners and
providing sufficient funds to finance growth.
2. General State of Economy : In the following cases, the business
may prefer to retain the whole or part of the earnings in order to build
up reserves: (a) where there is uncertain economic and business
conditions; (b) if there is a period of depression (management may
withhold the payment of dividends for maintaining the liquidity
position of the firm.) ; (c) if there is a period of prosperity (since there
is large profitable investment opportunities) ; (d) where there is a
period of inflation.
3. Capital Market Considerations: If easy access to the capital market
is possible whether due to financially strong or, big in size, the firm in
that case, may adopt a liberal dividend policy. In the opposite case,
i.e., if easy acces s to capital market is not possible, it must have to
adopt a low dividend payout ratio, i.e., they have to follow a
conservative dividend policy. As such, they must have to rely more on
their own funds, viz., retained earnings. munotes.in

Page 162


Financial Management
162 4. Legal, Contractual, Internal Constraints and Restrictions: The
dividend decision is also affected by certain legal, contractual, and
internal requirements and constraints. The Companies Act has put
several restrictions regarding payments and declaration of dividends.
Similarly, Incom e Tax Act, 1961 also lays down certain restrictions
on payment of dividends.
5. Profitable Position of the Firm: Dividend decision depends on the
profitable position of the business concern. When the firm earns more
profit, they can distribute more dividends to the shareholders. The
stability of earnings also has a significant bearing on the dividend
decision of a firm. Generally, the more stable the income stream, the
higher is the dividend payout ratio.
6. Uncertainty of Future Income: Future income is a very i mportant
factor, which affects the dividend policy. When the shareholder needs
regular income, the firm should maintain regular dividend policy.
7. Liquidity Position: Liquidity position of the firms leads to easy
payments of dividend. If the firms have high liquidity, the firms can
provide cash dividend otherwise, they have to pay stock dividend.
8. Sources of Finance: If the firm has finance sources, it will be easy to
mobilise large finance. The firm shall not go for retained earnings.
9. Growth Rate of the Firm: High growth rate implies that the firm can
distribute more dividends to its shareholders. The firm is required to
make plans for financing its expansion programmes and growth.
10. Tax Policy: Tax policy of the government also affects the dividend
policy of th e firm. When the government gives tax incentives, the
company pays more dividends. If a firm has a large percentage of
owners who are in high tax brackets, its dividend policy should seek
to have higher retentions. Such a policy will provide its owners wit h
income in the form of capital gains as against dividends.
11. Financial Requirements: Financial requirements of a firm are
directly related to its investment needs. If a firm has abundant
investment opportunities, it should prefer a low payout ratio, as it c an
usually reinvest earnings at a higher rate than the shareholders can.
Such firms, designated as 'growth' companies, are constantly in need
of funds.
12. Control: The management, in order to retain control of the company
in its own hands, may be reluctant to pay substantial dividends and
would prefer a smaller dividend payout ratio. This will particularly
hold good for companies which require funds to finance profitable
investment opportunities when an outside group is seeking to gain
control of the firm. Dilution in earnings results because low retentions
may necessitate the issue of new equity shares in the future, causing
an increase in the number of equity shares outstanding and ultimately
lowering earnings per share and their price in the market. By retai ning munotes.in

Page 163


Dividend Policy

163 a high percentage of its earnings, the firm can minimize the
possibility of dilution of earnings.
13. Owner's Considerations: The dividend policy is also likely to be
affected by the owner's considerations of (a) the tax status of the
shareholders, (b) th eir opportunities of investment, and (c) the
dilution of ownership. It is well -nigh impossible to establish a policy
that will maximise each owner's wealth.
14. Opportunities of Investment: If evaluation shows that the owners
have better opportunities outside, the firm should opt for a higher
dividend payout ratio. On the other hand, if the firm's investment
opportunities yield a higher rate than that obtained from similar
external investment, a low dividend payout is suggested.
15. Inflation: With rising prices, f unds which are generated by way of
depreciation may fall short in order to replace obsolete equipment.
The shortfall may be made from retained earnings (as a source of
funds). This is very significant when the assets are to be replaced in
the near future. As such, the dividend payout ratio tends to be low
during the periods of inflation.
16. Stability of Dividends: The dividend policy, of course, should have a
degree of stability, i.e., earnings/profits may fluctuate from year to
year but not the dividend since the equity shareholders prefer to value
stable dividends than the fluctuating ones. In other words, the
investors favour a stable dividend in as much as they do the payment
of dividend.
10.8 QUESTIONS:
1. Explain the various factors which influence the dividend decision of a
firm.
2. "A firm should follow a policy of very high dividend pay-out". Do
you agree? Why or why not?
3. What do you understand by a stable dividend policy? Why should it be
followed?
6. Discuss the various forms of dividends.
7. Explain the irrelevance and relevance dividend theories.
8. State the criticism of MM approach.
9. What are the assumptions of Walter's model?
10. What are the assumptions and criticisms of Gordon's model?
11. "Walter's and Gordon's models are based on the same assumptions.
Thus, there is no b asic difference between the two models". Do you
agree or not? Why? munotes.in

Page 164


Financial Management
164 12. "The assumptions underlying the irrelevance hypothesis of Modigliani
and Miller are unrealistic". Explain.
13. Explain the Modigliani -Miller hypothesis of dividend irrelevance.
Does this hypoth esis suffer from deficiencies?
14. The earnings per share of a company are Rs. 10. It has rate of return of
15% and the capitalization rate of risk class is 12.5%. If Walter's
model is used: (i) What should be the optimum payout ratio of the
firm? (ii) What would be the price of the share at this payout?
(iii) How shall the price of the share be affected if a different payout was
employed?
15. U Ltd. belongs to risk class of capitalization rate which is 14%. It has
currently 3000 shares outstanding at Rs. 50 each; during the year Rs. 5
is declared as dividend. The net income of the company is Rs. 83,000.
For the new project investment is required of Rs. 1,20,000. Calculate
under MM hypothesis that the payment of dividend does not affect the
value of the firm. (Ans. divide nd paid Rs. 52 number of equity shares
1000 and value of the firm Rs. 1,50,000. Dividend not paid Rs. 57.
Number of equity shares 37000/57 shares (approx. 650 shares) Value
of the firm is Rs. 1,50,000)
16. X Ltd., had 25,000 equity shares of Rs. 100 each outst anding on 1st
April, the shares are issued at par in the market, the company
removed restraint in the dividend policy, the company ready to pay
dividend of Rs. 15 per share for the current calendar year. The
capitalization rate is 15%. Using MM approach as suming that no
taxes, calculate the price of the shares at the end of the year:
(a) When dividend is not declared.
(b) When dividend is declared.
(c) Find out the number of new shares that the company issues to
meet its investment needs of Rs 15,00,000 assuming that n et
income of Rs. 7,50,000 and assuming that the dividend is paid.
(Ans. (a) Rs.105 (b) Rs.115 (c) 10,000 shares)
17. The following information is available in respect of a company‘s
capitalization rate is 15% earnings per share Rs. 75. Assured rate on
investme nt is 14%, 12%, 10%. The effect of dividend policy on market
price of shares applying Walter's model the dividend payout ratio is
(a) 0% (b) 40% (c) 60% (d) 100%)
18. The following data are available for R Ltd.
— Earnings per share Rs. 8
— Rate of return on invest ment 16% — Rate of return to shareholders
12%
munotes.in

Page 165


Dividend Policy

165 If Gordon's basic valuation formula is applied what will be the price per
share when the dividend payout ratio is 25%, 50%, 60% and 100%.
(Ans. Rs. 0, 100, 85.71, and 66.67)
10.9 REFERENCES
 Financial Management by Prasanna Chandra.
 Financial Management by I.M. Pandey.
 Financial Management by Khan & Jain.
 Organization & Management by R.D. Aggarwal.
 Financial Management and Policy by R.M. Srivastava.


munotes.in